Lewis: MED-SURG: Chapter 50: Endocrine Problems, Lewis Chapter 48, Endocrine, Lewis Chapter 50, Lewis: Chapter 50: Endocrine Problems, Lewis: Chapter 50 : DI & SIADH, Lewis: Chapter 50, Endocrine: Lewis: chapter 50, Lewis Ch 50 Nursing Management: En...

अब Quizwiz के साथ अपने होमवर्क और परीक्षाओं को एस करें!

What does a fruity breath indicate?

A high blood sugar

Which of the following symptoms is not typical of Cushing's syndrome? Answers: A. Osteoporosis B. Weight loss C. Diabetes D. Mood instability

B Cushing's syndrome tends to produce rapid weight gain, not weight loss.

What organs are affected by metabolic syndrome?

Brain, hear, and pancreas.

Following a thyroidectomy, a patient develops carpal spasm while the nurse is taking a blood pressure on the left arm. Which action by the nurse is appropriate?

b. Have the patient rebreathe using a paper bag.

The first nursing action indicated when a patient returns to the surgical nursing unit following a thyroidectomy is to

b. assess respiratory rate and effort

The nursing instructor asks a student to describe the pathophysiology that occurs in Cushing's disease. Which statement by the student indicates an accurate understanding of this disorder?

"Cushing's disease is characterized by an oversecretion of glucocorticoid hormones."

The nurse is reinforcing discharge teaching with a client who has Cushing's syndrome. Which statement by the client indicates that the instructions related to dietary management were understood?

"I can eat foods that contain potassium."

The nurse has reinforced instructions to the client with hyperparathyroidism regarding home care measures related to exercise. Which statement by the client indicates a need for further teaching? Select all that apply.

"I need to limit playing football to only the weekends." "I should exercise in the evening to encourage a good sleep pattern."

The nurse is reinforcing home care instructions to a client with a diagnosis of Cushing's syndrome. Which client statement reflects a need for further teaching?

"I need to read the labels on any over-the-counter medications I purchase."

The nurse is instructing a client with Addison's disease about a newly prescribed medication, fludrocortisone acetate (Florinef). Which statement by the client indicates a need for further teaching?

"I will be glad to gain weight."

The client has developed iatrogenic Cushing's disease. Which statement is the scientific rationale for the development of this diagnosis? 2. The client has been taking steroid medications for an extended period for another disease process

"Iatrogenic" means a problem has been caused by a medical treatment or procedure

A client with pheochromocytoma is scheduled for surgery and says to the nurse, "I'm not sure that surgery is the best thing to do." Which response by the nurse is appropriate?

"You have concerns about the surgical treatment for your condition?"

A nurse is reviewing discharge teaching with a client who has Cushing's syndrome. Which statement by the client indicates that the instructions related to dietary management were understood? 1. "I can eat foods that contain potassium." 2. "I will need to limit the amount of protein in my diet." 3. "I am fortunate that I can eat all the salty foods I enjoy." 4. "I am fortunate that I do not need to follow any special diet."

*1. "I can eat foods that contain potassium."* *rationale* A diet that is low in calories, carbohydrates, and sodium but ample in protein and potassium content is encouraged for a client with Cushing's syndrome. Such a diet promotes weight loss, the reduction of edema and hypertension, the control of hypokalemia, and the rebuilding of wasted tissue.

Which statement by the client would cause the nurse to suspect that the thyroid test results drawn on the client this morning may be inaccurate? 1. "I had a radionuclide test done 3 days ago." 2. "When I exercise I sweat more than normal." 3. "I drank some water before the blood was drawn." 4. "That hamburger I ate before the test sure tasted good."

*1. "I had a radionuclide test done 3 days ago."* *rationale* Option 1 indicates that a recent radionuclide scan had been performed. Recent radionuclide scans performed before the test can affect thyroid laboratory results. No food, fluid, or activity restrictions are required for this test, so options 2, 3, and 4 are incorrect.

A client has just been admitted with a diagnosis of myxedema coma. If all of the following interventions were prescribed, the nurse would place highest priority on completing which of the following first? 1. Administering oxygen 2. Administering thyroid hormone 3. Warming the client 4. Giving fluid replacement

*1. Administering oxygen* *rationale* As part of maintaining a patent airway, oxygen would be administered first. This would be quickly followed by fluid replacement, keeping the client warm, monitoring vital signs, and administering thyroid hormones.

A nurse is collecting data on a client with a diagnosis of hypothyroidism. Which of these behaviors, if present in the client's history, would the nurse determine as being likely related to the manifestations of this disorder? 1. Depression 2. Nervousness 3. Irritability 4. Anxiety

*1. Depression* *rationale* Hypothyroid clients experience a slow metabolic rate, and its manifestation includes apathy, fatigue, sleepiness, and depression. Options 2, 3, and 4 identify the clinical manifestations of hyperthyroidism.

A nurse is reviewing a plan of care for a client with Addison's disease. The nurse notes that the client is at risk for dehydration and suggests nursing interventions that will prevent this occurrence. Which nursing intervention is an appropriate component of the plan of care? *Select all that apply.* 1. Encouraging fluid intake of at least 3000 mL/day 2. Encouraging an intake of low-protein foods 3. Monitoring for changes in mental status 4. Monitoring intake and output 5. Maintaining a low-sodium diet

*1. Encouraging fluid intake of at least 3000 mL/day* *3. Monitoring for changes in mental status* *4. Monitoring intake and output* *rationale* The client at risk for deficient fluid volume should be encouraged to eat regular meals and snacks and to increase the intake of sodium, protein, and complex carbohydrates. Oral replacement of sodium losses is necessary, and maintenance of adequate blood glucose levels is required.

A client with Addison's disease asks the nurse how a newly prescribed medication, fludrocortisone acetate (Florinef), will improve the condition. When formulating a response, the nurse should incorporate that a key action of this medication is to: 1. Help restore electrolyte balance. 2. Make the body produce more cortisol. 3. Replace insufficient circulating estrogens. 4. Alter the body's immune system functioning.

*1. Help restore electrolyte balance.* *rationale* Fludrocortisone acetate is a long-acting oral medication with mineralocorticoid and moderate glucocorticoid activity. It is prescribed for the long-term management of Addison's disease. Mineralocorticoids cause renal reabsorption of sodium and chloride ions and the excretion of potassium and hydrogen ions. These actions help restore electrolyte balance in the body. The other options are incorrect.

In planning nutrition for the client with hypoparathyroidism, which diet would be appropriate? 1. High in calcium and low phosphorous 2. Low in vitamins A, D, E, and K 3. High in sodium with no fluid restriction 4. Low in water and insoluble fiber

*1. High in calcium and low phosphorous* *rationale* Hypocalcemia is the end result of hypoparathyroidism resulting from either a lack of parathyroid hormone (PTH) secretion or ineffective PTH influence on tissue. Calcium is the major controlling factor of PTH secretion. Because of this, the diet needs to be high in calcium but low in phosphorus because these two electrolytes must exist in inverse proportions in the body. The other options are not dietary interventions with hypoparathyroidism.

A client with hypoparathyroidism has hypocalcemia. The nurse avoids giving the client the prescribed vitamin and calcium supplement with which of the following liquids? 1. Milk 2. Water 3. Iced tea 4. Fruit juice

*1. Milk* *rationale* Milk products are high in phosphates, which should be avoided by a client with hypoparathyroidism. Otherwise, calcium products are best absorbed with milk because the vitamin D in the milk promotes calcium absorption.

A nurse notes in the medical record that a client with Cushing's syndrome is experiencing fluid overload. Which interventions should be included in the plan of care? *Select all that apply.* 1. Monitoring daily weight 2. Monitoring intake and output 3. Maintaining a low-potassium diet 4. Monitoring extremities for edema 5. Maintaining a low-sodium diet

*1. Monitoring daily weight* *2. Monitoring intake and output* *4. Monitoring extremities for edema* *5. Maintaining a low-sodium diet* *rationale* The client with Cushing's syndrome experiencing fluid overload should be maintained on a high-potassium and low-sodium diet. Decreased sodium intake decreases renal retention of sodium and water. Monitoring weight, intake, output, and extremities for edema are all appropriate interventions for such a nursing diagnosis.

A nurse is caring for a client with a diagnosis of hypoparathyroidism. The nurse reviews the laboratory results drawn on the client and notes that the calcium level is extremely low. The nurse would expect to note which of the following on data collection of the client? 1. Positive Trousseau's sign 2. Negative Chvostek's sign 3. Unresponsive pupils 4. Hyperactive bowel sounds

*1. Positive Trousseau's sign* *rationale* Hypoparathyroidism is related to a lack of parathyroid hormone secretion or to a decreased effectiveness of parathyroid hormone on target tissues. The end result of this disorder is hypocalcemia. When serum calcium levels are critically low, the client may exhibit positive Chvostek's and Trousseau's signs, which indicate potential tetany. Options 2, 3, and 4 are not related to the presence of hypocalcemia.

A health care provider has prescribed propylthiouracil (PTU) for a client with hyperthyroidism, and the nurse assists in developing a plan of care for the client. A priority nursing measure to be included in the plan regarding this medication is to monitor the client for: 1. Signs and symptoms of hypothyroidism 2. Signs and symptoms of hyperglycemia 3. Relief of pain 4. Signs of renal toxicity

*1. Signs and symptoms of hypothyroidism* *rationale* Excessive dosing with propylthiouracil may convert the client from a hyperthyroid state to a hypothyroid state. If this occurs, the dosage should be reduced. Temporary administration of thyroid hormone may be required. Propylthiouracil is not used for pain and does not cause hyperglycemia or renal toxicity.

A nurse is preparing to discharge a client who has had a parathyroidectomy. When teaching the client about the prescribed oral calcium supplement, what information should the nurse include? 1. Take the calcium 30 to 60 minutes following a meal. 2. Avoid sunlight because it can cause skin color change. 3. Store the calcium in the refrigerator to maintain potency. 4. Check the pulse daily and hold the dosage if it is below 60 beats per minute.

*1. Take the calcium 30 to 60 minutes following a meal.* *rationale* Oral calcium supplements can be taken 30 to 60 minutes after meals to enhance their absorption and decrease gastrointestinal irritation. All the other options are unrelated to oral calcium therapy.

An older client with a history of hyperparathyroidism and severe osteoporosis is hospitalized. The nurse caring for the client plans first to address which problem? 1. The possibility of injury 2. Constipation 3. Urinary retention 4. Need for teaching about the disorder

*1. The possibility of injury* *rationale* The client with severe osteoporosis as a result of hyperparathyroidism is at risk for injury as a result of pathological fractures that can occur from bone demineralization. The client may also have a risk for constipation from the disease process but this is a lesser priority than client safety. The client may or may not have urinary elimination problems, depending on other factors in the client's history. There is no information in the question to support whether the client needs teaching.

A nurse is monitoring a client following a thyroidectomy for signs of hypocalcemia. Which of the following signs, if noted in the client, likely indicates the presence of hypocalcemia? 1. Tingling around the mouth 2. Negative Chvostek's sign 3. Flaccid paralysis 4. Bradycardia

*1. Tingling around the mouth* *rationale* Following a thyroidectomy, the nurse assesses the client for signs of hypocalcemia and tetany. Early signs include tingling around the mouth and fingertips, muscle twitching or spasms, palpitations or dysrhythmias, and positive Chvostek's and Trousseau's signs. Options 2, 3, and 4 are not signs of hypocalcemia.

During routine postoperative assessment of a client who has undergone hypophysectomy, the client complains of thirst and frequent urination. Knowing the expected complications of this surgery, the nurse would next check the: 1. Urine specific gravity 2. Serum glucose 3. Respiratory rate 4. Blood pressure

*1. Urine specific gravity* *rationale* Following hypophysectomy, diabetes insipidus can occur temporarily because of antidiuretic hormone (ADH) deficiency. This deficiency is related to surgical manipulation. The nurse should assess specific gravity and notify the registered nurse if the results are less than 1.005. Although options 2, 3, and 4 may be components of the assessment, the nurse would next check urine specific gravity.

A nurse has reinforced dietary instructions to a client with a diagnosis of hypoparathyroidism. The nurse instructs the client to include which of the following items in the diet? 1. Vegetables 2. Meat 3. Fish 4. Cereals

*1. Vegetables* *rationale* The client with hypoparathyroidism is instructed to follow a calcium-rich diet and to restrict the amount of phosphorus in the diet. The client should limit meat, poultry, fish, eggs, cheese, and cereals. Vegetables are allowed in the diet.

A nurse is caring for a client with pheochromocytoma. The client is scheduled for an adrenalectomy. During the preoperative period, the priority nursing action would be to monitor the: 1. Vital signs 2. Intake and output 3. Blood urea nitrogen (BUN) level 4. Urine for glucose and acetone

*1. Vital signs* *rationale* Hypertension is the hallmark of pheochromocytoma. Severe hypertension can precipitate a brain attack (stroke) or sudden blindness. Although all of the options are accurate nursing interventions for the client with pheochromocytoma, the priority nursing action is to monitor the vital signs, particularly the blood pressure.

A nurse is collecting data on a client with hyperparathyroidism. Which of the following questions would elicit the accurate information about this condition from the client? 1. "Do you have tremors in your hands?" 2. "Are you experiencing pain in your joints?" 3. "Have you had problems with diarrhea lately?" 4. "Do you notice swelling in your legs at night?"

*2. "Are you experiencing pain in your joints?"* *rationale* Hyperparathyroidism causes an oversecretion of parathyroid hormone (PTH), which causes excessive osteoblast growth and activity within the bones. When bone reabsorption is increased, calcium is released from the bones into the blood, causing hypercalcemia. The bones suffer demineralization as a result of calcium loss, leading to bone and joint pain, and pathological fractures.

A nurse is collecting data on a client admitted to the hospital with a diagnosis of myxedema. Which data collection technique will provide data necessary to support the admitting diagnosis? 1. Auscultation of lung sounds 2. Inspection of facial features 3. Percussion of the thyroid gland 4. Palpation of the adrenal glands

*2. Inspection of facial features* *rationale* Inspection of facial features will reveal the characteristic coarse features, presence of edema around the eyes and face, and a blank expression that are characteristic of myxedema. The techniques in the remaining options will not reveal any data that would support the diagnosis of myxedema.

A nursing instructor asks a student to describe the pathophysiology that occurs in Cushing's disease. Which statement by the student indicates an accurate understanding of this disorder? 1. "Cushing's disease is characterized by an oversecretion of insulin." 2. "Cushing's disease is characterized by an oversecretion of glucocorticoid hormones." 3. "Cushing's disease is characterized by an undersecretion of corticotropic hormones." 4. "Cushing's disease is characterized by an undersecretion of glucocorticoid hormones."

*2. "Cushing's disease is characterized by an oversecretion of glucocorticoid hormones."* *rationale* Cushing's syndrome is characterized by an oversecretion of glucocorticoid hormones. Addison's disease is characterized by the failure of the adrenal cortex to produce and secrete adrenocortical hormones. Options 1 and 4 are inaccurate regarding Cushing's syndrome.

A nurse is reinforcing home care instructions to a client with a diagnosis of Cushing's syndrome. Which statement reflects a need for further client education? 1. "Taking my medications exactly as prescribed is essential." 2. "I need to read the labels on any over-the-counter medications I purchase." 3. "My family needs to be familiar with the signs and symptoms of hypoadrenalism." 4. "I could experience the signs and symptoms of hyperadrenalism because of Cushing's."

*2. "I need to read the labels on any over-the-counter medications I purchase."* *rationale* The client with Cushing's syndrome should be instructed to take the medications exactly as prescribed. The nurse should emphasize the importance of continuing medications, consulting with the health care provider before purchasing any over-the-counter medications, and maintaining regular follow-up care. The nurse should also instruct the client in the signs and symptoms of both hypoadrenalism and hyperadrenalism.

A nurse has just supervised a newly diagnosed diabetes mellitus client self-inject NPH insulin at 7:30 ᴀᴍ. The nurse reviews the time frames for peak insulin action with the client, telling the client to be especially watchful for a hypoglycemic reaction between: 1. 7:30 ᴀᴍ and 9:30 ᴀᴍ 2. 1:30 ᴘᴍ and 7:30 ᴘᴍ 3. 8:30 ᴘᴍ and 12:00 ᴀᴍ 4. 2:30 ᴀᴍ and 4:30 ᴀᴍ

*2. 1:30 ᴘᴍ and 7:30 ᴘᴍ* *rationale* NPH is an intermediate-acting insulin. It begins to work in 1 to 2 hours (onset), peaks in 6 to 12 hours, and lasts for 18 to 24 hours (duration). Hypoglycemic reactions most likely occur during peak time, which in this case is option 2.

Which of the following clients is at risk for developing thyrotoxicosis? 1. A client with hypothyroidism 2. A client with Graves' disease who is having surgery 3. A client with diabetes mellitus scheduled for debridement of a foot ulcer 4. A client with diabetes insipidus scheduled for an invasive diagnostic test

*2. A client with Graves' disease who is having surgery* *rationale* Thyrotoxicosis is usually seen in clients with Graves' disease with the symptoms precipitated by a major stressor. This complication typically occurs during periods of severe physiological or psychological stress such as trauma, sepsis, the birth process, or major surgery. It also must be recognized as a potential complication following a thyroidectomy.

A nurse is caring for a client with Addison's disease. The nurse checks the vital signs and determines that the client has orthostatic hypotension. The nurse determines that this finding relates to which of the following? 1. A decrease in cortisol release 2. A decreased secretion of aldosterone 3. An increase in epinephrine secretion 4. Increased levels of androgens

*2. A decreased secretion of aldosterone* *rationale* A decreased secretion of aldosterone results in a limited reabsorption of sodium and water; therefore the client experiences fluid volume deficit. A decrease in cortisol, an increase in epinephrine, and an increase in androgen secretion do not result in orthostatic hypotension.

While collecting data on a client being prepared for an adrenalectomy, the nurse obtains a temperature reading of 100.8° F. The nurse analyzes this temperature reading as: 1. Within normal limits 2. A finding that needs to be reported immediately 3. An expected finding caused by the operative stress response 4. Slightly abnormal but an insignificant finding

*2. A finding that needs to be reported immediately* *rationale* An adrenalectomy is performed because of excess adrenal gland function. Excess cortisol production impairs the immune response, which puts the client at risk for infection. Because of this, the client needs to be protected from infection, and minor variations in normal vital sign values must be reported so that infections are detected early, before they become overwhelming. In addition, the surgeon may elect to postpone surgery in the event of a fever because it can be indicative of infection. Options 1, 3, and 4 are not correct interpretations.

A nurse is reviewing the postoperative prescriptions for a client who had a transsphenoidal hypophysectomy. Which health care provider's prescription, if noted on the record, indicates the need for clarification? 1. Instruct the client about the need for a Medic-Alert bracelet. 2. Apply a loose dressing if any clear drainage is noted. 3. Monitor vital signs and neurological status. 4. Instruct the client to avoid blowing the nose.

*2. Apply a loose dressing if any clear drainage is noted.* *rationale* The nurse should observe for clear nasal drainage, constant swallowing, and a severe, persistent, generalized, or frontal headache. These signs and symptoms indicate cerebrospinal fluid leak into the sinuses. If clear drainage is noted following this procedure, the health care provider needs to be notified immediately. Options 1, 3, and 4 indicate appropriate postoperative interventions.

A client is admitted with a diagnosis of pheochromocytoma. The nurse would monitor which of the following to detect the most common sign of pheochromocytoma? 1. Skin temperature 2. Blood pressure 3. Urine ketones 4. Weight

*2. Blood pressure* *rationale* Hypertension is the major symptom associated with pheochromocytoma and is monitored by taking the client's blood pressure. Glycosuria, weight loss, and diaphoresis are other clinical manifestations of pheochromocytoma; however, hypertension is the most common sign.

A client has been diagnosed with hypoparathyroidism. The nurse teaches the client to include foods in the diet that are: 1. High in phosphorus and low in calcium 2. Low in phosphorus and low in calcium 3. Low in phosphorus and high in calcium 4. High in phosphorus and high in calcium

*3. Low in phosphorus and high in calcium* *rationale* Hypoparathyroidism results in hypocalcemia. A therapeutic diet for this disorder is one that is high in calcium but low in phosphorus because these two electrolytes have inverse proportions in the body. All of the other options are unrelated to this disorder and are incorrect.

During preoperative teaching for a patient scheduled for transsphenoidal hypophysectomy for treatment of a pituitary adenoma, the nurse instructs the patient about the need to

b. avoid brushing the teeth for at least 10 days after the surgery.

A client with a pituitary tumor will undergo transsphenoidal hypophysectomy. The nurse includes which priority item in the preoperative teaching plan for the client? 1. Brushing the teeth vigorously and frequently is important to minimize bacteria in the mouth. 2. Blowing the nose following surgery is prohibited. 3. A small area will be shaved at the base of the neck. 4. It will be necessary to cough and deep breathe following the surgery.

*2. Blowing the nose following surgery is prohibited.* *rationale* The approach used for this surgery is the oronasal route, specifically where the upper lip meets the gum. The surgeon then uses a route through the sphenoid sinus to get to the pituitary gland. The client is not allowed to blow the nose, sneeze, or cough vigorously because these activities could raise intracranial pressure. The client also is not allowed to brush the teeth, to avoid disrupting the surgical site. Alternate methods for performing mouth care are used.

A nurse is caring for a client diagnosed with hyperparathyroidism who is prescribed furosemide (Lasix). The nurse reinforces dietary instructions to the client. Which of the following is an appropriate instruction? 1. Increase dietary intake of calcium. 2. Drink at least 2 to 3 L of fluid daily. 3. Eat sparely when experiencing nausea. 4. Decrease dietary intake of potassium.

*2. Drink at least 2 to 3 L of fluid daily.* *rationale* The aim of treatment in the client with hyperparathyroidism is to increase the renal excretion of calcium and decrease gastrointestinal absorption and bone resorption. This is aided by the sufficient intake of fluids. Dietary restriction of calcium may be used as a component of therapy. The parathyroid is responsible for calcium production, and the term, "hyperparathyroidism" can be indicative of an increase in calcium. The client should eat foods high in potassium, especially if the client is taking furosemide. Limiting nutrients is not advisable.

When caring for a client diagnosed with pheochromocytoma, what information should the nurse know when assisting with planning care? 1. Profound hypotension may occur. 2. Excessive catecholamines are released. 3. The condition is not curable and is treated symptomatically. 4. Hypoglycemia is the primary presenting symptom.

*2. Excessive catecholamines are released.* *rationale* Pheochromocytoma is a catecholamine-producing tumor of the adrenal gland and causes secretion of excessive amounts of epinephrine and norepinephrine. Hypertension is the principal manifestation, and the client has episodes of a high blood pressure accompanied by pounding headaches. The excessive release of catecholamine also results in excessive conversion of glycogen into glucose in the liver. Consequently, hyperglycemia and glucosuria occur during attacks. Pheochromocytoma is curable. The primary treatment is surgical removal of one or both of the adrenal glands, depending on whether the tumor is unilateral or bilateral.

A client scheduled for a thyroidectomy says to the nurse, "I am so scared to get cut in my neck." Based on the client's statement, the nurse determines that the client is experiencing which problem? 1. Inadequate knowledge about the surgical procedure 2. Fear about impending surgery 3. Embarrassment about the changes in personal appearance 4. Lack of support related to the surgical procedure

*2. Fear about impending surgery* *rationale* The client is having a difficult time coping with the scheduled surgery. The client is able to express fears but is scared. No data in the question support options 1, 3, and 4.

A nurse is monitoring a client with Graves' disease for signs of thyrotoxic crisis (thyroid storm). Which of the following signs and symptoms, if noted in the client, will alert the nurse to the presence of this crisis? *Select all that apply.* 1. Bradycardia 2. Fever 3. Sweating 4. Agitation 5. Pallor

*2. Fever* *3. Sweating* *4. Agitation* *rationale* Thyrotoxic crisis (thyroid storm) is an acute, potentially life-threatening state of extreme thyroid activity that represents a breakdown in the body's tolerance to a chronic excess of thyroid hormones. The clinical manifestations include fever greater than 100° F, severe tachycardia, flushing and sweating, and marked agitation and restlessness. Delirium and coma can occur.

A nurse is caring for a client with pheochromocytoma. The client asks for a snack and something warm to drink. The appropriate choice for this client to meet nutritional needs would be which of the following? 1. Crackers with cheese and tea 2. Graham crackers and warm milk 3. Toast with peanut butter and cocoa 4. Vanilla wafers and coffee with cream and sugar

*2. Graham crackers and warm milk* *rationale* The client with pheochromocytoma needs to be provided with a diet that is high in vitamins, minerals, and calories. Of particular importance is that food or beverages that contain caffeine (e.g., chocolate, coffee, tea, and cola) are prohibited.

A nurse is caring for a client after thyroidectomy and monitoring for signs of thyroid storm. The nurse understands that which of the following is a manifestation associated with this disorder? 1. Bradycardia 2. Hypotension 3. Constipation 4. Hypothermia

*2. Hypotension* *rationale* Clinical manifestations associated with thyroid storm include a fever as high as 106° F (41.1° C), severe tachycardia, profuse diarrhea, extreme vasodilation, hypotension, atrial fibrillation, hyperreflexia, abdominal pain, diarrhea, and dehydration. With this disorder, the client's condition can rapidly progress to coma and cardiovascular collapse.

A nurse is caring for a client following an adrenalectomy and is monitoring for signs of adrenal insufficiency. Which of the following, if noted in the client, indicates signs and symptoms related to adrenal insufficiency? *Select all that apply.* 1. Double vision 2. Hypotension 3. Mental status changes 4. Weakness 5. Fever

*2. Hypotension* *3. Mental status changes* *4. Weakness* *5. Fever* *rationale* The nurse should be alert to signs and symptoms of adrenal insufficiency in a client following adrenalectomy. These signs and symptoms include weakness, hypotension, fever, and mental status changes. Double vision is generally not associated with this condition.

A client with Cushing's disease is being admitted to the hospital after a stab wound to the abdomen. The nurse plans care and places highest priority on which potential problem? 1. Nervousness 2. Infection 3. Concern about appearance 4. Inability to care for self

*2. Infection* *rationale* The client with a stab wound has a break in the body's first line of defense against infection. The client with Cushing's disease is at great risk for infection because of excess cortisol secretion and subsequent impaired antibody function and decreased proliferation of lymphocytes. The client may also have a potential for the problems listed in the other options but these are not the highest priority at this time.

A nurse is providing discharge instructions to a client who had a unilateral adrenalectomy. Which of the following will be a component of the instructions? 1. The reason for maintaining a diabetic diet 2. Instructions about early signs of a wound infection 3. Teaching regarding proper application of an ostomy pouch 4. The need for lifelong replacement of all adrenal hormones

*2. Instructions about early signs of a wound infection* *rationale* A client who is undergoing a unilateral adrenalectomy will be placed on corticosteroids temporarily to avoid a cortisol deficiency. These medications will be gradually weaned in the postoperative period until they are discontinued. Because of the anti-inflammatory properties of corticosteroids, clients who undergo an adrenalectomy are at increased risk for developing wound infections. Because of this increased risk for infection, it is important for the client to know measures to prevent infection, early signs of infection, and what to do if an infection is present. Options 1, 3, and 4 are incorrect instructions.

A nurse is caring for a postoperative parathyroidectomy client. Which of the following would require the nurse's immediate attention? 1. Incisional pain 2. Laryngeal stridor 3. Difficulty voiding 4. Abdominal cramps

*2. Laryngeal stridor* *rationale* During the postoperative period, the nurse carefully observes the client for signs of hemorrhage, which cause swelling and the compression of adjacent tissue. Laryngeal stridor is a harsh, high-pitched sound heard on inspiration and expiration that is caused by the compression of the trachea and that leads to respiratory distress. It is an acute emergency situation that requires immediate attention to avoid the complete obstruction of the airway.

After several diagnostic tests, a client is diagnosed with diabetes insipidus. The nurse understands that which symptom is indicative of this disorder? 1. Diarrhea 2. Polydipsia 3. Weight gain 4. Blurred vision

*2. Polydipsia* *rationale* Polydipsia and polyuria are classic symptoms of diabetes insipidus. The urine is pale in color, and its specific gravity is low. Anorexia and weight loss occur. Diarrhea, weight loss, and blurred vision are not manifestations of the disorder.

A nurse is collecting data from a client who is being admitted to the hospital for a diagnostic workup for primary hyperparathyroidism. The nurse understands that which client complaint would be characteristic of this disorder? 1. Diarrhea 2. Polyuria 3. Polyphagia 4. Weight gain

*2. Polyuria* *rationale* Hypercalcemia is the hallmark of hyperparathyroidism. Elevated serum calcium levels produce osmotic diuresis (polyuria). This diuresis leads to dehydration and the client would lose weight. Options 1, 3, and 4 are gastrointestinal (GI) symptoms but are not associated with the common GI symptoms typical of hyperparathyroidism (nausea, vomiting, anorexia, constipation).

The anticipated intended effect of fludrocortisone acetate (Florinef) for the treatment of Addison's disease is to: 1. Stimulate the immune response. 2. Promote electrolyte balance. 3. Stimulate thyroid production. 4. Stimulate thyrotropin production.

*2. Promote electrolyte balance.* *rationale* Florinef is a long-acting oral medication with mineralocorticoid and moderate glucocorticoid activity used for long-term management of Addison's disease. Mineralocorticoids act on the renal distal tubules to enhance the reabsorption of sodium and chloride ions and the excretion of potassium and hydrogen ions. In small doses, fludrocortisone acetate causes sodium retention and increased urinary potassium excretion. The client rapidly can develop hypotension and fluid and electrolyte imbalance if the medication is discontinued abruptly. Options 1, 3, and 4 are not associated with the effects of this medication.

What would the nurse anticipate being included in the plan of care for a client who has been diagnosed with Graves' disease? 1. Provide a high-fiber diet. 2. Provide a restful environment. 3. Provide three small meals per day. 4. Provide the client with extra blankets.

*2. Provide a restful environment.* *rationale* Because of the hypermetabolic state, the client with Graves' disease needs to be provided with an environment that is restful both physically and mentally. Six full meals a day that are well balanced and high in calories are required, because of the accelerated metabolic rate. Foods that increase peristalsis (e.g., high-fiber foods) need to be avoided. These clients suffer from heat intolerance and require a cool environment.

A nurse is caring for a client with hypothyroidism who is overweight. Which food items would the nurse suggest to include in the plan? 1. Peanut butter, avocado, and red meat 2. Skim milk, apples, whole-grain bread, and cereal 3. Organ meat, carrots, and skim milk 4. Seafood, spinach, and cream cheese

*2. Skim milk, apples, whole-grain bread, and cereal* *rationale* Clients with hypothyroidism may have a problem with being over-weight because of their decreased metabolic need. They should consume foods from all food groups, which will provide them with the necessary nutrients; however, the foods should be low in calories. Option 2 is the only option that identifies food items that are low in calories.

When caring for a client who is having clear drainage from his nares after transsphenoidal hypophysectomy, which action by the nurse is appropriate? 1. Lower the head of the bed. 2. Test the drainage for glucose. 3. Obtain a culture of the drainage. 4. Continue to observe the drainage.

*2. Test the drainage for glucose.* *rationale* After hypophysectomy, the client should be monitored for rhinorrhea, which could indicate a cerebrospinal fluid (CSF) leak. If this occurs, the drainage should be collected and tested for glucose, indicating the presence of CSF. The head of the bed should not be lowered to prevent increased intracranial pressure. Clear nasal drainage would not indicate the need for a culture. Continuing to observe the drainage without taking action could result in a serious complication.

A client who returned to the nursing unit 8 hours ago after hypophysectomy has clear drainage saturating the nasal dressing. The nurse should take which action first? 1. Continue to observe for further drainage. 2. Test the drainage for glucose.* 3. Put the head of the bed flat. 4. Test the drainage for occult blood.

*2. Test the drainage for glucose.* *rationale* Following hypophysectomy the client should be monitored for rhinorrhea (clear nasal drainage), which could indicate a cerebrospinal fluid (CSF) leak. If this occurs, the drainage should be collected and tested for the presence of CSF by testing it for glucose. CSF tests positive for glucose, whereas true nasal secretions would not. It is not necessary to test drainage that is clear for occult blood. The head of the bed should not be lowered, to prevent a rise in intracranial pressure. Continuing to observe the drainage without taking action could put the client at risk for developing a serious complication.

A nurse has reinforced instructions to the client with hyperparathyroidism regarding home care measures related to exercise. Which statement by the client indicates a need for further instruction? *Select all that apply.* 1. "I enjoy exercising but I need to be careful." 2. "I need to pace my activities throughout the day." 3. "I need to limit playing football to only the weekends." 4. "I should gauge my activity level by my energy level." 5. "I should exercise in the evening to encourage a good sleep pattern."

*3. "I need to limit playing football to only the weekends."* *5. "I should exercise in the evening to encourage a good sleep pattern."* *rationale* The client should be instructed to avoid high-impact activity or contact sports such as football. Exercising late in the evening may interfere with restful sleep. The client with hyperparathyroidism should pace activities throughout the day and plan for periods of uninterrupted rest. The client should plan for at least 30 minutes of walking each day to support calcium movement into the bones. The client should be instructed to use energy level as a guide to activity.

A client with Cushing's syndrome verbalizes concern to the nurse regarding the appearance of the buffalo hump that has developed. Which response by the nurse is appropriate? 1. "Don't be concerned, this problem can be covered with clothing." 2. "This is permanent, but looks are deceiving and not that important." 3. "Usually, these physical changes slowly improve following treatment." 4. "Try not to worry about it. There are other things to be concerned about."

*3. "Usually, these physical changes slowly improve following treatment."* *rationale* The client with Cushing's syndrome should be reassured that most physical changes resolve with treatment. Options 1, 2, and 4 are not therapeutic responses.

Following hypophysectomy, a client complains of being very thirsty and having to urinate frequently. The initial nursing action is to: 1. Document the complaints. 2. Increase fluid intake. 3. Check the urine specific gravity. 4. Check for urinary glucose.

*3. Check the urine specific gravity.* *rationale* Following hypophysectomy, diabetes insipidus can occur temporarily because of antidiuretic hormone deficiency. This deficiency is related to surgical manipulation. The nurse should check the urine for specific gravity and report the results if they are less than 1.005. Urinary glucose and diabetes mellitus is not a concern here. In this situation, increasing fluid intake would require a health care provider's prescription. The client's complaint would be documented but not as an initial action.

A nurse is caring for a client with pheochromocytoma. Which data would indicate a potential complication associated with this disorder? 1. A urinary output of 50 mL/hr 2. A coagulation time of 5 minutes 3. Congestion heard on auscultation of the lungs 4. A blood urea nitrogen (BUN) level of 20 mg/dL

*3. Congestion heard on auscultation of the lungs* *rationale* The complications associated with pheochromocytoma include hypertensive retinopathy and nephropathy, myocarditis, congestive heart failure (CHF), increased platelet aggregation, and stroke. Death can occur from shock, stroke, renal failure, dysrhythmias, or dissecting aortic aneurysm. Congestion heard on auscultation of the lungs is indicative of CHF. A urinary output of 50 mL/hr is an appropriate output; the nurse would become concerned if the output were less than 30 mL/hr. A coagulation time of 5 minutes is normal. A BUN level of 20 mg/dL is a normal finding.

A nurse is preparing to provide instructions to a client with Addison's disease regarding diet therapy. The nurse understands that which of the following diets would likely be prescribed for this client? 1. Low-protein diet 2. Low-sodium diet 3. High-sodium diet 4. Low-carbohydrate diet

*3. High-sodium diet* *rationale* A high-sodium, high-complex carbohydrate, and high-protein diet will be prescribed for the client with Addison's disease. To prevent excess fluid and sodium loss, the client is instructed to maintain an adequate salt intake of up to 8 g of sodium daily and to increase salt intake during hot weather, before strenuous exercise, and in response to fever, vomiting, or diarrhea.

A client is diagnosed with hyperparathyroidism. The nurse teaching the client about dietary alterations to manage the disorder tells the client to limit which of the following foods in the diet? 1. Bananas 2. Oatmeal 3. Ice cream 4. Chicken breast

*3. Ice cream* *rationale* The client with hyperparathyroidism is likely to have elevated calcium levels. This client should reduce intake of dairy products such as milk, cheese, ice cream, or yogurt. Apples, bananas, chicken, oatmeal, and pasta are low-calcium foods.

A nurse would expect to note which interventions in the plan of care for a client with hypothyroidism? *Select all that apply.* 1. Provide a cool environment for the client. 2. Instruct the client to consume a high-fat diet. 3. Instruct the client about thyroid replacement therapy. 4. Encourage the client to consume fluids and high-fiber foods in the diet. 5. Instruct the client to contact the health care provider if episodes of chest pain occur. 6. Inform the client that iodine preparations will be prescribed to treat the disorder.

*3. Instruct the client about thyroid replacement therapy.* *4. Encourage the client to consume fluids and high-fiber foods in the diet.* *5. Instruct the client to contact the health care provider if episodes of chest pain occur.* *rationale* The clinical manifestations of hypothyroidism are the result of decreased metabolism from low levels of thyroid hormone. Interventions are aimed at replacement of the hormones and providing measures to support the signs and symptoms related to a decreased metabolism. The nurse encourages the client to consume a well-balanced diet that is low in fat for weight reduction and high in fluids and high-fiber foods to prevent constipation. The client often has cold intolerance and requires a warm environment. The client would notify the health care provider if chest pain occurs since it could be an indication of overreplacement of thyroid hormone. Iodine preparations are used to treat hyperthyroidism. These medications decrease blood flow through the thyroid gland and reduce the production and release of thyroid hormone.

A client with newly diagnosed Cushing's syndrome expresses concern about personal appearance, specifically about the "buffalo hump" that has developed at the base of the neck. When counseling the client about this manifestation, the nurse should incorporate the knowledge that: 1. This is a permanent feature. 2. It can be minimized by wearing tight clothing. 3. It may slowly improve with treatment of the disorder. 4. It will quickly disappear once medication therapy is started.

*3. It may slowly improve with treatment of the disorder.* *rationale* The client with Cushing's syndrome should be reassured that most physical changes resolve over time with treatment. The other options are incorrect.

A nurse is collecting data regarding a client after a thyroidectomy and notes that the client has developed hoarseness and a week voice. What nursing action is appropriate

reassure the client that is usually a temporary condition

A nurse reviews a plan of care for a postoperative client following a thyroidectomy and notes that the client is at risk for breathing difficulty. Which of the following nursing interventions will the nurse suggest to include in the plan of care? 1. Maintain a supine position. 2. Encourage coughing and deep breathing exercises. 3. Monitor neck circumference frequently. 4. Maintain a pressure dressing on the operative site.

*3. Monitor neck circumference frequently.* *rationale* Following a thyroidectomy, the client should be placed in an upright position to facilitate air exchange. The nurse should assist the client with deep breathing exercises, but coughing is minimized to prevent tissue damage and stress to the incision. A pressure dressing is not placed on the operative site because it could affect breathing. The nurse should monitor the dressing closely and should loosen the dressing if necessary. Neck circumference is monitored at least every 4 hours to assess for postoperative edema.

Which nursing action would be appropriate to implement when a client has a diagnosis of pheochromocytoma? 1. Weigh the client. 2. Test the client's urine for glucose. 3. Monitor the client's blood pressure. 4. Palpate the client's skin to determine warmth.

*3. Monitor the client's blood pressure.* *rationale* Hypertension is the major symptom that is associated with pheochromocytoma. The blood pressure status is monitored by taking the client's blood pressure. Glycosuria, weight loss, and diaphoresis are also clinical manifestations of pheochromocytoma, but hypertension is the major symptom.

A nurse assists in developing a plan of care for a client with hyperparathyroidism receiving calcitonin-human (Cibacalcin). Which outcome has the highest priority regarding this medication? 1. Relief of pain 2. Absence of side effects 3. Reaching normal serum calcium levels 4. Verbalization of appropriate medication knowledge

*3. Reaching normal serum calcium levels* *rationale* Hypercalcemia can occur in clients with hyperparathyroidism, and calcitonin is used to lower plasma calcium level. The highest priority outcome in this client situation would be a reduction in serum calcium level. Option 1 is unrelated to this medication. Although options 2 and 4 are expected outcomes, they are not the highest priority for administering this medication.

A nurse caring for a client scheduled for a transsphenoidal hypophysectomy to remove a tumor in the pituitary gland assists to develop a plan of care for the client. The nurse suggests including which specific information in the preoperative teaching plan? 1. Hair will need to be shaved. 2. Deep breathing and coughing will be needed after surgery. 3. Toothbrushing will not be permitted for at least 2 weeks following surgery. 4. Spinal anesthesia is used.

*3. Toothbrushing will not be permitted for at least 2 weeks following surgery.* *rationale* Based on the location of the surgical procedure, spinal anesthesia would not be used. In addition, the hair would not be shaved. Although coughing and deep breathing are important, specific to this procedure is avoiding toothbrushing to prevent disruption of the surgical site. Also, coughing may disrupt the surgical site.

A preoperative client is scheduled for adrenalectomy to remove a pheochromocytoma. The nurse would most closely monitor which of the following items in the preoperative period? 1. Intake and output 2. Blood urea nitrogen (BUN) 3. Vital signs 4. Urine glucose and ketones

*3. Vital signs* *rationale* Hypertension is the hallmark of pheochromocytoma. Severe hypertension can precipitate a stroke or sudden blindness. Although all the items are appropriate nursing assessments for the client with pheochromocytoma, the priority is to monitor the vital signs, especially the blood pressure.

A health care provider prescribes a 24-hour urine collection for vanillylmandelic acid (VMA). The nurse instructs the client in the procedure for the collection of the urine. Which statement by the client would indicate a need for further instruction? 1. "I will start the collection in 2 days. I cannot eat or drink any tea, chocolate, vanilla, or fruit until the test is completed." 2. "When I start the collection, I will urinate and discard that specimen." 3. "I will pour the urine into the collection bottle each time I urinate and refrigerate the urine." 4. "I can take any medications if I need to before the collection."

*4. "I can take any medications if I need to before the collection."* *rationale* Because a 24-hour urine collection is a timed quantitative determination, it is essential that the client start the test with an empty bladder. Therefore the client is instructed to void and discard the first urine and note the time and start the test. The 24-hour urine specimen collection bottle must be kept on ice or refrigerated. In a VMA collection, the client is instructed to avoid tea, chocolate, vanilla, and all fruits for 2 days before urine collection begins. Also clients are reminded not to take certain medications for 2 to 3 days before the test.

A client with myxedema has changes in intellectual function such as impaired memory, decreased attention span, and lethargy. The client's husband is upset and shares his concerns with the nurse. Which statement by the nurse is helpful to the client's husband? 1. "Would you like me to ask the health care provider for a prescription for a stimulant?" 2. "Give it time. I've seen dozens of clients with this problem that fully recover." 3. "I don't blame you for being frustrated, because the symptoms will only get worse." 4. "It's obvious that you are concerned about your wife's condition, but the symptoms may improve with continued therapy."

*4. "It's obvious that you are concerned about your wife's condition, but the symptoms may improve with continued therapy."* *rationale* Using therapeutic communication techniques, the nurse acknowledges the husband's concerns and conveys that the client's symptoms are common with myxedema. With thyroid hormone therapy, these symptoms should decrease, and cognitive function often returns to normal. Option 1 is not helpful, and it blocks further communication. Option 3 is pessimistic and untrue. Option 2 is not appropriate and offers false reassurance.

A client with pheochromocytoma is scheduled for surgery and says to the nurse, "I'm not sure that surgery is the best thing to do." What response by the nurse is appropriate? 1. "I think you are making the right decision to have the surgery." 2. "You are very ill. Your health care provider has made the correct decision." 3. "There is no reason to worry. Your health care provider is a wonderful surgeon." 4. "You have concerns about the surgical treatment for your condition."

*4. "You have concerns about the surgical treatment for your condition."* *rationale* Paraphrasing is restating the client's message in the nurse's own words. Option 4 addresses the therapeutic communication technique of paraphrasing. The client is reaching out for understanding. In option 3, the nurse is offering a false reassurance, and this type of response will block communication. Option 2 also represents a communication block because it reflects a lack of the client's right to an opinion. In option 1, the nurse is expressing approval, which can be harmful to a nurse-client relationship.

The nurse caring for a client who has had a subtotal thyroidectomy reviews the plan of care and determines which problem is the priority for this client in the immediate postoperative period? 1. Dehydration 2. Infection 3. Urinary retention 4. Bleeding

*4. Bleeding* *rationale* Hemorrhage is one of the most severe complications that can occur following thyroidectomy. The nurse must frequently check the neck dressing for bleeding and monitor vital signs to detect early signs of hemorrhage, which could lead to shock. T3 and T4 do not regulate fluid volumes in the body. Infection is a concern for any postoperative client but is not the priority in the immediate postoperative period. Urinary retention can occur in postoperative clients as a result of medication and anesthesia but is not the priority from the options provided.

A nurse working on an endocrine nursing unit understands that which correct concept is used in planning care? 1. Clients with Cushing's syndrome are likely to experience episodic hypotension. 2. Clients with hyperthyroidism must be monitored for weight gain. 3. Clients who have diabetes insipidus should be assessed for fluid excess. 4. Clients who have hyperparathyroidism should be protected against falls.

*4. Clients who have hyperparathyroidism should be protected against falls.* *rationale* Hyperparathyroidism is a disease that involves excess secretion of parathyroid hormone (PTH). Elevation of PTH causes excess calcium to be removed from the bones. There is a decline in bone mass, which may cause a fracture if a fall occurs. Cushing's syndrome is likely to cause hypertension. Clients with hypothyroidism must be monitored for weight gain and clients with hyperthyroidism must be monitored for weight loss. Clients who have diabetes insipidus should be assessed for fluid deficit.

A nursing student notes in the medical record that a client with Cushing's syndrome is experiencing body image disturbances. The need for additional education regarding this problem is identified when the nursing student suggests which nursing intervention? 1. Encouraging the client's expression of feelings 2. Evaluating the client's understanding of the disease process 3. Encouraging family members to share their feelings about the disease process 4. Evaluating the client's understanding that the body changes need to be dealt with

*4. Evaluating the client's understanding that the body changes need to be dealt with* *rationale* Evaluating the client's understanding that the body changes that occur in this disorder need to be dealt with is an inappropriate nursing intervention. This option does not address the client's feelings. Options 1, 2, and 3 are appropriate because they address the client and family feelings regarding the disorder.

Which clinical manifestation should the nurse expect to note when assessing a client with Addison's disease? 1. Edema 2. Obesity 3. Hirsutism 4. Hypotension

*4. Hypotension* *rationale* Common manifestations of Addison's disease include postural hypotension from fluid loss, syncope, muscle weakness, anorexia, nausea, vomiting, abdominal cramps, weight loss, depression, and irritability. The manifestations in options 1, 2, and 3 are not associated with Addison's disease.

A client is brought to the emergency department in an unresponsive state, and a diagnosis of hyperglycemic hyperosmolar nonketotic syndrome (HHNS) is made. The nurse who is assisting to care for the client obtains which of the following immediately in preparation for the treatment of this syndrome? 1. NPH insulin 2. A nasal cannula 3. Intravenous (IV) infusion of sodium bicarbonate 4. IV infusion of normal saline

*4. IV infusion of normal saline* *rationale* The primary goal of treatment is to rehydrate the client to restore fluid volume and to correct electrolyte deficiency. IV fluid replacement is similar to that administered in diabetic ketoacidosis (DKA) and begins with IV infusion of normal saline. Regular insulin, not NPH insulin would be administered. The use of sodium bicarbonate to correct acidosis is avoided because it can precipitate a further drop in serum potassium levels. A nasal cannula for oxygen administration is not necessarily required to treat HHNS.

A client has an endocrine system dysfunction of the pancreas. The nurse anticipates that the client will exhibit impaired secretion of which of the following substances? 1. Amylase 2. Lipase 3. Trypsin 4. Insulin

*4. Insulin* *rationale* The pancreas produces both endocrine and exocrine secretions as part of its normal function. The organ secretes insulin as a key endocrine hormone to regulate the blood glucose level. Other pancreatic endocrine hormones are glucagon and somatostatin. The exocrine pancreas produces digestive enzymes such as amylase, lipase, and trypsin.

A nurse is caring for a client following a thyroidectomy. The client tells the nurse that she is concerned because of voice hoarseness. The client asks the nurse whether the hoarseness will subside. The nurse appropriately tells the client that the hoarseness: 1. Indicates nerve damage 2. Is harmless but permanent 3. Will worsen before it subsides 4. Is normal and will gradually subside

*4. Is normal and will gradually subside* *rationale* Hoarseness that develops in the postoperative period is usually the result of laryngeal pressure or edema and will resolve within a few days. The client should be reassured that the effects are transitory. Options 1, 2, and 3 are incorrect.

Which nursing measure would be effective in preventing complications in a client with Addison's disease? 1. Restricting fluid intake 2. Offering foods high in potassium 3. Checking family support systems 4. Monitoring the blood glucose

*4. Monitoring the blood glucose* *rationale* The decrease in cortisol secretion that characterizes Addison's disease can result in hypoglycemia. Therefore monitoring the blood glucose would detect the presence of hypoglycemia so that it can be treated early to prevent complications. Fluid intake should be encouraged to compensate for dehydration. Potassium intake should be restricted because of hyperkalemia. Option 3 is not a priority for this client.

A client with Graves' disease has exophthalmos and is experiencing photophobia. Which intervention would best assist the client with this problem? 1. Administering methimazole (Tapazole) every 8 hours 2. Lubricating the eyes with tap water every 2 to 4 hours 3. Instructing the client to avoid straining or heavy lifting 4. Obtaining dark glasses for the client

*4. Obtaining dark glasses for the client* *rationale* Because photophobia (light intolerance) accompanies this disorder, dark glasses are helpful in alleviating the symptom. Medical therapy for Graves' disease does not help alleviate the clinical manifestation of exophthalmos. Other interventions may be used to relieve the drying that occurs from not being able to completely close the eyes; however, the question is asking what the nurse can do for photophobia. Tap water, which is hypotonic, could actually cause more swelling to the eye because it could pull fluid into the interstitial space. In addition, the client is at risk for developing an eye infection because the solution is not sterile. There is no need to prevent straining with exophthalmos.

A nurse is caring for a client experiencing thyroid storm. Which of the following would be a priority concern for this client? 1. Inability to cope with the treatment plan 2. Lack of sexual drive 3. Self-consciousness about body appearance 4. Potential for cardiac disturbances

*4. Potential for cardiac disturbances* *rationale* Clients in thyroid storm are experiencing a life-threatening event, which is associated with uncontrolled hyperthyroidism. It is characterized by high fever, severe tachycardia, delirium, dehydration, and extreme irritability. The signs and symptoms of the disorder develop quickly, and therefore emergency measures must be taken to prevent death. These measures include maintaining hemodynamic status and patency of airway as well as providing adequate ventilation. Options 1, 2, and 3 are not a priority in the care of the client in thyroid storm.

A nurse is collecting data regarding a client after a thyroidectomy and notes that the client has developed hoarseness and a weak voice. Which nursing action is appropriate? 1. Check for signs of bleeding. 2. Administer calcium gluconate. 3. Notify the registered nurse immediately. 4. Reassure the client that this is usually a temporary condition.

*4. Reassure the client that this is usually a temporary condition.* *rationale* Weakness and hoarseness of the voice can occur as a result of trauma of the laryngeal nerve. If this develops, the client should be reassured that the problem will subside in a few days. Unnecessary talking should be discouraged. It is not necessary to notify the registered nurse immediately. These signs do not indicate bleeding or the need to administer calcium gluconate.

A nurse is caring for a postoperative adrenalectomy client. Which of the following does the nurse specifically monitor for in this client? 1. Peripheral edema 2. Bilateral exophthalmos 3. Signs and symptoms of hypocalcemia 4. Signs and symptoms of hypovolemia

*4. Signs and symptoms of hypovolemia* *rationale* Following adrenalectomy, the client is at risk for hypovolemia. Aldosterone, secreted by the adrenal cortex, plays a major role in fluid volume balance by retaining sodium and water. A deficiency of adrenocortical hormones does not cause the clinical manifestations noted in options 1, 2, and 3.

An indication of Chvostek' sign is: Answers: A. Twitching of the lips after tapping the face B. Elevated blood sugar after glucose infusion C. Inability to hold one's arms straight D. Spasms of the hand after blood circulation is cut off

. A Twitching of the lips after tapping the face in the right place is an indication of Chvostek's sign and a sign of hypocalcaemia. Spasms of the hand are associated with Trousseau's sign.

The nurse manager of a medical-surgical unit is asked to determine if the unit should adopt a new care delivery system. Which behavior is an example of an autocratic style of leadership? 1. Call a meeting and educate the staff on the new delivery system being used.

. An autocratic style is one in which the person in charge makes the decision without consulting anyone else.

What is a hormone secreted from the posterior lobe of the pituitary gland? Answers: A. LH B. MSH C. ADH D. GnRH

. C ADH is secreted from the posterior pituitary. LH comes from the anterior pituitary, MSH from the intermediate. GnRH is released from the hypothalamus.

The nurse is performing discharge teaching for a client diagnosed with Cushing's disease. Which statement by the client demonstrates an understanding of the instructions? 1. "I will be sure to notify my health-care provider if I start to run a fever."

. Cushing's syndrome/disease predis- poses the client to develop infections as a result of the immunosuppressive nature of the disease.

A client with Graves' disease experiences a thyroid storm and has tachycardia and hypertension. What medication is most likely to be used? Answers: A. Levofloxcin B. Chlorothiazide C. Percocet D. Propylthiouracil

. D Propylthiouracil is a commonly used medication for treating hyperthyroidism. Levofloxacin is an antibiotic, chlorothiazide is a diuretic, and Percocet a painkiller.

The client is diagnosed with hypothyroidism. Which signs/symptoms should the nurse expect the client to exhibit? 1. Complaints of extreme fatigue and hair loss.

1 . A decrease in thyroid hormone causes decreased metabolism, which leads to fatigue and hair loss.

Which laboratory data make the nurse suspect the client with primaryhyperparathyroidism is experiencing a complication? 1. A serum creatinine level of 2.8 mg/dL.

1 . A serum creatinine level of 2.8 mg/dL indicates the client is in renal failure, which is a complication of hyperparathy- roidism.

The nurse is planning the care of a client diagnosed with Addison's disease. Which intervention should be included? 1. Administer steroid medications.

1 . Clients diagnosed with Addison's disease have adrenal gland hypofunction.

The nurse identifies the client problem "risk for imbalanced body temperature" for the client diagnosed with hypothyroidism. Which intervention should be included in the plan of care? 1. Discourage the use of an electric blanket.

1 . External heat sources (heating pads,electric or warming blankets) should be discouraged because they increase the risk of peripheral vasodilation and vascular collapse.

The client diagnosed with Addison's disease is admitted to the emergency department after a day at the lake. The client is lethargic, forgetful, and weak. Which intervention should the nurse implement? 1. Start an IV with an 18-gauge needle and infuse NS rapidly.

1 . The client was exposed to wind and sun at the lake during the hours prior to being admitted to the emergency department.

. The nurse assessing a female client with Cushing's syndrome would expect to note which of the following? a) hirsutism b) hypotension c) hypoglycemia d) pallor

1) A - An increased production of androgens that accompanies a rise in cortisol levels with Cushing's syndrome produces hirsutism and acne in women. Other clinical findings of Cushing's syndrome include hypertension caused by sodium retention, impaired glucose tolerance or diabetes mellitus caused by cortisol's anti-insulin effect and ability to enhance gluconeogenesis, and skin changes including bruising and purplish red striae caused by protein catabolism.

Acromegaly is most frequently diagnosed in: a. Middle-aged adults b. Newborns c. Children ages 2 to 5 d. Adults age 65 and older

1. A: Acromegaly results from benign tumors on the pituitary gland that produce excessive amounts of growth hormone. Although symptoms may present at any age, the diagnosis generally occurs in middle-aged persons. Untreated, the consequences of acromegaly include type 2 diabetes, hypertension and increased risk of cardiovascular disease, arthritis and colon polyps.

The nurse is teaching the client diagnosed with hyperthyroidism. Which information should be taught to the client? Select all that apply. 1. Notify the HCP if a three (3)-pound weight loss occurs in two (2) days. 2. Discuss ways to cope with the emotional lability. 3. Notify the HCP if taking over-the-counter medication. 4. Carry a medical identification card or bracelet.

1. Notify the HCP if a three (3)-pound weight loss occurs in two (2) days. 2. Discuss ways to cope with the emotional lability. 3. Notify the HCP if taking over-the-counter medication. 4. Carry a medical identification card or bracelet.

A nurse provides dietary instructions to a client with a diagnosis of hyperparathyroidism. Which statement by the client indicates the need for further instructions? a) I need to drink 3000 ml of fluid per day b) I should drink cranberry juice daily c) I should eat foods high in calcium d) I should eat foods high in fiber

10) C - The client with hyperparathyroidism should consume at least 3000 mL of fluid per day. Measures to prevent dehydration are necessary because dehydration increases serum calcium levels and promotes the formation of renal stones. Cranberry juice and prune juice help make the urine more acidic. A high urinary acidity helps prevent renal stone formation because calcium is more soluble in acidic urine than in alkaline urine. Clients should be on a low-calcium, low-vitamin D diet. High-fiber foods are important to prevent constipation and fecal impaction resulting from the hypercalcemia that occurs with this disorder.

All of the following statements about Hashimoto's disease are true except: a. Many patients are entirely asymptomatic b. Not all patients become hypothyroid c. Most cases of obesity are attributable to Hashimoto's disease d. Hypothyroidism may be subclinical

10. C: Although weight gain may be a symptom of Hashimoto's disease, the majority of obese people have normal thyroid function; rarely is thyroid disorder the sole cause of obesity. Other symptoms of Hashimoto's disease include fatigue, cold intolerance, joint pain, myalgias, constipation, dry hair, skin and nails, impaired fertility, slow heart rate, and depression.

In a patient with central diabetes insipidus, what will the administration of ADH during a water deprivation test result in? a. Decrease in body weight b. Increase in urinary output c. Decrease in blood pressure d. Increase in urine osmolality

10. d. A patient with central diabetes insipidus has a deficiency of ADH with excessive loss of water from the kidney, hypovolemia, hypernatremia, and dilute urine with a low specific gravity. When vasopressin is administered, the symptoms are reversed, with water retention, decreased urinary output that increases urine osmolality, and an increase in BP.

A nurse is caring for a client with hyperthyroidism and is instructing the client about dietary measures. The nurse tells the client that it is important to eat foods that are: a) high in bulk and fiber b) low in calories c) low in carbohydrates and fats d) high in calories

16) D - The client with hyperthyroidism is usually extremely hungry because of increased metabolism. The client should be instructed to consume a high-calorie diet with six full meals a day. The client should be instructed to eat foods that are nutritious and contain ample amounts of protein, carbohydrates, fats, and minerals. Clients should be discouraged from eating foods that increase peristalsis and thus result in diarrhea, such as highly seasoned, bulky, and fibrous foods.

Untreated hyperthyroidism during pregnancy may result in all of the following except: a. Premature birth and miscarriage b. Low birthweight c. Autism d. Preeclampsia

17. C: In addition to the above-mentioned complications of uncontrolled hyperthyroidism in pregnancy, expectant mothers may suffer congestive heart failure and thyroid storm, which is life-threatening thyrotoxicosis with symptoms that include agitation, confusion, tachycardia, shaking, sweating, diarrhea, fever, and restlessness.

Endocrine disorders may be triggered by all of the following except: a. Stress b. Infection c. Chemicals in the food chain and environment d. Cell phone use

19. D: Endocrine function may be influenced by myriad factors. In addition to the above-mentioned, there is evidence that exposure to naturally occurring and man-made endocrine disruptors such as tributyltin, certain bioaccumulating chlorinated compounds, and phytoestrogens is widespread and in susceptible individuals, may trigger endocrine disorders.

A nurse is admitting a client with a diagnosis of Addison's disease to the hospital. On assessment, the nurse would expect to note which finding that is a manifestation of this disorder? a) peripheral edema b) excessive facial hair c) lower than normal blood glucose level d) high blood pressure

2) C - Blood glucose levels are low in Addison's disease as a result of decreased secretion of glucocorticoids (cortisol). Edema is absent, and aldosterone secretion is decreased so the client develops a deficient fluid volume. Facial hair increases with adrenocortical hyperfunction. Clients with Addison's disease develop hypotension as a result of deficient fluid volume. Options A, B and D are unrelated to Addison's disease.

The client diagnosed with hypothyroidism is prescribed the thyroid hormone levothyroxine (Synthroid). Which assessment data indicate the medication has been effective? 3. The client's temperature is WNL.

3. The client with hypothyroidism frequently has a subnormal temperature, so a temperature WNL indicates the medication is effective.

The nurse is assessing a client in an outpatient clinic. Which assessment data are a risk factor for developing pheochromytoma? 3. A family history of adrenal tumors.

3. There is a high incidence of pheochromocytomas in family members with adrenal tumors,

The client diagnosed with a pituitary tumor developed syndrome of inappropriate antidiuretic hormone (SIADH). Which interventions should the nurse implement? 2. Assess for nausea and vomiting and weigh daily.

2. Early signs and symptoms are nausea and vomiting.

Which signs/symptoms should make the nurse suspect the client is experiencing a thyroid storm? 2. Hyperpyrexia and extreme tachycardia.

2. Hyperpyrexia (high fever) and heart rate above 130 beats per minute are signs of thyroid storm,

The client is admitted to rule out Cushing's syndrome. Which laboratory tests should the nurse anticipate being ordered? 2. Plasma levels of ACTH and cortisol.

2. The adrenal gland secretes cortisol and the pituitary gland secretes adrenocorticotropic hormone (ACTH), a hormone used by the body to stimulate the production of cortisol.

The nurse is admitting a client to the neurological intensive care unit who is postoperative transsphenoidal hypophysectomy. Which data warrant immediate intervention? 2. The client has an output of 2,500 mL since surgery and an intake of 1,000 mL.

2. The output is more than double the intake in a short time.

Which question should the nurse ask when assessing the client for an endocrine dysfunction? 2. "Have you had any unexplained weight loss?"

2. Weight loss with normal appetite may indicate hyperthyroidism.

Which sign/symptom indicates to the nurse the client is experiencing hyperparathyroidism? 2. A positive Chvostek's sign.

2. When a sharp tapping over the facial nerve elicits a spasm or twitching of the mouth, nose, or eyes, the client is hypocalcemic

A husband of a client with graves' disease expresses concern regarding his wife's health because during the past 3 months she has been experiencing nervousness, inability to concentrate even on trivial tasks, and outbursts of temper. On the basis of this information, which nursing diagnosis would the nurse identify as appropriate for the client? a) ineffective coping b) disturbed sensory perception c) social isolation d) grieving

20) A - Frequently, family and friends may report that the client with Graves' disease has become more irritable or depressed. The signs and symptoms in the question are supporting data for the nursing diagnosis of Ineffective coping and are not related to options B, C, and D. The question does not provide data to support options B, C, and D.

A nurse is caring for a client with hypoparathyroidism. In planning for discharge from the hospital, the nurse identifies which of the following as a potential psychosocial nursing diagnosis? a) impaired comfort related to cold intolerance secondary to decreased metabolic rate b) constipation related to decreased peristaltic action secondary to decreased metabolic rate c) high risk for impaired skin integrity related to edema d) anxiety related to the need for lifelong dietary interventions to control the disease

21) D - Medical management of hypoparathyroidism is aimed at correcting the hypocalcemia. This is accomplished with prescribed medications as well as lifelong compliance to dietary guidelines, which include consumption of foods high in calcium but low in phosphorus. Knowing that the interventions are lifelong can create some anxiety for the client, and this problem needs to be addressed before hospital discharge. The other options are unrelated to this condition and to a psychosocial concern.

A nurse provides instructions to a client who is scheduled for a radioactive iodine uptake test. Which statement by the client indicates a need for further instructions? a) the test measures the rate of iodine uptake by my thyroid gland b) I will need to drink a small dose of radioactive iodine before the test c) a 24 hour urine specimen will need to be collected to measure iodine excretion d) I need to minimize close contact with others in my family for a period of 48 hours after the test because of the radioactivity in my system

27) D - The client undergoing a radioactive iodine uptake test needs to be reassured that the amount of radioactive iodine used is very small, that it is not harmful to the client, and that the client will not be radioactive. The other options are correct regarding this diagnostic test.

A nurse is caring for a client with Cushing's syndrome who demonstrates withdrawn behavior. The nurse recognizes that this client's behavior is likely related to which nursing diagnosis? a) deficient diversional activity b) powerlessness c) hopelessness d) disturbed body image

29) D - Physical changes in the client's appearance can occur with Cushing's syndrome. Such changes include hirsutism, moon face, buffalo hump, acne, and striae. These changes cause a body image disturbance. Options A, B, and C are not commonly associated with Cushing's syndrome.

A nurse is preparing to perform an assessment on a client being admitted to the hospital with a diagnosis of Cushing's syndrome. When performing the assessment, the nurse checks for which significant manifestation of the disorder? a) fluid retention b) stretch marks c) goiter d) melanosis

3) A - Excessive secretion of adrenocortical hormones results in water and sodium reabsorption, causing fluid retention. Stretch marks (striae) are a common feature and can result in a disturbed body image, but are not significant and do not represent a life-threatening situation. Goiter is not a manifestation of Cushing's syndrome. Melanosis is a common manifestation associated with Addison's disease.

The nurse writes a problem of "altered body image" for a 34-year-old client diagnosed with Cushing's disease. Which intervention should be implemented? 3. Use therapeutic communication to allow the client to discuss feelings.

3. Allowing the client to ventilate feelings about the altered body image is the most appropriate intervention.

The nurse is admitting a client diagnosed with primary adrenal cortex insufficiency (Addison's disease). Which clinical manifestations should the nurse expect to assess? 3. Bronze pigmentation, hypotension, and anorexia.

3. Bronze pigmentation of the skin, particularly of the knuckles and other areas of skin creases, occurs in Addison's disease.

The client diagnosed with Cushing's disease has undergone a unilateral adrenalectomy. Which discharge instructions should the nurse discuss with the client? 3. Explain the signs and symptoms of infection and when to call the health-care provider.

3. Notifying the HCP if signs/symptoms of infection develop is an instruction given to all surgical clients on discharge.

The nurse is discussing the endocrine system with the client. Which endocrine gland secretes epinephrine and norepinephrine? 3. The adrenal medulla.

3. The adrenal medulla secretes the catecholamines epinephrine and norepinephrine.

The charge nurse of an intensive care unit is making assignments for the night shift.Which client should be assigned to the most experienced intensive care nurse? 3. The client diagnosed with Addison's disease who is lethargic and has a BP of 80/45, P 124, and R 28.

3. This client has a low blood pressure and tachycardia. This client may be experiencing an addisonian crisis

The nurse is preparing to administer the following medications. Which medication should the nurse question administering? 3. The loop diuretic to the client with a potassium level of 3.3 mEq/L.

3. This potassium level is below normal, which is 3.5 to 5.5 mEq/L.

A client with Addison's disease makes all of the following statements. Which one does the nurse analyze as requiring further discussion? a) I wear a Medic-Alert bracelet at all times b) I need to weigh myself daily and record it c) It is important that I drink enough fluids and increase my salt intake d) my medication doses will not need to be adjusted for any reason

30) D - The client with Addison's disease is experiencing deficits of mineralocorticoids, glucocorticoids, and androgens. Aldosterone deficiency affects the ability of the nephrons to conserve sodium, so the client experiences sodium and fluid volume deficit. The client needs to manage this problem with daily hormone replacement and increased fluid and sodium intake. Clients are instructed to weigh themselves daily as a means of monitoring fluid volume balance. Glucocorticoids and mineralocorticoids are essential components of the stress response. Additional doses of hormone replacement therapy are needed with any type of physical or psychological stressor. This information needs to be conveyed to the client and requires that the client wear a Medic-Alert bracelet so that health care professionals are aware of this problem if the client were to experience a medical emergency.

A clinic nurse is performing an assessment on a client who has hypothyroidism. The nurse would expect to note which clinical manifestation? a) complaints of difficulty sleeping b) complaints of diarrhea c) significant weight loss since the last clinic visit d) complaints of intolerance to cold weather

4) D - An insufficient level of thyroid hormone causes a decrease in metabolic rate and heat production. Intolerance to cold would be noted. Options A, B and C are clinical manifestations of hyperthyroidism.

A client with Addison's disease will most likely exhibit which symptom? A. Hypertension B. Bronze pigmentation C. Hirsutism D. Purple striae

Answer B is correct. Answer B is correct because a bronze pigmentation is a sign of Addison's disease. Answers A, C, and D are symptoms of Cushing's syndrome, making them incorrect.

Which vitamin is directly involved in the metabolism of the hormones secreted by the parathyroid? A. Vitamin C B. Vitamin D C. Vitamin K D. Vitamin B9

Answer B is correct. Vitamin D is related to absorption of calcium and phosphorus. A, C, and D are incorrect because they are not related to the absorption of calcium and phosphorus.

The nurse is providing an in-service on thyroid disorders. One of the attendees asks the nurse, "Why don't the people in the United States get goiters as often?" Which statement by the nurse is the best response? 1. "It is because of the screening techniques used in the United States." 2. "It is a genetic predisposition rare in North Americans." 3. "The medications available in the United States decrease goiters." 4. "Iodized salt helps prevent the development of goiters in the United States."

4. Almost all of the iodine entering the body is retained in the thyroid gland. A deficiency in iodine will cause the thyroid gland to work hard and enlarge, which is called a goiter. Goiters are commonly seen in geographical regions having an iodine deficiency. Most table salt in the United States has iodine added.

The client is three (3) days postoperative unilateral adrenalectomy. Which discharge instructions should the nurse teach? 4. Tell the client to notify the HCP if the incision is inflamed.

4. Any inflammation of the incision indicates an infection

The nurse is admitting a client to rule out aldosteronism. Which assessment data support the client's diagnosis? 4. Blood pressure.

4. Blood pressure is affected by aldosteronism, with hypertension being the most prominent and universal sign of aldosteronism.

The nurse is developing a plan of care for the client diagnosed with acquired immunodeficiency syndrome (AIDS) who has developed an infection in the adrenal gland. Which client problem is highest priority? 4. Fluid volume deficit.

4. Fluid volume deficit (dehydration) can lead to circulatory impairment and hyperkalemia.

Which statement made by the client makes the nurse suspect the client is experiencing hyperthyroidism? 4. "I have noticed all my collars are getting tighter."

4. The thyroid gland (in the neck) en larges as a result of the increased need for thyroid hormone production; an enlarged gland is called a goiter.

A patient with SIADH is treated with water restriction. What does the patient experience when the nurse determines that treatment has been effective? a. Increased urine output, decreased serum sodium, and increased urine specific gravity b. Increased urine output, increased serum sodium, and decreased urine specific gravity c. Decreased urine output, increased serum sodium, and decreased urine specific gravity d. Decreased urine output, decreased serum sodium, and increased urine specific gravity

8. b. The patient with SIADH has water retention with hyponatremia, decreased urine output, and concentrated urine with high specific gravity. Improvement in the patient's condition is reflected by increased urine output, normalization of serum sodium, and more water in the urine, thus decreasing the specific gravity.

A nurse is monitoring a client for complications following thyroidectomy. The nurse notes that the client's voice is very hoarse, and the client is concerned about the hoarseness and asks the nurse about it. The nurse makes which response to alleviate the client's concern? a) hoarseness and weak voice indicate permanent damage to the nerves b) this complication is expected c) this problem is temporary and will probably subside in a few days d) it is best that you not talk at all until the problem is further evaluated

9) C Temporary hoarseness and a weak voice may occur if there has been unilateral injury to the laryngeal nerve during surgery. If hoarseness or a weak voice is present, the client is reassured that the problem will probably subside in a few days. Unnecessary talking is discouraged to minimize hoarseness. The statements in options A, B, and D will not alleviate the client's concern.

Persons at increased risk of developing Hashimoto's disease include all of the following except: a. Persons with vitiligo b. Asian-Americans c. Persons with rheumatoid arthritis d. Persons with Addison's disease

9. B: Along with the above-mentioned groups, persons with type 1 diabetes and persons suffering from pernicious anemia (insufficient vitamin b12) are at increased risk of developing Hashimoto's disease. Because it tends to run in families, there is likely a genetic susceptibility as well. Environmental factors such as excessive iodine consumption and selected drugs also have been implicated as potential risk factors.

A client has been diagnosed with goiter. The nurse looks for documentation of which of the following in the client's medical record? a) decreased wound healing b) chronic fatigue c) enlarged thyroid gland d) heart damage

6) C - An enlarged thyroid gland occurs in goiter. Decreased wound healing, chronic fatigue, and heart damage are not specifically associated with this condition.

During care of the patient with SIADH, what should the nurse do? a. Monitor neurologic status at least every 2 hours. b. Teach the patient receiving treatment with diuretics to restrict sodium intake. c. Keep the head of the bed elevated to prevent antidiuretic hormone (ADH) release. d. Notify the health care provider if the patient's blood pressure decreases more than 20 mm Hg from baseline.

7. a. The patient with syndrome of inappropriate antidiuretic hormone (SIADH) has marked dilutional hyponatremia and should be monitored for decreased neurologic function and seizures every 2 hours. Sodium intake is supplemented because of the hyponatremia and sodium loss caused by diuretics. ADH release is reduced by keeping the head of the bed flat to increase left atrial filling pressure. A reduction in blood pressure (BP) indicates a reduction in total fluid volume and is an expected outcome of treatment.

A nurse is caring for a client following thyroidectomy and is monitoring for complications. Which of the following if noted in the client, would indicate a need for physician notification? a) surgical pain in the neck area b) voice hoarseness c) numbness and tingling around the mouth d) weakness of the voice

8) C - Hypocalcemia can develop after thyroidectomy if the parathyroid glands are accidentally removed or traumatized during surgery. If the client develops numbness and tingling around the mouth or in the fingertips or toes, muscle spasms, or twitching, the physician should be called immediately. A hoarse or weak voice may occur temporarily if there has been unilateral injury to the laryngeal nerve during surgery. Pain is expected in the postoperative period. Calcium gluconate ampules should be available at the bedside, and the client should have a patent intravenous (IV) line in the event that hypocalcemic tetany occurs

Hashimoto's disease is: a. Chronic inflammation of the thyroid gland b. Diagnosed most frequently in Asian-Americans and Pacific Islanders c. A form of hyperthyroidism d. A rare form of hypothyroidism

8. A: Hashimoto's disease is the most common cause of hypothyroidism. It is an autoimmune disease that produces chronic inflammation of the thyroid gland. More women are affected than men and it is generally diagnosed in persons ages 40 to 60. When treatment is indicated, synthetic T4 is administered.

Which of the following would be an indication of Androgen Insensitivity Syndrome? Answers: A. A 33 year old woman with a karyotype of XY B. A 16 year old male with reduced kidney function C. Failure to respond to cortisol therapy D. Several pregnancies all of which ended in miscarriages

A Androgen Insensitivity Syndrome is when the body does not respond to androgens such as testosterone. This can result in genetic males being born with the appearance of women.

Symptoms of Grave's ophthalmopathy include all of the following except: a. Bulging eyeballs b. Dry, irritated eyes and puffy eyelids c. Cataracts d. Light sensitivity

A Grave's ophthalmopathy is an inflammation of tissue behind the eye causing the eyeballs to bulge. In addition to the above-mentioned symptoms, Grave's ophthalmopathy may cause pressure or pain in the eyes, double vision, and trouble moving the eyes. About one-quarter of persons with Grave's disease develop Grave's ophthalmopathy. The condition is frequently self-limiting, resolving without treatment over the course of a year or two.

In educating a client, the nurse is likely to explain the following is the cause of Hashimoto's disease: Answers: A. Antibodies attacking the thyroid gland B. Inflammation in the kidneys C. An adenocarcinoma in the brain D. Overactivation of the pituitary gland

A Hashimoto's disease is caused by autoimmunity to the thyroid gland, often involving antibodies.

Which client is at risk for developing thyrotoxicosis?

A client with Graves' disease who is having surgery

The nurse is caring for a client with Addison's disease. The nurse checks the client's vital signs and determines that the client has orthostatic hypotension. The nurse determines that this finding relates to which factor?

A decreased secretion of aldosterone

While collecting data on a client being prepared for an adrenalectomy, the nurse obtains a temperature reading of 100.8° F. The nurse analyzes this temperature reading as which?

A finding that needs to be reported immediately

The client is admitted to the intensive care department diagnosed with myxedema coma. Which assessment data warrant immediate intervention by the nurse? 2. Pulse oximeter reading of 90%..

A pulse oximeter reading of less than 93% is significant.

Which of the following symptoms are expected in the patient with pheochromocytoma? Select all that Apply (SATA): A) Hypertension B) HA C) Hypoglycemia D) Hypermetabolism E) Dry skin

A, B, and D. The five H's: HA, Hypertension, Hyperhydrosis (sweating), Hypermetabolism and Hyperglycemia.

Which of the following would the nurse monitor for as signs of thyroid storm? SATA: A) T 101.5 B) HR 59 C) Constipation D) Edema E) Chest pain

A, D, and E. Thyroid storm is a form of severe hyperthyroidism usually of abrupt onset. S/S are typically characterized by exaggeration of hyperthyroidism. An extreme fever over 101.3 degrees often occurs along with extreme tachycardia (>130) During thyroid storm increased cardiac compromise may occur as evidenced by edema (HF), chest pain, dyspnea, or palpitations.

A patient with acromegaly is treated with transsphenoidal hypophysectomy. Postoperatively the nurse a. ensures that any clear nasal drainage is tested for glucose b. maintains the patient flat in bed to prevent CSF leakage c. assists the patient with toothbrushing every 4 hours to keep the surgical area clean d. encourages deep breathing, coughing, and turning to prevent respiratory complications

A- A transsphenoidal hypophysectomy involves entry into the sella turcica through an incision in the upper lip and gingiva into the floor of the nose and the sphenoid sinuses. Postoperative clear nasal drainage with glucose content indicates CSF leakage from an open connection to the brain, putting the patient at risk for meningitis. After surgery, the patient is positioned with the head of the bed elevated to avoid pressure on the sella turcica. Coughing and straining are avoided to prevent increased ICP and CSF leakage, and although mouth care is required every 4 hours, toothbrushing should not be performed because injury to the suture line may occur.

Preoperative instructions for the patient scheduled for a subtotal thyroidectomy include teaching the patient a. how to support the head with the hands when moving b. that coughing should be avoided to prevent pressure on the incision c. that the head and neck will need to remain immobile until the incision heals d. that any tingling around the lips or in the fingers after surgery is expected and temporary

A- To prevent strain on the suture line postoperatively, the patient's head must be manually supported while turning and moving in bed, but ROM exercises for the head and neck are also taught preoperatively to be gradually implemented after surgery. There is no contraindication for coughing and deep breathing, and these should be carried out postoperatively. Tingling around the lips or fingers is a sign of hypocalcemia, which may occur if the parathyroid glands are inadvertently removed during surgery. This sign should be reported immediately.

During the care of the patient with SIADH, the nurse should a. monitor neurologic status at least every 2 hours b. keep the head of the bed elevated to prevent ADH release c. teach the patient receiving treatment with diuretics to restrict sodium intake d. notify the health care provider if the patient's BP decreases more than 20 mmHg from baseline

A- the patient with syndrome of inappropriate secretion of antidiuretic hormone (SIADH) has marked dilutional hyponatremia and should be monitored for decreased neurologic function and convulsions every 2 hours. ADH release is reduced by keeping the head of the bed flat to increase left atrial filling pressure, and sodium intake is supplemented because of the hyponatremia and sodium loss caused by diuretics. A reduction in BP indicates a reduction in total fluid volume and is an expected outcome of treatment.

Nurse Louie is developing a teaching plan for a male client diagnosed with diabetes insipidus. The nurse should include information about which hormone lacking in clients with diabetes insipidus? a. antidiuretic hormone (ADH). b. thyroid-stimulating hormone (TSH). c. follicle-stimulating hormone (FSH). d. luteinizing hormone (LH).

A. ADH is the hormone clients with diabetes insipidus lack. The client's TSH, FSH, and LH levels won't be affected.

Jemma, who weighs 210 lb (95 kg) and has been diagnosed with hyperglycemia tells the nurse that her husband sleeps in another room because her snoring keeps him awake. The nurse notices that she has large hands and a hoarse voice. Which of the following would the nurse suspect as a possible cause of the client's hyperglycemia? a. Acromegaly b. Type 1 diabetes mellitus c. Hypothyroidism d. Deficient growth hormone

A. Acromegaly, which is caused by a pituitary tumor that releases excessive growth hormone, is associated with hyperglycemia, hypertension, diaphoresis, peripheral neuropathy, and joint pain. Enlarged hands and feet are related to lateral bone growth, which is seen in adults with this disorder. The accompanying soft tissue swelling causes hoarseness and often sleep apnea. Type 1 diabetes is usually seen in children, and newly diagnosed persons are usually very ill and thin. Hypothyroidism isn't associated with hyperglycemia, nor is growth hormone deficiency.

When caring for a male client with diabetes insipidus, nurse Juliet expects to administer: a. vasopressin (Pitressin Synthetic). b. furosemide (Lasix). c. regular insulin. d. 10% dextrose.

A. Because diabetes insipidus results from decreased antidiuretic hormone (vasopressin) production, the nurse should expect to administer synthetic vasopressin for hormone replacement therapy. Furosemide, a diuretic, is contraindicated because a client with diabetes insipidus experiences polyuria. Insulin and dextrose are used to treat diabetes mellitus and its complications, not diabetes insipidus.

Which outcome indicates that treatment of a male client with diabetes insipidus has been effective? a. Fluid intake is less than 2,500 ml/day. b. Urine output measures more than 200 ml/hour. c. Blood pressure is 90/50 mm Hg. d. The heart rate is 126 beats/minute.

A. Diabetes insipidus is characterized by polyuria (up to 8 L/day), constant thirst, and an unusually high oral intake of fluids. Treatment with the appropriate drug should decrease both oral fluid intake and urine output. A urine output of 200 ml/hour indicates continuing polyuria. A blood pressure of 90/50 mm Hg and a heart rate of 126 beats/minute indicate compensation for the continued fluid deficit, suggesting that treatment hasn't been effective.

Following a unilateral adrenalectomy, nurse Betty would assess for hyperkalemia shown by which of the following? a. Muscle weakness b. Tremors c. Diaphoresis d. Constipation

A. Muscle weakness, bradycardia, nausea, diarrhea, and paresthesia of the hands, feet, tongue, and face are findings associated with hyperkalemia, which is transient and occurs from transient hypoaldosteronism when the adenoma is removed. Tremors, diaphoresis, and constipation aren't seen in hyperkalemia.

A female adult client with a history of chronic hyperparathyroidism admits to being noncompliant. Based on initial assessment findings, nurse Julia formulates the nursing diagnosis of Risk for injury. To complete the nursing diagnosis statement for this client, which "related-to" phrase should the nurse add? a. Related to bone demineralization resulting in pathologic fractures b. Related to exhaustion secondary to an accelerated metabolic rate c. Related to edema and dry skin secondary to fluid infiltration into the interstitial spaces d. Related to tetany secondary to a decreased serum calcium level

A. Poorly controlled hyperparathyroidism may cause an elevated serum calcium level. This, in turn, may diminish calcium stores in the bone, causing bone demineralization and setting the stage for pathologic fractures and a risk for injury. Hyperparathyroidism doesn't accelerate the metabolic rate. A decreased thyroid hormone level, not an increased parathyroid hormone level, may cause edema and dry skin secondary to fluid infiltration into the interstitial spaces. Hyperparathyroidism causes hypercalcemia, not hypocalcemia; therefore, it isn't associated with tetany.

A female client has a serum calcium level of 7.2 mg/dl. During the physical examination, nurse Noah expects to assess: a. Trousseau's sign. b. Homans' sign. c. Hegar's sign. d. Goodell's sign.

A. This client's serum calcium level indicates hypocalcemia, an electrolyte imbalance that causes Trousseau's sign (carpopedal spasm induced by inflating the blood pressure cuff above systolic pressure). Homans' sign (pain on dorsiflexion of the foot) indicates deep vein thrombosis. Hegar's sign (softening of the uterine isthmus) and Goodell's sign (cervical softening) are probable signs of pregnancy.

The parathyroid glands play a major role in regulating which substances? A. Calcium and Phosphorus B. Cholride and potassium C. Potassium and calcium D. Sodium and potassium

A. Calcium and Phosphorus

A patient is admitted to the hospital with a diagnosis of Cushing syndrome. On physical assessment of the patient, the nurse would expect to find a. hypertension, peripheral edema, and petechiae b. weight loss, buffalo hump, and moon face with acne c. abdominal and buttock striae, truncal obesity, and hypotension d. anorexia, signs of dehydration, and hyperpigmentation of the skin

A. The effects of glucocorticoid excess include weight gain from accumulation and redistribution of adipose tissue, sodium and water retention, glucose intolerance, protein wasting, loss of bone structure, loss of collagen, and capillary fragility. Clinical manifestations of corticosteroid deficiency include hypotension, dehydration, weight loss, and hyperpigmentation of the skin.

The nurse is providing preoperative teaching to the patient about to undergo a thyroidectomy. Which statement, if made by the patient, indicates the need for further teaching? A) I need to eat super healthy before the surgery and should limit my calories to a healthy amount B) Before the surgery I should practice raising my elbows and placing both hands behind my neck to provide support C) Well, I will have to quit drinking my coffee and alcohol for now D) I will be sure increase my intake of protein and carbohydrates

A. a high calorie diet should be encouraged preoperatively because of the increased metabolic needs of the body. Practicing supporting the neck before the surgery is a good way to remember what to do after the surgery. The patient should avoid consuming any stimulants including tea, coffee, and cola. The patient should increase intake of protein and carbohydrates preoperatively because of increased metabolic needs.

What does the pituitary gland secrete?

ADH, Thyroid stimulating hormone, sex hormones, growth hormone, and ACTH.

A patient is to receive methylprednisolone (Solu-Medrol) 100 mg. The label on the medication states: methylprednisolone 125 mg in 2 mL. How many milliliters will the nurse administer?

ANS: 1.6 A concentration of 125 mg in 2 mL will result in 100 mg in 1.6 mL.

Which prescribed medication should the nurse administer first to a 60-year-old patient admitted to the emergency department in thyroid storm? a. Propranolol (Inderal) b. Propylthiouracil (PTU) c. Methimazole (Tapazole) d. Iodine (Lugol's solution)

ANS: A Adrenergic blockers work rapidly to decrease the cardiovascular manifestations of thyroid storm. The other medications take days to weeks to have an impact on thyroid function.

Which finding for a patient who has hypothyroidism and hypertension indicates that the nurse should contact the health care provider before administering levothyroxine (Synthroid)? a. Increased thyroxine (T4) level b. Blood pressure 112/62 mm Hg c. Distant and difficult to hear heart sounds d. Elevated thyroid stimulating hormone level

ANS: A An increased thyroxine level indicates the levothyroxine dose needs to be decreased. The other data are consistent with hypothyroidism and the nurse should administer the levothyroxine.

Which finding by the nurse when assessing a patient with Hashimoto's thyroiditis and a goiter will require the most immediate action? a. New-onset changes in the patient's voice b. Apical pulse rate at rest 112 beats/minute c. Elevation in the patient's T3 and T4 levels d. Bruit audible bilaterally over the thyroid gland

ANS: A Changes in the patient's voice indicate that the goiter is compressing the laryngeal nerve and may lead to airway compression. The other findings will also be reported but are expected with Hashimoto's thyroiditis and do not require immediate action.

Which finding indicates to the nurse that the current therapies are effective for a patient with acute adrenal insufficiency? a. Increasing serum sodium levels b. Decreasing blood glucose levels c. Decreasing serum chloride levels d. Increasing serum potassium levels

ANS: A Clinical manifestations of Addison's disease include hyponatremia and an increase in sodium level indicates improvement. The other values indicate that treatment has not been effective.

A 56-year-old female patient has an adrenocortical adenoma, causing hyperaldosteronism. The nurse providing care should a. monitor the blood pressure every 4 hours. b. elevate the patient's legs to relieve edema. c. monitor blood glucose level every 4 hours. d. order the patient a potassium-restricted diet.

ANS: A Hypertension caused by sodium retention is a common complication of hyperaldosteronism. Hyperaldosteronism does not cause an elevation in blood glucose. The patient will be hypokalemic and require potassium supplementation before surgery. Edema does not usually occur with hyperaldosteronism.

The nurse determines that additional instruction is needed for a 60-year-old patient with chronic syndrome of inappropriate antidiuretic hormone (SIADH) when the patient says which of the following? a. "I need to shop for foods low in sodium and avoid adding salt to food." b. "I should weigh myself daily and report any sudden weight loss or gain." c. "I need to limit my fluid intake to no more than 1 quart of liquids a day." d. "I will eat foods high in potassium because diuretics cause potassium loss."

ANS: A Patients with SIADH are at risk for hyponatremia, and a sodium supplement may be prescribed. The other patient statements are correct and indicate successful teaching has occurred.

The nurse is caring for a patient admitted with diabetes insipidus (DI). Which information is most important to report to the health care provider? a. The patient is confused and lethargic. b. The patient reports a recent head injury. c. The patient has a urine output of 400 mL/hr. d. The patient's urine specific gravity is 1.003.

ANS: A The patient's confusion and lethargy may indicate hypernatremia and should be addressed quickly. In addition, patients with DI compensate for fluid losses by drinking copious amounts of fluids, but a patient who is lethargic will be unable to drink enough fluids and will become hypovolemic. A high urine output, low urine specific gravity, and history of a recent head injury are consistent with diabetes insipidus, but they do not require immediate nursing action to avoid life-threatening complications.

A patient who was admitted with myxedema coma and diagnosed with hypothyroidism is improving and expected to be discharged in 2 days. Which teaching strategy will be best for the nurse to use? a. Provide written reminders of self-care information. b. Offer multiple options for management of therapies. c. Ensure privacy for teaching by asking visitors to leave. d. Delay teaching until patient discharge date is confirmed.

ANS: A Written instructions will be helpful to the patient because initially the hypothyroid patient may be unable to remember to take medications and other aspects of self-care. Because the treatment regimen is somewhat complex, teaching should be initiated well before discharge. Family members or friends should be included in teaching because the hypothyroid patient is likely to forget some aspects of the treatment plan. A simpler regimen will be easier to understand until the patient is euthyroid.

The nurse is assessing a 41-year-old African American male patient diagnosed with a pituitary tumor causing panhypopituitarism. Assessment findings consistent with panhypopituitarism include a. high blood pressure. b. decreased facial hair. c. elevated blood glucose. d. tachycardia and cardiac palpitations.

ANS: B Changes in male secondary sex characteristics such as decreased facial hair, testicular atrophy, diminished spermatogenesis, loss of libido, impotence, and decreased muscle mass are associated with decreases in follicle stimulating hormone (FSH) and luteinizing hormone (LH). Fasting hypoglycemia and hypotension occur in panhypopituitarism as a result of decreases in adrenocorticotropic hormone (ACTH) and cortisol. Bradycardia is likely due to the decrease in thyroid stimulating hormone (TSH) and thyroid hormones associated with panhypopituitarism.

A 45-year-old male patient with suspected acromegaly is seen at the clinic. To assist in making the diagnosis, which question should the nurse ask? a. "Have you had a recent head injury?" b. "Do you have to wear larger shoes now?" c. "Is there a family history of acromegaly?" d. "Are you experiencing tremors or anxiety?"

ANS: B Acromegaly causes an enlargement of the hands and feet. Head injury and family history are not risk factors for acromegaly. Tremors and anxiety are not clinical manifestations of acromegaly.

The nurse is planning postoperative care for a patient who is being admitted to the surgical unit form the recovery room after transsphenoidal resection of a pituitary tumor. Which nursing action should be included? a. Palpate extremities for edema. b. Measure urine volume every hour. c. Check hematocrit every 2 hours for 8 hours. d. Monitor continuous pulse oximetry for 24 hours.

ANS: B After pituitary surgery, the patient is at risk for diabetes insipidus caused by cerebral edema. Monitoring of urine output and urine specific gravity is essential. Hemorrhage is not a common problem. There is no need to check the hematocrit hourly. The patient is at risk for dehydration, not volume overload. The patient is not at high risk for problems with oxygenation, and continuous pulse oximetry is not needed.

The nurse determines that demeclocycline (Declomycin) is effective for a patient with syndrome of inappropriate antidiuretic hormone (SIADH) based on finding that the patient's a. weight has increased. b. urinary output is increased. c. peripheral edema is decreased. d. urine specific gravity is increased.

ANS: B Demeclocycline blocks the action of antidiuretic hormone (ADH) on the renal tubules and increases urine output. An increase in weight or an increase in urine specific gravity indicates that the SIADH is not corrected. Peripheral edema does not occur with SIADH. A sudden weight gain without edema is a common clinical manifestation of this disorder.

The cardiac telemetry unit charge nurse receives status reports from other nursing units about four patients who need cardiac monitoring. Which patient should be transferred to the cardiac unit first? a. Patient with Hashimoto's thyroiditis and a heart rate of 102 b. Patient with tetany who has a new order for IV calcium chloride c. Patient with Cushing syndrome and a blood glucose of 140 mg/dL d. Patient with Addison's disease who takes hydrocortisone twice daily

ANS: B Emergency treatment of tetany requires IV administration of calcium; ECG monitoring will be required because cardiac arrest may occur if high calcium levels result from too-rapid administration. The information about the other patients indicates that they are more stable than the patient with tetany.

Which nursing assessment of a 69-year-old patient is most important to make during initiation of thyroid replacement with levothyroxine (Synthroid)? a. Fluid balance b. Apical pulse rate c. Nutritional intake d. Orientation and alertness

ANS: B In older patients, initiation of levothyroxine therapy can increase myocardial oxygen demand and cause angina or dysrhythmias. The medication also is expected to improve mental status and fluid balance and will increase metabolic rate and nutritional needs, but these changes will not result in potentially life-threatening complications.

After receiving change-of-shift report about the following four patients, which patient should the nurse assess first? a. A 31-year-old female with Cushing syndrome and a blood glucose level of 244 mg/dL b. A 70-year-old female taking levothyroxine (Synthroid) who has an irregular pulse of 134 c. A 53-year-old male who has Addison's disease and is due for a scheduled dose of hydrocortisone (Solu-Cortef). d. A 22-year-old male admitted with syndrome of inappropriate antidiuretic hormone (SIADH) who has a serum sodium level of 130 mEq/L

ANS: B Initiation of thyroid replacement in older adults may cause angina and cardiac dysrhythmias. The patient's high pulse rate needs rapid investigation by the nurse to assess for and intervene with any cardiac problems. The other patients also require nursing assessment and/or actions but are not at risk for life-threatening complications.

Which question will the nurse in the endocrine clinic ask to help determine a patient's risk factors for goiter? a. "How much milk do you drink?" b. "What medications are you taking?" c. "Are your immunizations up to date?" d. "Have you had any recent neck injuries?"

ANS: B Medications that contain thyroid-inhibiting substances can cause goiter. Milk intake, neck injury, and immunization history are not risk factors for goiter.

A patient who had radical neck surgery to remove a malignant tumor developed hypoparathyroidism. The nurse should plan to teach the patient about a. bisphosphonates to reduce bone demineralization. b. calcium supplements to normalize serum calcium levels. c. increasing fluid intake to decrease risk for nephrolithiasis. d. including whole grains in the diet to prevent constipation.

ANS: B Oral calcium supplements are used to maintain the serum calcium in normal range and prevent the complications of hypocalcemia. Whole grain foods decrease calcium absorption and will not be recommended. Bisphosphonates will lower serum calcium levels further by preventing calcium from being reabsorbed from bone. Kidney stones are not a complication of hypoparathyroidism and low calcium levels.

Which information will the nurse include when teaching a 50-year-old male patient about somatropin (Genotropin)? a. The medication will be needed for 3 to 6 months. b. Inject the medication subcutaneously every day. c. Blood glucose levels may decrease when taking the medication. d. Stop taking the medication if swelling of the hands or feet occurs.

ANS: B Somatropin is injected subcutaneously on a daily basis, preferably in the evening. The patient will need to continue on somatropin for life. If swelling or other common adverse effects occur, the health care provider should be notified. Growth hormone will increase blood glucose levels.

Which intervention will the nurse include in the plan of care for a 52-year-old male patient with syndrome of inappropriate antidiuretic hormone (SIADH)? a. Monitor for peripheral edema. b. Offer patient hard candies to suck on. c. Encourage fluids to 2 to 3 liters per day. d. Keep head of bed elevated to 30 degrees.

ANS: B Sucking on hard candies decreases thirst for a patient on fluid restriction. Patients with SIADH are on fluid restrictions of 800 to 1000 mL/day. Peripheral edema is not seen with SIADH. The head of the bed is elevated no more than 10 degrees to increase left atrial filling pressure and decrease antidiuretic hormone (ADH) release.

A client with Cushing's syndrome should be instructed to: A. Avoid alcoholic beverages B. Limit the sodium in her diet C. Increase servings of dark green vegetables D. Limit the amount of protein in her diet

Answer B is correct. A client with Cushing's syndrome has adrenocortical hypersecretion, so she retains sodium and water. The client may drink alcohol in moderation, so answer A is incorrect, and there is no need to eat more green vegetables or limit protein, so answers C and D are incorrect.

The nurse will plan to monitor a patient diagnosed with a pheochromocytoma for a. flushing. b. headache. c. bradycardia. d. hypoglycemia.

ANS: B The classic clinical manifestations of pheochromocytoma are hypertension, tachycardia, severe headache, diaphoresis, and abdominal or chest pain. Elevated blood glucose may also occur because of sympathetic nervous system stimulation. Bradycardia and flushing would not be expected.

A 37-year-old patient has just arrived in the postanesthesia recovery unit (PACU) after a thyroidectomy. Which information is most important to communicate to the surgeon? a. The patient reports 7/10 incisional pain. b. The patient has increasing neck swelling. c. The patient is sleepy and difficult to arouse. d. The patient's cardiac rate is 112 beats/minute.

ANS: B The neck swelling may lead to respiratory difficulty, and rapid intervention is needed to prevent airway obstruction. The incisional pain should be treated but is not unusual after surgery. A heart rate of 112 is not unusual in a patient who has been hyperthyroid and has just arrived in the PACU from surgery. Sleepiness in the immediate postoperative period is expected.

A 38-year-old male patient is admitted to the hospital in Addisonian crisis. Which patient statement supports a nursing diagnosis of ineffective self-health management related to lack of knowledge about management of Addison's disease? a. "I frequently eat at restaurants, and my food has a lot of added salt." b. "I had the stomach flu earlier this week, so I couldn't take the hydrocortisone." c. "I always double my dose of hydrocortisone on the days that I go for a long run." d. "I take twice as much hydrocortisone in the morning dose as I do in the afternoon."

ANS: B The need for hydrocortisone replacement is increased with stressors such as illness, and the patient needs to be taught to call the health care provider because medication and IV fluids and electrolytes may need to be given. The other patient statements indicate appropriate management of the Addison's disease.

Which nursing action will be included in the plan of care for a 55-year-old patient with Graves' disease who has exophthalmos? a. Place cold packs on the eyes to relieve pain and swelling. b. Elevate the head of the patient's bed to reduce periorbital fluid. c. Apply alternating eye patches to protect the corneas from irritation. d. Teach the patient to blink every few seconds to lubricate the corneas.

ANS: B The patient should sit upright as much as possible to promote fluid drainage from the periorbital area. With exophthalmos, the patient is unable to close the eyes completely to blink. Lubrication of the eyes, rather than eye patches, will protect the eyes from developing corneal scarring. The swelling of the eye is not caused by excessive blood flow to the eye, so cold packs will not be helpful.

A 63-year-old patient with primary hyperparathyroidism has a serum phosphorus level of 1.7 mg/dL (0.55 mmol/L) and calcium of 14 mg/dL (3.5 mmol/L). Which nursing action should be included in the plan of care? a. Restrict the patient to bed rest. b. Encourage 4000 mL of fluids daily. c. Institute routine seizure precautions. d. Assess for positive Chvostek's sign.

ANS: B The patient with hypercalcemia is at risk for kidney stones, which may be prevented by a high fluid intake. Seizure precautions and monitoring for Chvostek's or Trousseau's sign are appropriate for hypocalcemic patients. The patient should engage in weight-bearing exercise to decrease calcium loss from bone.

A patient who had a subtotal thyroidectomy earlier today develops laryngeal stridor and a cramp in the right hand upon returning to the surgical nursing unit. Which collaborative action will the nurse anticipate next? a. Suction the patient's airway. b. Administer IV calcium gluconate. c. Plan for emergency tracheostomy. d. Prepare for endotracheal intubation.

ANS: B The patient's clinical manifestations of stridor and cramping are consistent with tetany caused by hypocalcemia resulting from damage to the parathyroid glands during surgery. Endotracheal intubation or tracheostomy may be needed if the calcium does not resolve the stridor. Suctioning will not correct the stridor.

A 56-year-old patient who is disoriented and reports a headache and muscle cramps is hospitalized with possible syndrome of inappropriate antidiuretic hormone (SIADH). The nurse would expect the initial laboratory results to include a(n) a. elevated hematocrit. b. decreased serum sodium. c. low urine specific gravity. d. increased serum chloride.

ANS: B When water is retained, the serum sodium level will drop below normal, causing the clinical manifestations reported by the patient. The hematocrit will decrease because of the dilution caused by water retention. Urine will be more concentrated with a higher specific gravity. The serum chloride level will usually decrease along with the sodium level.

A 29-year-old woman with systemic lupus erythematosus has been prescribed 2 weeks of high-dose prednisone therapy. Which information about the prednisone is most important for the nurse to include? a. "Weigh yourself daily to monitor for weight gain caused by increased appetite." b. "A weight-bearing exercise program will help minimize the risk for osteoporosis." c. "The prednisone dose should be decreased gradually rather than stopped suddenly." d. "Call the health care provider if you experience mood alterations with the prednisone."

ANS: C Acute adrenal insufficiency may occur if exogenous corticosteroids are suddenly stopped. Mood alterations and weight gain are possible adverse effects of corticosteroid use, but these are not life-threatening effects. Osteoporosis occurs when patients take corticosteroids for longer periods.

The nurse is caring for a patient following an adrenalectomy. The highest priority in the immediate postoperative period is to a. protect the patient's skin. b. monitor for signs of infection. c. balance fluids and electrolytes. d. prevent emotional disturbances.

ANS: C After adrenalectomy, the patient is at risk for circulatory instability caused by fluctuating hormone levels, and the focus of care is to assess and maintain fluid and electrolyte status through the use of IV fluids and corticosteroids. The other goals are also important for the patient but are not as immediately life threatening as the circulatory collapse that can occur with fluid and electrolyte disturbances.

A patient has just arrived on the unit after a thyroidectomy. Which action should the nurse take first? a. Observe the dressing for bleeding. b. Check the blood pressure and pulse. c. Assess the patient's respiratory effort. d. Support the patient's head with pillows.

ANS: C Airway obstruction is a possible complication after thyroidectomy because of swelling or bleeding at the site or tetany. The priority nursing action is to assess the airway. The other actions are also part of the standard nursing care postthyroidectomy but are not as high of a priority.

After a 22-year-old female patient with a pituitary adenoma has had a hypophysectomy, the nurse will teach about the need for a. sodium restriction to prevent fluid retention. b. insulin to maintain normal blood glucose levels. c. oral corticosteroids to replace endogenous cortisol. d. chemotherapy to prevent malignant tumor recurrence.

ANS: C Antidiuretic hormone (ADH), cortisol, and thyroid hormone replacement will be needed for life after hypophysectomy. Without the effects of adrenocorticotropic hormone (ACTH) and cortisol, the blood glucose and serum sodium will be low unless cortisol is replaced. An adenoma is a benign tumor, and chemotherapy will not be needed.

Which finding by the nurse when assessing a patient with a large pituitary adenoma is most important to report to the health care provider? a. Changes in visual field b. Milk leaking from breasts c. Blood glucose 150 mg/dL d. Nausea and projectile vomiting

ANS: D Nausea and projectile vomiting may indicate increased intracranial pressure, which will require rapid actions for diagnosis and treatment. Changes in the visual field, elevated blood glucose, and galactorrhea are common with pituitary adenoma, but these do not require rapid action to prevent life-threatening complications.

Which information will the nurse teach a 48-year-old patient who has been newly diagnosed with Graves' disease? a. Exercise is contraindicated to avoid increasing metabolic rate. b. Restriction of iodine intake is needed to reduce thyroid activity. c. Antithyroid medications may take several months for full effect. d. Surgery will eventually be required to remove the thyroid gland.

ANS: C Medications used to block the synthesis of thyroid hormones may take 2 to 3 months before the full effect is seen. Large doses of iodine are used to inhibit the synthesis of thyroid hormones. Exercise using large muscle groups is encouraged to decrease the irritability and hyperactivity associated with high levels of thyroid hormones. Radioactive iodine is the most common treatment for Graves' disease although surgery may be used.

An expected nursing diagnosis for a 30-year-old patient admitted to the hospital with symptoms of diabetes insipidus is a. excess fluid volume related to intake greater than output. b. impaired gas exchange related to fluid retention in lungs. c. sleep pattern disturbance related to frequent waking to void. d. risk for impaired skin integrity related to generalized edema.

ANS: C Nocturia occurs as a result of the polyuria caused by diabetes insipidus. Edema, excess fluid volume, and fluid retention are not expected.

A 37-year-old patient is being admitted with a diagnosis of Cushing syndrome. Which findings will the nurse expect during the assessment? a. Chronically low blood pressure b. Bronzed appearance of the skin c. Purplish streaks on the abdomen d. Decreased axillary and pubic hair

ANS: C Purplish-red striae on the abdomen are a common clinical manifestation of Cushing syndrome. Hypotension and bronzed-appearing skin are manifestations of Addison's disease. Decreased axillary and pubic hair occur with androgen deficiency.

Which information obtained by the nurse in the endocrine clinic about a patient who has been taking prednisone (Deltasone) 40 mg daily for 3 weeks is most important to report to the health care provider? a. Patient's blood pressure is 148/94 mm Hg. b. Patient has bilateral 2+ pitting ankle edema. c. Patient stopped taking the medication 2 days ago. d. Patient has not been taking the prescribed vitamin D.

ANS: C Sudden cessation of corticosteroids after taking the medication for a week or more can lead to adrenal insufficiency, with problems such as severe hypotension and hypoglycemia. The patient will need immediate evaluation by the health care provider to prevent and/or treat adrenal insufficiency. The other information will also be reported, but does not require rapid treatment.

Which assessment finding of a 42-year-old patient who had a bilateral adrenalectomy requires the most rapid action by the nurse? a. The blood glucose is 176 mg/dL. b. The lungs have bibasilar crackles. c. The blood pressure (BP) is 88/50 mm Hg. d. The patient reports 5/10 incisional pain.

ANS: C The decreased BP indicates possible adrenal insufficiency. The nurse should immediately notify the health care provider so that corticosteroid medications can be administered. The nurse should also address the elevated glucose, incisional pain, and crackles with appropriate collaborative or nursing actions, but prevention and treatment of acute adrenal insufficiency is the priority after adrenalectomy.

A patient develops carpopedal spasms and tingling of the lips following a parathyroidectomy. Which action should the nurse take first? a. Administer the ordered muscle relaxant. b. Give the ordered oral calcium supplement. c. Have the patient rebreathe from a paper bag. d. Start the PRN oxygen at 2 L/min per cannula.

ANS: C The patient's symptoms suggest mild hypocalcemia. The symptoms of hypocalcemia will be temporarily reduced by having the patient breathe into a paper bag, which will raise the PaCO2 and create a more acidic pH. The muscle relaxant will have no impact on the ionized calcium level. Although severe hypocalcemia can cause laryngeal stridor, there is no indication that this patient is experiencing laryngeal stridor or needs oxygen. Calcium supplements will be given to normalize calcium levels quickly, but oral supplements will take time to be absorbed.

Which assessment finding for a 33-year-old female patient admitted with Graves' disease requires the most rapid intervention by the nurse? a. Bilateral exophthalmos b. Heart rate 136 beats/minute c. Temperature 103.8° F (40.4° C) d. Blood pressure 166/100 mm Hg

ANS: C The patient's temperature indicates that the patient may have thyrotoxic crisis and that interventions to lower the temperature are needed immediately. The other findings also require intervention but do not indicate potentially life-threatening complications.

A 62-year-old patient with hyperthyroidism is to be treated with radioactive iodine (RAI). The nurse instructs the patient a. about radioactive precautions to take with all body secretions. b. that symptoms of hyperthyroidism should be relieved in about a week. c. that symptoms of hypothyroidism may occur as the RAI therapy takes effect. d. to discontinue the antithyroid medications taken before the radioactive therapy.

ANS: C There is a high incidence of postradiation hypothyroidism after RAI, and the patient should be monitored for symptoms of hypothyroidism. RAI has a delayed response, with the maximum effect not seen for 2 to 3 months, and the patient will continue to take antithyroid medications during this time. The therapeutic dose of radioactive iodine is low enough that no radiation safety precautions are needed.

A 42-year-old female patient is scheduled for transsphenoidal hypophysectomy to treat a pituitary adenoma. During preoperative teaching, the nurse instructs the patient about the need to a. cough and deep breathe every 2 hours postoperatively. b. remain on bed rest for the first 48 hours after the surgery. c. avoid brushing teeth for at least 10 days after the surgery. d. be positioned flat with sandbags at the head postoperatively.

ANS: C To avoid disruption of the suture line, the patient should avoid brushing the teeth for 10 days after surgery. It is not necessary to remain on bed rest after this surgery. Coughing is discouraged because it may cause leakage of cerebrospinal fluid (CSF) from the suture line. The head of the bed should be elevated 30 degrees to reduce pressure on the sella turcica and decrease the risk for headaches.

An 82-year-old patient in a long-term care facility has several medications prescribed. After the patient is newly diagnosed with hypothyroidism, the nurse will need to consult with the health care provider before administering a. docusate (Colace). b. ibuprofen (Motrin). c. diazepam (Valium). d. cefoxitin (Mefoxin).

ANS: C Worsening of mental status and myxedema coma can be precipitated by the use of sedatives, especially in older adults. The nurse should discuss the use of diazepam with the health care provider before administration. The other medications may be given safely to the patient.

Which information is most important for the nurse to communicate rapidly to the health care provider about a patient admitted with possible syndrome of inappropriate antidiuretic hormone (SIADH)? a. The patient has a recent weight gain of 9 lb. b. The patient complains of dyspnea with activity. c. The patient has a urine specific gravity of 1.025. d. The patient has a serum sodium level of 118 mEq/L.

ANS: D A serum sodium of less than 120 mEq/L increases the risk for complications such as seizures and needs rapid correction. The other data are not unusual for a patient with SIADH and do not indicate the need for rapid action

A 23-year-old patient is admitted with diabetes insipidus. Which action will be most appropriate for the registered nurse (RN) to delegate to an experienced licensed practical/vocational nurse (LPN/LVN)? a. Titrate the infusion of 5% dextrose in water. b. Teach the patient how to use desmopressin (DDAVP) nasal spray. c. Assess the patient's hydration status every 8 hours. d. Administer subcutaneous DDAVP.

ANS: D Administration of medications is included in LPN/LVN education and scope of practice. Assessments, patient teaching, and titrating fluid infusions are more complex skills and should be done by the RN.

A 44-year-old female patient with Cushing syndrome is admitted for adrenalectomy. Which intervention by the nurse will be most helpful for a nursing diagnosis of disturbed body image related to changes in appearance? a. Reassure the patient that the physical changes are very common in patients with Cushing syndrome. b. Discuss the use of diet and exercise in controlling the weight gain associated with Cushing syndrome. c. Teach the patient that the metabolic impact of Cushing syndrome is of more importance than appearance. d. Remind the patient that most of the physical changes caused by Cushing syndrome will resolve after surgery.

ANS: D The most reassuring communication to the patient is that the physical and emotional changes caused by the Cushing syndrome will resolve after hormone levels return to normal postoperatively. Reassurance that the physical changes are expected or that there are more serious physiologic problems associated with Cushing syndrome are not therapeutic responses. The patient's physiological changes are caused by the high hormone levels, not by the patient's diet or exercise choices.

A client has just been admitted with a diagnosis of myxedema coma. If all of the following interventions were prescribed, the nurse should place highest priority on completing which action first?

Administering oxygen

What should be taught to a person with Addison's disease? check all that apply

Adrenal insufficiency=cause (adrenalectomy or coming off steroids too fast), hirsutism (hair distribution changes, avoid stress (infection, rm changes, or arguing), STEROIDS rest of life in am, Diet: Salt and decreased K (no salt substitutes), Immunocompromised.

A client is admitted for removal of a goiter. Which nursing intervention should receive priority during the post-operative period? A. Maintaining fluid and electrolyte balance B. Assessing the client's airway C. Providing needed nutrition and fluids D. Providing pain relief with narcotic analgesics

Answer B is correct. A goiter is hyperplasia of the thyroid gland. Removal of a goiter can result in laryngeal spasms and airway occlusion. The other answers are lesser in priority.

A client with acromegaly will most likely experience which symptom? A. Bone pain B. Frequent infections C. Fatigue D. Weight loss

Answer A is correct. Acromegaly is an increase in secretion of growth hormone. The growth hormones cause expansion and elongation of the bones. Answers B, C, and D are not directly associated with acromegaly, so they are incorrect.

The client with a suspected pituitary tumor will most likely exhibit symptoms of: A. Alteration in visual acuity B. Frequent diarrhea C. Alterations in blood glucose D. Urticaria

Answer A is correct. The pituitary is located in the middle of the skull adjacent to the optic nerve and brain. Pressure on the optic nerve can cause an increase in intracranial pressure. Clients frequently complain of headache, nausea, vomiting, and decreasing visual acuity as the intracranial pressure increases. B, C, and D are incorrect because they are not associated with a pituitary tumor.

A client is admitted for treatment of hypoparathyroidism. Based on the client's diagnosis, the nurse would anticipate an order for: A. Potassium B. Magnesium C. Calcium D. Iron

Answer C is correct. The parathyroid is responsible for calcium and phosphorus absorption. Clients with hypoparathyroidism have hypocalcemia. Answers A, B, and D are not associated with hypoparathyroidism therefore they are incorrect.

Which item should be kept at the bedside of a client who has just returned from having a thyroidectomy? A. A padded tongue B. An endotracheal tube C. An airway D. A tracheostomy set

Answer D is correct. Laryngeal swelling is not uncommon in clients following a thyroidectomy. A tracheostomy tray should be kept available. The ventilator is not necessary, so answer A is incorrect. The endotracheal tube is very difficult, if not impossible, to intubate if swelling has already occurred, so answer B is incorrect. The airway will do no good because the swelling is in the trachea, so answer C is incorrect.

The nurse is reviewing the postoperative prescriptions for a client who had a transsphenoidal hypophysectomy. Which health care provider's prescription noted on the record indicates the need for clarification?

Apply a loose dressing if any clear drainage is noted.

When caring for a patient with nephrogenic diabetes insipidus, the nurse would expect treatment to include a. fluid restriction b. thiazide diuretics c. a high sodium diet d. chlorprpamide (Diabinese)

B- In nephrogenic diabetes insipidus, the kidney is unable to respond to ADH, so vasopressin or hormone analogs are not effective. Thiazide diuretics slow the glomerular filtration rate in the kidney and produce a decrease in urine output. Low sodium diets (<3g/day) are also thought to decrease urine output. Fluids are not restricted because the patient could easily become dehydrated

A 26 year old female client presents with the symptom of unwanted facial hair. What of the following conditions is most likely? Answers: A. Graves' disease B. PCOS C. Hyperthyroidism D. Addison's disease

B PCOS is well known to cause hormonal irregularities in women which can result in hair growth.

The patient is brought to the ED following a car accident and is wearing medical identification that says she has Addison's disease. What should the nurse expect to be included in the collaborative care of this patient? A) Low sodium diet B) Increased glucocorticoid replacement C) Suppression of pituitary ACTH synthesis D) Elimination of mineralocorticoid replacement

B) Increased glucocorticoid replacement The patient with Addison's disease needs lifelong glucocorticoid and mineralocorticoid replacement and has an increased need with illness, injury, or stress, as this patient is experiencing. The patient with Addison's may also need a high sodium diet. Suppression of pituitary ACTH synthesis is done for Cushing syndrome. Elimination of mineralocorticoid replacement cannot be done for Addison's disease.

A patient with SIADH is treated with water restriction and administration of IV fluids. The nurse evaluates that treatment has been effective when the patient experiences a. increased urine output, decreased serum sodium, and increased urine specific gravity b. increased urine output, increased serum sodium, and decreased urine specific gravity c. decreased urine output, increased serum sodium, and decreased urine specific gravity d. decreased urine output, decreased serum sodium, and increased urine specific gravity

B- The patient with SIADH has water retention with hyponatremia, decreased urine output, and concentrated urine with high specific gravity. Improvement in the patient's condition is reflected by increased urine output, normalization of serum sodium, and more water in the urine, decreasing specific gravity.

The patient with systemic lupus erythematosus had been diagnosed with syndrome of inappropriate antidiuretic hormone (SIADH). What should the nurse expect to include in this patient's plan of care (select all that apply)? A) Obtain weekly weights. B) Limit fluids to 1000 mL per day. C) Monitor for signs of hypernatremia. D) Minimize turning and range of motion. E) Keep the head of the bed at 10 degrees or less elevation.

B) Limit fluids to 1000 mL per day. E) Keep the head of the bed at 10 degrees or less elevation. The care for the patient with SIADH will include limiting fluids to 1000 mL per day or less to decrease weight, increase osmolality, and improve symptoms; and keeping the head of the bed elevated at 10 degrees or less to enhance venous return to the heart and increase left atrial filling pressure, thereby reducing the release of ADH. The weights should be done daily along with intake and output. Signs of hyponatremia should be monitored, and frequent turning, positioning, and range-of-motion exercises are important to maintain skin integrity and joint mobility.

The nurse is providing discharge instructions to a patient with diabetes insipidus. Which instructions regarding desmopressin acetate (DDAVP) would be most appropriate? A) The patient can expect to experience weight loss resulting from increased diuresis. B) The patient should alternate nostrils during administration to prevent nasal irritation. C) The patient should monitor for symptoms of hypernatremia as a side effect of this drug. D) The patient should report any decrease in urinary elimination to the health care provider.

B) The patient should alternate nostrils during administration to prevent nasal irritation. DDAVP is used to treat diabetes insipidus by replacing the antidiuretic hormone that the patient is lacking. Inhaled DDAVP can cause nasal irritation, headache, nausea, and other signs of hyponatremia. Diuresis will be decreased and is expected, and hypernatremia should not occur.

A patient with hypothyroidism is treat with levothyroxin (Synthroid). When teaching the patient about the therapy, the nurse a. explains that alternate-day dosage may be used if side effects occur b. provides written instructions for all information related to the medication therapy c. assures the patient that a return to normal function will occur with replacement therapy d. informs the patient that medications must be taken until hormone balance in reestablished

B- Because of the mental sluggishness, inattentiveness, and memory loss that occurs with hypothyroidism, it is important to provide written instructions and repeat information when teaching the patient. Replacement therapy must be taken for life, and alternate-day dosing is not therapeutic. Although most patients return to a normal state with treatment, cardiovascular conditions and psychoses may persist.

What are the 3 Ps of DM?

Polyphagia, polydipsia, and polyuria (others= fatigue and increase UTIs)

Early this morning, a female client had a subtotal thyroidectomy. During evening rounds, nurse Tina assesses the client, who now has nausea, a temperature of 105° F (40.5° C), tachycardia, and extreme restlessness. What is the most likely cause of these signs? a. Diabetic ketoacidosis b. Thyroid crisis c. Hypoglycemia d. Tetany

B. Thyroid crisis usually occurs in the first 12 hours after thyroidectomy and causes exaggerated signs of hyperthyroidism, such as high fever, tachycardia, and extreme restlessness. Diabetic ketoacidosis is more likely to produce polyuria, polydipsia, and polyphagia; hypoglycemia, to produce weakness, tremors, profuse perspiration, and hunger. Tetany typically causes uncontrollable muscle spasms, stridor, cyanosis, and possibly asphyxia.

Nurse Oliver should expect a client with hypothyroidism to report which health concerns? a. Increased appetite and weight loss b. Puffiness of the face and hands c. Nervousness and tremors d. Thyroid gland swelling

B. Hypothyroidism (myxedema) causes facial puffiness, extremity edema, and weight gain. Signs and symptoms of hyperthyroidism (Graves' disease) include an increased appetite, weight loss, nervousness, tremors, and thyroid gland enlargement (goiter).

An appropriate nursing intervention for the patient with hyperparathyroidism is to a. pad side rails as a seizure precaution b. increase fluid intake to 3000 to 4000 mL daily c. maintain bed rest to prevent pathologic fractures d. monitor the patient for Trousseau's phenomenon and Chvostek's sign

B. A high fluid intake is indicated in hyperparathyroidism to dilute hypercalcemia and flush the kindeys so that calcium stone formation is reduced. Seizures are not associated with hyperparathyroidism, but impending tetany of hypoparathyroidism can be noted with Trousseau's phenomenon and Chvostek's sign. The patient with hyperparathyroidism is at risk for pathologic fractures resulting from decreased bone density, but mobility is encouraged to promote bone calcification.

You are the nursing student and you are taking care of the patient recently diagnosed with hyperthyroidism. You can visibly see the thryroid gland enlarged. What is the next thing you should assess? A) Assess for a bruit or a thrill B) Listen to respiratory rate and look at SpO2 C) Check for peripheral pulses D) Poke the neck to see if it is soft or hard

B. Because an enlarged thyroid gland can constrict the neck (often caused by a goiter) it is very important to monitor the patient's respiratory status. After that a bruit or thrill may be felt for over the distended thyroid gland. Palpating to see if the neck is soft or hard may also be done, as a hard nodule may be indicative thyroid cancer.

What should you remember when testing someone's blood glucose at the bedside?

Don't squeeze puncture site (dilutes w tissue fluid), choose outer edge of fingertip or earlobe for adult (heel for infant), hold pts finger in a dependent position and gently massage finger twd puncture site, clean with alcohol swab.

Which of the following would the nurse most expect to find in the patient with hypoparathyroidism? A) Hypertension and cramps in extremities B) Dysphagia and stiffness in hands or feet C) Positive Trousseau's sign and profuse sweating D) Increased Phosphorous levels and tremors

B. Dysphagia related to laryngeal spasms and stiffness or camping in extremities would be expected signs and symptoms of hypoparathyroidism. Hypertension, profuse sweating, and increased phosphorous levels would not be expected with this condition.

When the patient with parathyroid disease experiences symptoms of hypocalcemia, a measure that can be used to raise calcium levels temporarily is to a. administer IV normal saline b. have the patient rebreathe in a paper bag c. administer furosemid (Lasix) as ordered d. administer oral phosphorus supplements

B. Rebreathing in a paper bag promotes carbon dioxide retention in the blood, which lowers pH and creates acidosis. An acidemia enhances the solubility and ionization of calcium, increasing the proportion of total body calcium available in physiologically active form and relieving the symptoms of hypocalcemia. Saline promotes calcium excretion, as does furosemide. Phosphate levels in the blood are reciprocal to calcium, and an increase in phosphates promotes calcium excretion.

Uh oh, the patient records got scrambled up. You need to go in and see each of the patients in the clinic to see which problem they have. Which of the following patients does the nurse suspect to be the patient with hyperparathyroidism? A) The patient with shakiness in his hands and c/o numbness and tingling in fingers and toes B) The patient c/o back pain and abdominal cramping with nausea C) The patient with a BP of 167/87 and HR 110 with profuse sweating D) The patient c/o of fatigue and depression with bronze skin pigmentation

B. Remember, the patient with hyperparathyroidism is experiencing symptoms r/t too much calcium in the bloodstream. That means they will have issues with Stones (Renal), Bones (bone pain, joint pain, osteoporosis), Moans (GI upset, abdominal cramps), and psychic moans (mental changes). Patient A might be a patient with hypoparathyroidism. Patient C might be a patient with Pheochromocytoma. Patient D might be a patient with Addison's disease

The nurse is counseling the patient with hypoparathyroidism about nutrition choices r/t her disease process. Which of the following trays demonstrates correct understanding of the teaching? A) Yogurt, cream of chicken soup, and apple sauce with sweet tea B) Fried salmon, green beans, and a white wheat bread roll C) Sausage and egg sandwich with whole wheat bread and coffee with creamer D) Turkey sandwich with lettuce, tomato, fruit cup and a glass of milk

B. The recommended diet for patients with hypoparathyroidism includes foods that are high in calcium yet low in phosphorous (remember the balancing act) Fish, green beans, and white wheat are recommended foods because they are low in phosphorous. Milk and dairy products, while are high in calcium are also high in phosphorous and are thus restricted. Hypoparathyroidism is as much an issue with too much phosphorus as calcium (although s/s are r/t hypocalcemia) Other restricted foods include eggs and spinach. Other foods encouraged include broccoli, cucumbers, and nondairy creamer.

All of the following organs may be affected by multiple endocrine neoplasia type 1 except: a. Parathyroid glands b. Kidneys c. Pancreas and Duodenum d. Pituitary gland

B: Kidneys Multiple endocrine neoplasia type 1, also known as Werner's syndrome, is a heritable disorder that causes tumors in endocrine glands and the duodenum. Although the tumors associated with multiple endocrine neoplasia type 1 are generally benign, they can produce symptoms chemically by releasing excessive amounts of hormones or mechanically by pressing on adjacent tissue.

The most common benign tumor of the pituitary gland is a: a. Glioma b Prolactinoma c. Carcinoid tumor d. Islet cell tumor

B: Prolactinomas can cause symptoms by releasing excessive amounts of prolactin into the blood or mechanically by pressing on surrounding tissues. In women, symptoms may include menstrual irregularities and infertility; in men erectile dysfunction and libido may be impaired.

An ACTH stimulation test is commonly used to diagnose: a. Grave's disease b. Adrenal insufficiency and Addison's disease c. Cystic fibrosis d. Hashimoto's disease

B: The ACTH stimulation test measures blood and urine cortisol before and after injection of ACTH. Persons with chronic adrenal insufficiency or Addison's disease generally do not respond with the expected increase in cortisol levels. An abnormal ACTH stimulation test may be followed with a CRH stimulation test to pinpoint the cause of adrenal insufficiency.

A client is admitted with a diagnosis of pheochromocytoma. The nurse should monitor which parameter to detect the most common sign of pheochromocytoma?

Blood pressure elevation

The nurse is caring for a client after a thyroidectomy and monitoring for signs of thyroid storm. The nurse determines that which sign/symptom is indicative that a thyroid storm may be occurring?

Blood pressure of 80/60 mmHg

What should you suspect with a pregnant woman who is craving ice cubes?

Blood sugar imbalance (thirsty). Possibly gestational diabetes.

A client with a pituitary tumor will undergo transsphenoidal hypophysectomy. The nurse reinforces which information in the preoperative teaching plan for the client?

Blowing the nose following surgery is prohibited.

The nurse is caring for a client diagnosed with hyperparathyroidism who is prescribed furosemide (Lasix). The nurse reinforces dietary instructions to the client. Which is an appropriate instruction?

Drink at least 2 to 3 L of fluid daily.

A client presents with hypocalcemia, hyperphosphatemia, muscle cramps, and positive Trosseau's sign. What diagnosis does this support? Answers: A. Diabetes insipidus B. Conn's syndrome C. Hypoparathyroidism D. Acromegaly

C Hypoparathyroidism often leads to the symptoms mentioned. Conn's syndrome is an aldosterone-producing adenoma.

The nurse is caring for a patient admitted with suspected hyperparathyroidism. Because of the potential effects of this disease on electrolyte balance, the nurse should assess this patient for what manifestation? A) Neurologic irritability B) Declining urine output C) Lethargy and weakness D) Hyperactive bowel sounds

C) Lethargy and weakness Hyperparathyroidism can cause hypercalcemia. Signs of hypercalcemia include muscle weakness, polyuria, constipation, nausea and vomiting, lethargy, and memory impairment. Neurologic irritability, declining urine output, and hyperactive bowel sounds do not occur with hypercalcemia.

The surgeon was unable to spare a patient's parathyroid gland during a thyroidectomy. Which assessments should the nurse prioritize when providing postoperative care for this patient? A) Assessing the patient's white blood cell levels and assessing for infection B) Monitoring the patient's hemoglobin, hematocrit, and red blood cell levels C) Monitoring the patient's serum calcium levels and assessing for signs of hypocalcemia D) Monitoring the patient's level of consciousness and assessing for acute delirium or agitation

C) Monitoring the patient's serum calcium levels and assessing for signs of hypocalcemia Loss of the parathyroid gland is associated with hypocalcemia. Infection and anemia are not associated with loss of the parathyroid gland, whereas cognitive changes are less pronounced than the signs and symptoms of hypocalcemia.

What is a nursing priority in the care of a patient with a diagnosis of hypothyroidism? A) Providing a dark, low-stimulation environment B) Closely monitoring the patient's intake and output C) Patient teaching related to levothyroxine (Synthroid) D) Patient teaching related to radioactive iodine therapy

C) Patient teaching related to levothyroxine (Synthroid) A euthyroid state is most often achieved in patients with hypothyroidism by the administration of levothyroxine (Synthroid). It is not necessary to carefully monitor intake and output, and low stimulation and radioactive iodine therapy are indicated in the treatment of hyperthyroidism.

A patient has been taking oral prednisone for the past several weeks after having a severe reaction to poison ivy. The nurse has explained the procedure for gradual reduction rather than sudden cessation of the drug. What is the rationale for this approach to drug administration? A) Prevention of hypothyroidism B) Prevention of diabetes insipidus C) Prevention of adrenal insufficiency D) Prevention of cardiovascular complications

C) Prevention of adrenal insufficiency Sudden cessation of corticosteroid therapy can precipitate life-threatening adrenal insufficiency. Diabetes insipidus, hypothyroidism, and cardiovascular complications are not common consequences of suddenly stopping corticosteroid therapy.

The nurse educator is asking the nursing student to recall the signs/symptoms of hypothyroidism. The nurse educator determines that the student understands this disorder if which are included in the student's response? Select all that apply

Dry skin Constipation Cold intolerance

A patient is admitted to the hospital in thyrotoxic crisis. On physical assessment of the patient, the nurse would expect to find a. hoarseness and laryngeal stridor b. bulging eyeballs and dysrhythmias c. elevated temperature and signs of heart failure d. lethargy progressing suddenly to impairment of consciousness

C- A hyperthyroid crisis results in marked manifestations of hyperthyroidism, with severe tachycardia, heart failure, shock, hyperthermia, agitation, nausea, vomiting, diarrhea, delirium, and coma. Although exopthalmus may be present in the patient with Grave's disease, it is not a significant factor in hyperthyroid crisis. Hoarseness and laryngeal stridor are characteristic of tetany of hypoparathyroidism, and lethargy progressing to coma is characteristic of myxedema coma, a complication of hypothyroidism.

A patient with diabetes insipidus is treated with nasal demospressin (DDAVP). The nurse determines that the drug is not having an adequate therapeutic effect when the patient experiences a. headache and weight gain b nasal irritation and nausea c. urine specific gravity of 1.002 d. an oral intake greater than urinary output

C- Normal urine specific gravity of 1.003 to 1.030, and urine with a specific gravity of 1.002 is very dilute, indicating that there continues to be excessive water loss and that treatment of diabetes insipidus is inadequate. Headache, weight gain, and oral intake greater than urinary output are signs of volume excess that occur with over medication. Nasal irritation and nausea may also indicate overdosage

A female client with hypothyroidism (myxedema) is receiving levothyroxine (Synthroid), 25 mcg P.O. daily. Which finding should nurse Hans recognize as an adverse drug effect? a. Dysuria b. Leg cramps c. Tachycardia d. Blurred vision

C. Levothyroxine, a synthetic thyroid hormone, is given to a client with hypothyroidism to simulate the effects of thyroxine. Adverse effects of this agent include tachycardia. The other options aren't associated with levothyroxine.

An incoherent female client with a history of hypothyroidism is brought to the emergency department by the rescue squad. Physical and laboratory findings reveal hypothermia, hypoventilation, respiratory acidosis, bradycardia, hypotension, and nonpitting edema of the face and pretibial area. Knowing that these findings suggest severe hypothyroidism, nurse Libby prepares to take emergency action to prevent the potential complication of: a. Thyroid storm. b. Cretinism. c. myxedema coma. d. Hashimoto's thyroiditis.

C. Severe hypothyroidism may result in myxedema coma, in which a drastic drop in the metabolic rate causes decreased vital signs, hypoventilation (possibly leading to respiratory acidosis), and nonpitting edema. Thyroid storm is an acute complication of hyperthyroidism. Cretinism is a form of hypothyroidism that occurs in infants. Hashimoto's thyroiditis is a common chronic inflammatory disease of the thyroid gland in which autoimmune factors play a prominent role.

A male client is admitted for treatment of the syndrome of inappropriate antidiuretic hormone (SIADH). Which nursing intervention is appropriate? a. Infusing I.V. fluids rapidly as ordered b. Encouraging increased oral intake c. Restricting fluids d. Administering glucose-containing I.V. fluids as ordered

C. To reduce water retention in a client with the SIADH, the nurse should restrict fluids. Administering fluids by any route would further increase the client's already heightened fluid load.

A nursing student is studying for a test on care of the client with endocrine disorders. Which of the following statements demonstrates an understanding of the difference between hyperthyroidism and hypothyroidism? a. "Deficient amounts of TH cause abnormalities in lipid metabolism, with decreased serum cholesterol and triglyceride levels." b. "Graves' disease is the most common cause of hypothyroidism." c. "Decreased renal blood flow and glomerular filtration rate reduces the kidney's ability to excrete water, which may cause hyponatremia." d. "Increased amounts of TH cause a decrease in cardiac output and peripheral blood flow."

C. "Decreased renal blood flow and glomerular filtration rate reduces the kidney's ability to excrete water, which may cause hyponatremia." Rationale: # 1 is incorrect because deficient amounts of TH cause abnormalities in lipid metabolism with elevated serum cholesterol and triglyceride levels. # 2 is incorrect because Graves' disease is the most common cause of hyperthyroidism, not hypothyroidism. # 4 is incorrect because increased amounts of TH cause an increase in cardiac output and peripheral blood flow.

During discharge teaching for the patient with Addison's disease, the nurse identifies the need for additional instruction when the patient says, a. "I should always call the doctor is I develop vomiting or diarrhea" b. "If my weight goes down, my dosage of steroid is probably too high" c. "I should double or triple my steroid dose if I undergo rigorous physical exercise" d. "I need to carry an emergency kit with injectable hydrocortisone in case I can't take my medication by mouth"

C. A weight reduction in the patient with Addison's disease may indicate a fluid loss and a dose of replacement therapy that is too low rather than too high. Patient's with Addison's disease are taught to take two to three times their usual dose of steroids if they become ill, have teeth extracted, or engage in rigorous physical activity and should always have injectable hydrocortisone available if oral doses cannot be taken. Because vomiting and diarrhea are early signs of crisis and because fluid and electrolytes must be replaced, patients should notify their health care provider if these symptoms occur.

A patient is scheduled for a bilateral adrenalectomy. During the postoperative period, the nurse would expect administration of corticosteroids to be a. reduced to promote wound healing b. withheld until symptoms of hypocortisolism appear c. increased to promote an adequate response to the stress of surgery d. reduced because excessive hormones are released during surgical manipulation of the glands

C. Although the patient with Cushing syndrome has excess corticosteroids, removal of the glands the stress of surgery require that high doses of cortisone be administered postoperatively for several days before weaning the dose. The nurse should monitor the patient postoperatively to detect whether large amounts of hormones were released during surgical manipulation and to ensure that healing is satisfactory.

You the nurse are taking care of the patient with hypoparathyroidism. Based on your knowledge of the disease process. Which of the following do you expect to be your priority in planning care for this patient? A) Acute Pain B) Fluid Volume Deficit C) Ineffective Airway Maintenance D) Impaired Cerebral Perfusion

C. Due to bronchospasm that can be caused by decreased calcium levels, it is important for the nurse to assess airway and breathing pattern. Signs associated with hypoparathyroidism are d/t hypocalcemia and the number one sign is tetany.

The patient with Addison's disease presents to the clinic saying "I've gained five pounds in the past week!" You note some edema in both lower extremities. What does the nurse suspect to be the problem? A) Precipitation of Addisonian Crisis B) Heart Failure d/t end stage Addison's disease C) Excessive hormone replacement dose D) Adrenocarcinoma development

C. Signs of too high a corticosteroid dose include edema and weight gain. S/S of an Addisonian crisis include hypotension, tachycardia, cyanosis, pallor, and tachypnea. Heart failure is not associated with Addison's disease. There is no evidence of a tumor in the adrenal glands based on these symptoms.

A patient with hypoparathyroidism resulting form surgical treatment of hyperparathyroidism is preparing for discharge. The nurse teaches the patient that a. milk and milk products should be increased in the diet b. parenteral replacement of parathyroid hormone (PTH) will be required for life c. calcium supplements with vitamin D can effectively maintain calcium balance d. bran and whole-grain foods should be used to prevent GI effects of replacement therapy

C. The hypocalcemia that results from parathyroid hormone (PTH) deficiency is controlled with calcium and vitamin D supplementation and possibly oral phosphate binders. Replacement with PTH is not used because of antibody formation to PTH, the need for parenteral administration, and cost. Milk products, although good sources of calcium, also have high levels of phosphate, which reduce calcium absorption. Whole grains and foods containing oxalic acid also impair calcium absorption.

A patient with Addison's disease comes to the emergency department with complaints of nausea, vomiting, diarrhea, and fever. The nurse would expect collaborative care to include a. parenteral injections of adrenocorticotropic hormone (ACTH) b. IV administrations of vasopressors c. IV administration of hydrocortisone d. IV administration of D5W with 20 mEq KCL

C. Vomiting and diarrhea are early indicators of addisonian crisis, and fever indicates infection, which is causing additional stress for the patient. Treatment of a crisis requires immediate glucocorticoid replacement, and IV hydrocortisone, fluids, sodium, and glucose are necessary for 24 hours. Addison's disease is a primary insufficiency of the adrenal gland, and adrenocorticotropic hormone (ACTH) is not effective, nor would vasopressors be effective with fluid deficiency of Addison's disease. Potassium levels are increased in Addison's disease, and KCl would be contraindicated.

Grave's disease is: a. The most common cause of hypothyroidism b. The most common cause of hyperparathyroidism c. The most common cause of hyperthyroidism d. The most common cause of adrenal insufficiency

C: Grave's disease is an autoimmune disorder characterized by an enlarged thyroid gland and overproduction of thyroid hormones producing symptoms of hyperthyroidism such as rapid heartbeat, heat intolerance, agitation or irritability, weight loss, and trouble sleeping. It usually presents in persons age 20 to 40 and it is much more common in women than in men.

What is the treatment for hyperparathyroidism? a. Synthetic thyroid hormone b. Desiccated thyroid hormone c. Surgical removal of the glands d. Calcium and phosphate

C: When hyperparathyroidism requires treatment, surgery is the treatment of choice and is considered curative for 95% of cases. Because untreated hyperparathyroidism may elevate blood and urine levels of calcium and deplete phosphorus, bones and teeth may lose the minerals needed to remain strong.

Following hypophysectomy, a client complains of being very thirsty and having to urinate frequently. Which is the initial nursing action?

Check the urine specific gravity.

What sign should you assess for decreased calcium?

Chvosteks (face spasm) or trousseaus (arm spasm).

The nurse working on an endocrine nursing unit understands that which correct concept is used in planning care?

Clients who have hyperparathyroidism should be protected against falls.

Which signs/symptoms should the nurse expect to note when collecting data on a client with Addison's disease?

Hypotension and vomiting

The nurse is collecting data from a client who is being admitted to the hospital for a diagnostic workup for primary hyperparathyroidism. The nurse understands that which client complaint would be characteristic of this disorder?

Polyuria

The nurse should monitor for increases in which laboratory value for the patient as a result of being treated with dexamethasone (Decadron)? A) Sodium B) Calcium C) Potassium D) Blood glucose

D) Blood glucose Hyperglycemia or increased blood glucose level is an adverse effect of corticosteroid therapy. Sodium, calcium, and potassium levels are not directly affected by dexamethasone.

The patient with an adrenal hyperplasia is returning from surgery for an adrenalectomy. For what immediate postoperative risk should the nurse plan to monitor the patient? A) Vomiting B) Infection C) Thomboembolism D) Rapid BP changes

D) Rapid BP change The risk of hemorrhage is increased with surgery on the adrenal glands as well as large amounts of hormones being released in the circulation, which may produce hypertension and cause fluid and electrolyte imbalances to occur for the first 24 to 48 hours after surgery. Vomiting, infection, and thromboembolism may occur postoperatively with any surgery.

The nurse should expect to note which interventions in the plan of care for a client with hypothyroidism? Select all that apply.

Instruct the client about thyroid replacement therapy. Encourage the client to consume fluids and high-fiber foods in the diet. Instruct the client to contact the health care provider if episodes of chest pain occur.

In explaining the condition to a client, a nurse would say that Cushing's syndrome is caused primarily by: Answers: A. Low levels of glucocorticoids B. Excess secretion of sodium C. Autoimmunity in the pancreas D. Elevated levels of cortisol

D Cushing's syndrome is caused by elevated levels of cortisol. Glucocorticoids tend to cause this.

The doctor walks into the patient room and says. "You have pheochromocytoma" and leaves. The patient begins to cry and says, "What does that mean?" What is the best response by the nurse? A) It is a tumor in your pituitary gland. You are probably going to die. B) You seem upset. Tell me how you are feeling. C) Pheochromocytoma is a disorder of the adrenal glands where too little steroids are released into the blood stream D) This is a caused by a benign tumor in certain glands in your body located above your kidneys

D This is a caused by a benign tumor in certain glands in your body located above your kidneys

A patient with a severe pounding headache has been diagnosed with hypertension. However, the hypertension is not responding to traditional treatment. What should the nurse expect as the next step in management of this patient? A) Administration of β-blocker medications B) Abdominal palpation to search for a tumor C) Administration of potassium-sparing diuretics D) A 24-hour urine collection for fractionated metanephrines

D) A 24-hour urine collection for fractionated metanephrines Pheochromocytoma should be suspected when hypertension does not respond to traditional treatment. The 24-hour urine collection for fractionated metanephrines is simple and reliable with elevated values in 95% of people with pheochromocytoma. In a patient with pheochromocytoma preoperatively an α-adrenergic receptor blocker is used to reduce BP. Abdominal palpation is avoided to avoid a sudden release of catecholamines and severe hypertension. Potassium-sparing diuretics are not needed. Most likely they would be used for hyperaldosteronism, which is another cause of hypertension.

A 50-year-old female patient smokes, is getting a divorce, and is reporting eye problems. On assessment of this patient, the nurse notes exophthalmos. What other abnormal assessments should the nurse expect to find in this patient? A) Puffy face, decreased sweating, and dry hair B) Muscle aches and pains and slow movements C) Decreased appetite, increased thirst, and pallor D) Systolic hypertension and increased heart rate

D) Systolic hypertension and increased heart rate The patient's manifestations point to Graves' disease or hyperthyroidism, which would also include systolic hypertension and increased heart rate and increased thirst. Puffy face, decreased sweating; dry, coarse hair; muscle aches and pains and slow movements; decreased appetite and pallor are all manifestations of hypothyroidism.

A patient suspected of having acromegaly has an elevated plasma growth hormone (GH) level. In acromegaly, the nurse would also expect the patient's diagnostic results to include a. hyperinsulinemia b. a plasma glucose of < 70 mg/dL c. decreased GH levels with an oral glucose challenge test d. elevated serum somatomedin C (insulin like growth factor-1 (IGF-1))

D- A normal response to growth hormone (GH) secretion is stimulation of the liver to produce somatomedin C, or IGF-1, which stimulates growth of bones and soft tissues. The increased levels of somatomedin C normally inhibit GH, but in acromegaly, the pituitary gland secretes GH despite elevated IGF-1 levels. When both GH and IGF-1 levels are increased, overproduction of GH is confirmed. GH also causes elevation of blood glucose, and normally GH levels fall during an oral glucose challenge but not in acromegaly.

In a patient with central diabetes insipidus, administration of ADH during a water deprivation test will result in a(n) a. decrease in body weight b. increase in urinary output c. decrease in blood pressure d. increase in urine osmolality

D- A patient with diabetes insipidus has a deficiency of ADH with excessive loss of water from the kidney, hypovolemia, hypernatremia, and dilute urine with low specific gravity. When a vasopressin is administered, the symptoms are reversed, with water retention, decreased urinary output that increases urine osmolality, and an increase in BP

When replacement therapy is started for a patient with long-standing hypothyroidism, it is most important for the nurse to monitor the patient for a. insomnia b. weight loss c. nervousness d. dysrhythmias

D- All these manifestations may occur with treatment of hypothyroidism, but as a result of the effects of hypothyroidism on the cardiovascular system, when thyroid replacement therapy is started, myocardial oxygen consumption is increased, and resultant oxygen demand may cause angina, cardiac dysrhythmias, and heart failure

Causes of primary hypothyroidism in adults include a. malignant or benign thyroid nodules b. surgical removal or failure of pituitary gland c. surgical removal or radiation of thyroid gland d. autoimmune induced atrophy of the thryoid gland.

D- Both Grave's disease and Hashimoto's thyroiditis are autoimmune disorders that eventually destroy the thyroid gland, leading to primary hypothyroidism. Thyroid tumors most often result in hyperthyroidism. Secondary hyperthyroidism occurs as a result of pituitary failure, and iatrogenic hypothyroidism results from thyroidectomy or radiation of thyroid gland.

A patient with Grave's disease asks the nurse what caused the disorder. The best response by the nurse is, a. "The cause of Grave's disease is not known, although it is thought to be genetic" b. "It is usually associated with goiter formation from an iodine deficiency over a long period of time" c. "Antibodies develop against thyroid tissue and destroy it, causing a deficiency of thyroid hormones" d. "In genetically susceptible persons, antibodies are formed that cause excessive thyroid hormone secretion"

D- In Grave's disease, antibodies to the TSH receptor are formed, attach to the receptors, and stimulate the thyroid gland to release triiodothyronin (T3), thyroxine (T4), or both, creating hyperthyroidism. The disease is not directly genetic, but individuals appear to have a genetic susceptibility to develop autoimmune antibodies. Goiter formation from insufficient iodine intake is usually associated with hypothyroidism.

When providing discharge instructions to a patient following a subtotal thyroidectomy, the nurse advises the patient to a. never miss a daily dose of thyroid replacement therapy b. avoid regular exercises until thyroid function is normalized c. use warm saltwater gargles several times a day to relieve throat pain d. reduce caloric intake by at least half the amount taken before surgery

D- With the decrease in thyroid hormone postoperatively, calories need to be reduced substantially to prevent weight gain. When a patient has had a subtotal thyroidectomy, thyroid replacement therapy is not given because exogenous hormone inhibits pituitary production of TSH and delays or prevents the restoration of thyroid tissue regeneration. Regular exercise stimulates the thyroid gland and is encouraged. Saltwater gargles are use for dryness or irritation of the mouth and throat following radioactive iodine therapy.

The nurse is aware that the following is the most common cause of hyperaldosteronism? a. Excessive sodium intake b. A pituitary adenoma c. Deficient potassium intake d. An adrenal adenoma

D. An autonomous aldosterone-producing adenoma is the most common cause of hyperaldosteronism. Hyperplasia is the second most frequent cause. Aldosterone secretion is independent of sodium and potassium intake as well as of pituitary stimulation.

When assessing a male client with pheochromocytoma, a tumor of the adrenal medulla that secretes excessive catecholamine, nurse April is most likely to detect: a. a blood pressure of 130/70 mm Hg. b. a blood glucose level of 130 mg/dl. c. bradycardia. d. a blood pressure of 176/88 mm Hg.

D. Pheochromocytoma, a tumor of the adrenal medulla that secretes excessive catecholamine, causes hypertension, tachycardia, hyperglycemia, hypermetabolism, and weight loss. It isn't associated with the other options.

During preoperative teaching for a female client who will undergo subtotal thyroidectomy, the nurse should include which statement? a. "The head of your bed must remain flat for 24 hours after surgery." b. "You should avoid deep breathing and coughing after surgery." c. "You won't be able to swallow for the first day or two." d. "You must avoid hyperextending your neck after surgery."

D. To prevent undue pressure on the surgical incision after subtotal thyroidectomy, the nurse should advise the client to avoid hyperextending the neck. The client may elevate the head of the bed as desired and should perform deep breathing and coughing to help prevent pneumonia. Subtotal thyroidectomy doesn't affect swallowing.

A client with hypoparathyroidism has hypocalcemia. The nurse avoids giving the client the prescribed vitamin and calcium supplement with which liquid?

Milk

A 67-year-old male client has been complaining of sleeping more, increased urination, anorexia, weakness, irritability, depression, and bone pain that interferes with her going outdoors. Based on these assessment findings, nurse Richard would suspect which of the following disorders? a. Diabetes mellitus b. Diabetes insipidus c. Hypoparathyroidism d. Hyperparathyroidism

D. Hyperparathyroidism is most common in older women and is characterized by bone pain and weakness from excess parathyroid hormone (PTH). Clients also exhibit hypercaliuria-causing polyuria. While clients with diabetes mellitus and diabetes insipidus also have polyuria, they don't have bone pain and increased sleeping. Hypoparathyroidism is characterized by urinary frequency rather than polyuria.

The patient diagnosed with SIADH (Syndrome of inappropriate antidiuretic hormone) has a sodium level of 129 mEq/L. He is receiving IV 3% saline at 100 mL/hr. Which of the following is the nurse most concerned about? A) The patient complains of cramps in his hands B) There is trace edema in the lower extremities C) HR 110 D) The patient is suddenly states "Who are you?"

D. Decreased LOC is a sign of CMP (Central Pontine Myelinlysis) An extremely life-threatening condition that can occur with patient receiving high levels of sodium rich IV fluids. It is a medical emergency and the infusion should be stopped immediately. Cramps in the hands and trace edema are being treated with the infusion of 3% saline solution. HR 110 is also important, but not as emergent as sudden confusion.

You are providing discharge teaching to the client with primary Addison's disease. Which statement, if made by the patient, indicates the need for further teaching? A) I will have to take my Florinef and prednisone for the rest of my life B) During hot weather I will be sure to eat lots of salty foods C) I should continue to monitor my blood pressure for any drops D) Because I can't tolerate the heat I will cancel my trip to the Bahamas but I can still go skiing

D. Pt. should avoid cold weather in order to prevent precipitation of an Addisonian crisis which can be caused by stress on the body. The patient should be taught that medication will be taken for the rest of the patient's life. Salt intake should be increased during hot weather d/t hyponatremia. A drop in blood pressure could be a sign of an impending Addisonian crisis

All of the following are symptoms of Cushing's syndrome except: a. Severe fatigue and weakness b. Hypertension and elevated blood glucose c. A protruding hump between the shoulders d. Hair loss

D: Cushing's syndrome also may cause fragile, thin skin prone to bruises and stretch marks on the abdomen and thighs as well as excessive thirst and urination and mood changes such as depression and anxiety. Women who suffer from high levels of cortisol often have irregular menstrual cycles or amenorrhea and present with hair on their faces, necks, chests, abdomens, and thighs.

Which of the following conditions is caused by long-term exposure to high levels of cortisol? a. Addison's disease b. Crohn's disease c. Adrenal insufficiency d. Cushing's syndrome

D: Cushing's syndrome is a form of hypercortisolism. Risk factors for Cushing's syndrome are obesity, diabetes, and hypertension. Cushing's syndrome is most frequently diagnosed in persons ages 20 to 50 who have characteristic round faces, upper body obesity, large necks, and relatively thin limbs.

The nurse is collecting data on a client with a diagnosis of hypothyroidism. Which of these behaviors, if present in the client's history, should the nurse determine as being likely related to the symptoms of this disorder?

Depression

The nurse is caring for a child with a diagnosis of diabetes insipidus. The nurse anticipates that the health care provider will prescribe which medications?

Desmopressin acetate (DDAVP)

What electrolytes/ values is Addison low in?

Everything except K (Na, H20, BP, and Blood Sugar)

When caring for a client diagnosed with pheochromocytoma, which information should the nurse know when assisting with planning care?

Excessive catecholamines are released.

After several diagnostic tests, a client is diagnosed with diabetes insipidus. The nurse understands that which signs/symptoms are indicative of this disorder?

Excessive thirst and urine output

The nurse is monitoring a client with Graves' disease for signs of thyrotoxic crisis (thyroid storm). Which signs and symptoms noted in the client should alert the nurse to the presence of this crisis? Select all that apply.

Fever Sweating Agitation

The nurse is caring for a client following an adrenalectomy and is monitoring for signs of adrenal insufficiency. Which are signs and symptoms related to adrenal insufficiency? Select all that apply.

Fever Weakness Hypotension Mental status changes

The nurse is caring for a client with pheochromocytoma. The client asks for a snack and something warm to drink. Which is the appropriate choice for this client to meet nutritional needs?

Graham crackers and warm milk

What are the components of the food pyramid from greatest need to smallest?

Grains, veggies, fruits, milk, and meat&beans

What can cause diabetes insipidus?

Head trauma, brain tumor, punched in nose, or infection of the brain.

The nurse is preparing to reinforce instructions to a client with Addison's disease regarding diet therapy. The nurse understands that which diet should be prescribed for this client?

High-sodium, high-carbohydrate diet

Place the insulins in order of onset? NPH, Lantus, regular, humalog, lente

Humalog, regular, lente, NPH, and lantus

A nurse is reviewing discharge teaching with a client you has Cushing's syndrome. Which statement made by the client indicates that the instructions related to dietary management were understood

I can eat foods that contain potassium

A client is diagnosed with hyperparathyroidism. The nurse teaching the client about dietary alterations to manage the disorder tells the client to limit which food in the diet?

Ice cream

What does a cushings syndrome diet look like?

Increased protein & K but decreased calories, carbs, Na.

What should you have at the bedside in case adrenal insuffiency arises?

Inderal (lowers BP/stress).

The nurse is reinforcing discharge instructions to a client who had a unilateral adrenalectomy. Which information should be a component of the instructions?

Instructions about early signs of a wound infection

What is characteristic of type II diabetes?

Insufficient insulin production, adults after 40 mostly, was adult onset dm, familial but not dt obesity, may need insulin

A client has an endocrine system dysfunction of the pancreas. The nurse anticipates that the client will exhibit impaired secretion of which substance?

Insulin

What is pheochromocytoma? S&S? check all that apply

It is a tumor on the adrenal gland. S&S= HTN, tachycardia, palpitations, sweating, fever, and headaches.

What is acromegaly? What causes acromegaly? When does it occur?

It is large hands, ear, feet, etc. Hyperpituitary gland causes acromegaly. It occurs after 18 years of age bc bone thickness increases and cartilage is overproduced. Before 18 years of age=overproduction of growth hormone=giant.

A nurses caring for a postoperative parathyroidectomy client. Which of the following would require the nurses immediate attention

Laryngeal strider

The nurse is caring for a postoperative parathyroidectomy client. Which would require the nurse's immediate attention?

Laryngeal stridor

What are the S&S of hyperglycemia? check all that apply

Like being drunk! Extreme thirst, FREQUENT URINATION, dry skin, HUNGER, BLURRED VISION, drowsiness, and nausea.

What are the S&S of hypoglycemia? check all that apply

Like hangover. SHAKING, SWEATING, HUNGER, WEAKNESS/FATIGUE, HA, IRRITABLE, anxious, dizziness, fast heartbeat, and impaired vision.

A client has been diagnosed with hypoparathyroidism. Which food groups should be included in the diet?

Low in phosphorus and high in calcium

What are two complication of transphenoidal hypophysectomy?

Meningitis (check for clear fluid from nose, decrease stimulation, seizure precautions) and DIABETES INSIPIDUS (decrease ADH=decreased concentration=decreas SG<1.010).

After receiving furosemide (Lasix) 40 mg slow intravenous push for chest pain related to shortness of breath and generalized edema, the client responds poorly. The client has no relief of the chest pain, shortness of breath, or edema and only minimal urine output (less than 40 mL of urine). The health care provider is notified, and after reviewing the chart, suspects the client has syndrome of inappropriate antidiuretic hormone (SIADH). Which findings would lead to this specific diagnosis? Refer to chart.

Minimal responsiveness to furosemide (Lasix) and small cell lung cancer

The nurse reviews a plan of care for a postoperative client following a thyroidectomy and notes that the client is at risk for breathing difficulty. Which nursing intervention should the nurse include in the plan of care?

Monitor neck circumference frequently.

The nurse is reviewing a plan of care for a client with Addison's disease. The nurse notes that the client is at risk for dehydration and suggests nursing interventions that will prevent this occurrence. Which nursing intervention is an appropriate component of the plan of care? Select all that apply.

Monitoring intake and output Monitoring for changes in mental status Encouraging fluid intake of at least 3000 mL/day

What are the S&S of cushings syndrome? select all that apply

Moon face, hyperglycemia, purple striae, buffalo hump, GI distress, Na and fluid retention, everything increased except K (can have salt substitutes), and osteoporosis.

Can Cushing's patients have steroids?

NO because their blood sugar is already elevated.

The nurse is caring for a client with a diagnosis of hypoparathyroidism. The nurse reviews the client's laboratory results and notes that the calcium level is extremely low. The nurse should expect to note which sign/symptom on data collection?

Positive Trousseau's sign

The nurse is collecting data regarding a client after a thyroidectomy and notes that the client has developed hoarseness and a weak voice. Which nursing action is appropriate?

Reassure the client that this is usually a temporary condition.

What is a transphenoidal hypophysectomy?

Removal of pituitary gland by approach through the nose.

What if the medical tx for cushings is unsuccessful?

Remove adrenal glands and turn into addisons patient (can control addisons with steroids)

A health care provider has prescribed propylthiouracil (PTU) for a client with hyperthyroidism, and the nurse assists in developing a plan of care for the client. Which nursing measure would be included in the plan regarding this medication?

Signs and symptoms of hypothyroidism

The nurse is caring for a client with hypothyroidism who is overweight. Which food items should the nurse suggest to include in the plan?

Skim milk, apples, whole-grain bread, and cereal

What changes would you see in someone who has developed Type II diabetes?

Slow onset, weight gain, and eye problems.

The client diagnosed with Cushing's disease has developed 1 peripheral edema. Theclient has received intravenous fluids at 100 mL/hr via IV pump for the past 79 hours. The client received IVPB medication in 50 mL of fluid every 6 hours for 15 doses. How many mL of fluid did the client receive? ________

The client has received 8,650 mL of intravenous fluid.

The nurse is caring for a client following a thyroidectomy. The client tells the nurse that she is concerned because of voice hoarseness. The client asks the nurse whether the hoarseness will subside. Which statement by the nurse regarding the hoarseness is accurate?

The hoarseness is normal and will gradually subside.

An older client with a history of hyperparathyroidism and severe osteoporosis is hospitalized. The nurse caring for the client plans to address which problem first?

The possibility of injury

The nurse is monitoring a client following a thyroidectomy for signs/symptoms of hypocalcemia. Which sign/symptom noted in the client indicates the presence of hypocalcemia?

Tingling around the mouth

The nurse caring for a client scheduled for a transsphenoidal hypophysectomy to remove a tumor in the pituitary gland assists in developing a plan of care for the client. The nurse suggests including which specific information in the preoperative teaching plan?

Toothbrushing will not be permitted for at least 2 weeks following surgery.

What should you have available post thyroidectomy?

Tracheostomy set in case an emergency airway is needed. (tight sutures so possible compromise)

What should you suspect if a 13 year old is wetting bed, eating alot, and not gaining weight?

Type 1 diabetes.

What type of DM is ketosis prone?

Type I

Which medication order should the nurse question in the client diagnosed with untreated hypothyroidism? 3. Sedatives

Untreated hypothyroidism is characterized by an increased susceptibility to the effects of most hypnotic and sedative agents; therefore, the nurse should question this medication.

What should you teach the patient about a i131 test?

Used to test thyroid function, can't have an iodine allergy, stop thyroid meds 1 week before test, can't be pregnant, MAKE SURE DONE BEFORE OTHER IODINE TESTS!

What will need life long replacement following transphenoidal hypophysectomy?

Vasopressin (suppresses pee), Cortisone, Thyroid, and Sex hormones

The nurse has reinforced dietary instructions to a client with a diagnosis of hypoparathyroidism. The nurse instructs the client to include which item in the diet?

Vegetables

A preoperative client is scheduled for adrenalectomy to remove a pheochromocytoma. The nurse should most closely monitor which assessment in the preoperative period?

Vital signs

The most important nursing intervention during the medical and surgical treatment of the patient with a pheochromocytoma is

b. monitoring blood pressure38 rationale- a pheochromocytoma is a catecholamine-producing tumor of the adrenal medulla, which may cause severe, episodic HTN;

A patient with primary hyperparathyroidism has a serum calcium level of 14 mg/dl (3.5 mmol/L), phosphorus of 1.7 mg/dl (0.55 mmol/L), serum creatinine of 2.2 mg/dl (194 mmol/L), and a high urine calcium. While the patient awaits surgery, the nurse should

d. encourage the pt to drink 4000 ml of fluid daily.

When teaching a patient with chronic SIADH about long-term management of the disorder, the nurse determines that additional instruction is needed when the patient says,

a. "I need to shop for foods that are low in sodium and avoid adding salt to foods." Rationale: Pts with SIADH are at risk for hyponatremia, and a sodium supplement may be prescribed.

A 22-year-old patient is being seen in the clinic with increased secretion of the anterior pituitary hormones. The nurse would expect the laboratory results to show a. increased urinary cortisol. b. decreased serum thyroxine. c. elevated serum aldosterone levels. d. low urinary catecholamines excretion.

a. increased urinary cortisol.

A patient with hyperthyroidism is treated with radioactive iodine (RAI) at a clinic. Before the patient is discharged, the nurse instructs the pt

a. to monitor for symptoms of hypothyroidism, such as easy bruising and cold intolerance.

Can an insulin pump be delegated to an LPN?

Yes

An African American woman with a history of breast cancer has panhypopituitarism from radiation therapy for primary pituitary tumors. Which medications should the nurse teach her about needing for the rest of her life (select all that apply)? a. Cortisol b. Vasopressin c. Sex hormones d. Levothyroxine (Synthroid) e. Growth hormone (somatropin [Omnitrope]) f. Dopamine agonists (bromocriptine [Parlodel])

a, b, d, e. With panhypopituitarism, lifetime hormone replacement is needed for cortisol, vasopressin, thyroid, and GH. Sex hormones will not be replaced because of the patient's history of breast cancer. Dopamine agonists will not be used because they reduce secretion of GH, which has already been achieved with the radiation.

Which characteristics describe the use of RAI (select all that apply)? a. Often causes hypothyroidism over time b. Decreases release of thyroid hormones c. Blocks peripheral conversion of T4 to T3 d. Treatment of choice in nonpregnant adults e. Decreases thyroid secretion by damaging thyroid gland f. Often used with iodine to produce euthyroid before surgery

a, d, e. RAI causes hypothyroidism over time by damaging thyroid tissue and is the treatment of choice for nonpregnant adults. Potassium iodide decreases the release of thyroid hormones and decreases the size of the thyroid gland preoperatively. Propylthiouracil (PTU) blocks peripheral conversion of T4 to T3 and may be used with iodine to produce a euthyroid state before surgery.

As a precaution for vocal cord paralysis from damage to the recurrent laryngeal nerve during thyroidectomy surgery, what equipment should be in the room in case it is needed for this emergency situation? a. Tracheostomy tray c. IV calcium gluconate b. Oxygen equipment d. Paper and pencil for communication

a. A tracheostomy tray is in the room to use if vocal cord paralysis occurs from recurrent laryngeal nerve damage or for laryngeal stridor from tetany. The oxygen equipment may be useful but will not improve oxygenation with vocal cord paralysis without a tracheostomy. IV calcium salts will be used if hypocalcemia occurs from parathyroid damage. The paper and pencil for communication may be helpful, especially if a tracheostomy is performed, but will not aid in emergency oxygenation of the patient.

A patient with acromegaly is treated with a transsphenoidal hypophysectomy. What should the nurse do postoperatively? a. Ensure that any clear nasal drainage is tested for glucose. b. Maintain the patient flat in bed to prevent cerebrospinal fluid (CSF) leakage. c. Assist the patient with toothbrushing every 4 hours to keep the surgical area clean. d. Encourage deep breathing, coughing, and turning to prevent respiratory complications.

a. A transsphenoidal hypophysectomy involves entry into the sella turcica through an incision in the upper lip and gingiva into the floor of the nose and the sphenoid sinuses. Postoperative clear nasal drainage with glucose content indicates cerebrospinal fluid (CSF) leakage from an open connection to the brain, putting the patient at risk for meningitis. After surgery, the patient is positioned with the head elevated to avoid pressure on the sella turcica. Coughing and straining are avoided to prevent increased intracranial pressure and CSF leakage. Although mouth care is required every 4 hours, toothbrushing should not be performed because injury to the suture line may occur.

A patient with acromegaly is treated with a transsphenoidal hypophysectomy. What should the nurse do postoperatively? a. Ensure that any clear nasal drainage is tested for glucose. b. Maintain the patient flat in bed to prevent cerebrospinal fluid (CSF) leakage. c. Assist the patient with toothbrushing every 4 hours to keep the surgical area clean. d. Encourage deep breathing, coughing, and turning to prevent respiratory complications.

a. A transsphenoidal hypophysectomy involves entry into the sella turcica through an incision in the upper lip and gingiva into the floor of the nose and the sphenoid sinuses. Postoperative clear nasal drainage with glucose content indicates cerebrospinal fluid (CSF) leakage from an open connection to the brain, putting the patient at risk for meningitis. After surgery, the patient is positioned with the head elevated to avoid pressure on the sella turcica. Coughing and straining are avoided to prevent increased intracranial pressure and CSF leakage. Although mouth care is required every 4 hours, toothbrushing should not be performed because injury to the suture line may occur.

Which statement accurately describes Graves' disease? a. Exophthalmos occurs in Graves' disease. b. It is an uncommon form of hyperthyroidism. c. Manifestations of hyperthyroidism occur from tissue desensitization to the sympathetic nervous system. d. Diagnostic testing in the patient with Graves' disease will reveal an increased thyroid-stimulating hormone (TSH) level.

a. Exophthalmos or protrusion of the eyeballs may occur in Graves' disease from increased fat deposits and fluid in the orbital tissues and ocular muscles, forcing the eyeballs outward. Graves' disease is the most common form of hyperthyroidism. Increased metabolic rate and sensitivity of the sympathetic nervous system lead to the clinical manifestations. Thyroid-stimulating hormone (TSH) level is decreased in Graves' disease.

The nurse is caring for a 63-year-old with a possible pituitary tumor who is scheduled for a computed tomography (CT) scan with contrast. Which information about the patient is most important to discuss with the health care provider before the test? a. History of renal insufficiency b. Complains of chronic headache c. Recent bilateral visual field loss d. Blood glucose level of 134 mg/dL

a. History of renal insufficiency

When caring for a patient with primary hyperaldosteronism, the nurse would question a health care provider's prescription for which drug? a. Furosemide (Lasix) c. Spironolactone (Aldactone) b. Amiloride (Midamor) d. Aminoglutethimide (Cytadren)

a. Hyperaldosteronism is an excess of aldosterone, which is manifested by sodium and water retention and potassium excretion. Furosemide is a potassium-wasting diuretic that would increase the potassium deficiency. Aminoglutethimide blocks aldosterone synthesis. Spironolactone and amiloride are potassium-sparing diuretics.

When caring for a patient with primary hyperaldosteronism, the nurse would question a health care provider's prescription for which drug? a. Furosemide (Lasix) b. Amiloride (Midamor) c. Spironolactone (Aldactone) d. Aminoglutethimide (Cytadren)

a. Hyperaldosteronism is an excess of aldosterone, which is manifested by sodium and water retention and potassium excretion. Furosemide is a potassium-wasting diuretic that would increase the potassium deficiency. Aminoglutethimide blocks aldosterone synthesis. Spironolactone and amiloride are potassium-sparing diuretics.

The nurse is caring for a client who is about to undergo an adrenalectomy. Which of the following Preoperative interventions is most appropriate for this client? a. Maintain careful use of medical and surgical asepsis when providing care and treatments. b. Teach the client about a diet high in sodium to correct any potential sodium imbalances preoperatively. c. Explain to the client that electrolytes and glucose levels will be measured postoperatively. d. Teach the client how to effectively cough and deep breathe once surgery is complete.

a. Maintain careful use of medical and surgical asepsis when providing care and treatments. Rationale: Use careful medical and surgical asepsis when providing care and treatments since Cortisol excess increases the risk of infection. # 2 is incorrect. Nutrition should be addressed preoperatively. Request a dietary consultation to discuss with the client about a diet high in vitamins and proteins. If hypokalemia exists, include foods high in potassium. Glucocorticoid excess increases catabolism. Vitamins and proteins are necessary for tissue repair and wound healing following surgery. # 3 is incorrect. Monitor the results of laboratory tests of electrolytes and glucose levels. Electrolyte and glucose imbalances are corrected

A nurse on a general medical-surgical unit is caring for a client with Cushing's syndrome. Which of the following statements is correct about the medication regimen for Cushing's syndrome? a. Mitotane is used to treat metastatic adrenal cancer. b. Aminogluthimide may be administered to clients with ectopic ACTH-secreting tumors before surgery is performed. c. Ketoconazole increases cortisol synthesis by the adrenal cortex. d. Somatostatin analog increases ACTH secretion in some clients.

a. Mitotane is used to treat metastatic adrenal cancer. Rationale: Mitotane directly suppresses activity of the adrenal cortex and decreases peripheral metabolism of corticosteroids. It is used to treat metastatic adrenal cancer. # 2 is incorrect because aminogluthimide may be administered to clients with ectopic ACTH-secreting tumors that cannot be surgically removed. # 3 is incorrect because ketoconazole inhibits, not increases, cortisol synthesis by the adrenal cortex. # 4 is incorrect because somatostatin suppresses, not increases, ACTH secretion.

The nurse is caring for a client with pheochromocytoma. Which of the following must be included in planning the nursing care for this client ? a. Monitor blood pressure frequently, assessing for hypertension. b. Assess only for physical stressors present. c. Collect a random urine sample. d. Prepare the client for chemotherapy to shrink the tumor.

a. Monitor blood pressure frequently, assessing for hypertension. Rationale: Pheochromocytomas are tumors of chromaffin tissues in the adrenal medulla. These tumors which are usually benign produce catecholamines (epinephrine or norepinephrine) that stimulate the sympathetic nervous system. Although many organs are affected, the most dangerous effects are peripheral vasoconstriction and increased cardiac rate and contractility with resultant paroxysmal hypertension. Systolic blood pressure may rise to 200 to 300 mmHg, the diastolic to 150 to 175 mmHg. # 1 is correct because the careful monitoring of blood pressure is essential. Attacks are often precipitated by physical, emotional, or environmental stimuli, so # 2 is incorrect because more than physical stressors are considered. This condition is life threatening and is usually treated with surgery as the preferred treatment. # 3 is incorrect because it is a random sample and not a 24 hour urine collection. Because catecholamine secretion is episodic, a 24-hour urine is a better surveillance method than serum catecholamines. (Pagana & Pagana, 2002). Surgical removal of the tumor(s) by adrenalectomy is the treatment of choice. # 4 is incorrect because surgery would be the treatment usually completed.

RN observes a nursing assistant (NA) caring for a patient after a hypophysectomy. Which action by the NA requires that the RN intervene?

a. The NA lowers the HOB to the flat position. R: HOB should be elevated about 30 degrees to decrease pressure on the sella turcica and avoid headaches.

A patient is admitted to the hospital with a diagnosis of Cushing syndrome. On physical assessment of the patient, what should the nurse expect to find? a. Hypertension, peripheral edema, and petechiae b. Weight loss, buffalo hump, and moon face with acne c. Abdominal and buttock striae, truncal obesity, and hypotension d. Anorexia, signs of dehydration, and hyperpigmentation of the skin

a. The effects of adrenocortical hormone excess, especially glucocorticoid excess, include weight gain from accumulation and redistribution of adipose tissue, sodium and water retention, glucose intolerance, protein wasting, loss of bone structure, loss of collagen, and capillary fragility leading to petechiae. Clinical manifestations of adrenocortical hormone deficiency include hypotension, dehydration, weight loss, and hyperpigmentation of the skin.

A patient is admitted to the hospital with a diagnosis of Cushing syndrome. On physical assessment of the patient, what should the nurse expect to find? a. Hypertension, peripheral edema, and petechiae b. Weight loss, buffalo hump, and moon face with acne c. Abdominal and buttock striae, truncal obesity, and hypotension d. Anorexia, signs of dehydration, and hyperpigmentation of the skin

a. The effects of adrenocortical hormone excess, especially glucocorticoid excess, include weight gain from accumulation and redistribution of adipose tissue, sodium and water retention, glucose intolerance, protein wasting,loss of bone structure, loss of collagen, and capillary fragility leading to petechiae. Clinical manifestations of adrenocortical hormone deficiency include hypotension, dehydration, weight loss, and hyperpigmentation of the skin.

During care of the patient with SIADH, what should the nurse do? a. Monitor neurologic status at least every 2 hours. b. Teach the patient receiving treatment with diuretics to restrict sodium intake. c. Keep the head of the bed elevated to prevent antidiuretic hormone (ADH) release. d. Notify the health care provider if the patient's blood pressure decreases more than 20 mm Hg from baseline.

a. The patient with syndrome of inappropriate antidiuretic hormone (SIADH) has marked dilutional hyponatremia and should be monitored for decreased neurologic function and seizures every 2 hours. Sodium intake is supplemented because of the hyponatremia and sodium loss caused by diuretics. ADH release is reduced by keeping the head of the bed flat to increase left atrial filling pressure. A reduction in blood pressure (BP) indicates a reduction in total fluid volume and is an expected outcome of treatment.

A pt is taking high doses of prednisone to control the symptoms of an acute exacerbation of systemic lupus erythematosus. When teaching the pt about use of prednisone, which information is most important for the nurse to include?

b. Do not stop taking the prednisone suddenly; it should be decreased gradually. R: Acute adrenal insufficiency may occur if exogenous glucocorticoids are suddenly stopped.

What preoperative instruction should the nurse give to the patient scheduled for a subtotal thyroidectomy? a. How to support the head with the hands when turning in bed b. Coughing should be avoided to prevent pressure on the incision c. Head and neck will need to remain immobile until the incision heals d. Any tingling around the lips or in the fingers after surgery is expected and temporary

a. To prevent strain on the suture line postoperatively, the patient's head must be manually supported while turning and moving in bed but range-of-motion exercises for the head and neck are also taught preoperatively to be gradually implemented after surgery. There is no contraindication for coughing and deep breathing and these should be carried out postoperatively. Tingling around the lips or fingers is a sign of hypocalcemia, which may occur if the parathyroid glands are inadvertently removed during surgery. This sign should be reported immediately.

What preoperative instruction should the nurse give to the patient scheduled for a subtotal thyroidectomy? a. How to support the head with the hands when turning in bed b. Coughing should be avoided to prevent pressure on the incisionm c. Head and neck will need to remain immobile until the incision heals d. Any tingling around the lips or in the fingers after surgery is expected and temporary

a. To prevent strain on the suture line postoperatively, the patient's head must be manually supported while turning and moving in bed but range-of-motion exercises for the head and neck are also taught preoperatively to be gradually implemented after surgery. There is no contraindication for coughing and deep breathing and these should be carried out postoperatively. Tingling around the lips or fingers is a sign of hypocalcemia, which may occur if the parathyroid glands are inadvertently removed during surgery. This sign should be reported immediately.

Identify the rationale for having the following items immediately available in the patient's room following thyroidectomy a. Tracheostomy tray b. Calcium salts for IV administration c. Oxygen equipment

a. To use in case airway obstruction occurs because of vocal cord paralysis from recurrent laryngeal nerve damage during surgery or laryngeal stridor occurs with tetany b. needed in case hypocalcemia occurs form parathyroid gland removal or damage during surgery, resulting in tetany c. in case of airway obstruction, laryngeal stridor or edema around trachea

After neck surgery, a patient develops hypoparathyroidism. The nurse should plan to teach the patient about

a. calcium supplementation to normalize serum calcium levels.

A patient with acromegaly is treated with a transphenoidal hypophysectomy. Postoperatively, the nurse a. ensures that any clear nasal drainage is tested for glucose

a. ensures that any clear nasal drainage is tested for glucose

A patient with symptoms of DI is admitted to the hospital for evaluation and treatment of the condition. An appropriate nursing diagnosis for the patient is

a. insomnia related to waking at night to void.

A nurse is preparing a diet plan for a 50 yr old with simple goiter. Which of the following should be included in the clients diet to decrease the enlargement of he thyroid gland? a. iodine b. sodium c. potassium d. calcium

a. iodine

During care of a patient with syndrome of inappropriate ADH (SIADH), the nurse should

a. monitor neurologic status Q2H or more often if needed Rationale- the pt with SIADH has marked dilution hyponatremia and should be monitored for decreased neurologic function and convulsions every 2 hours.

Following a transsphenoidal resection of a pituitary tumor, an important N assessment is

a. monitoring hourly urine output. Rationale- After pituitary surgery, the pt is at risk for diabetes insipidus

A patient is hospitalized with possible SIADH. The patient is confused and reports a headache, muscle cramps, and twitching. The nurse would expect the initial laboratory results to include a

a. serum sodium of 125 mEq/L (125 mmol/L).

A patient who is on corticosteroid therapy for treatment of an autoimmune disorder has the following additional drugs ordered. How is the need for these drugs related to the effects of corticosteroids a. Furosemide (Lasix) b. Pantoprazole (Protonix) c. Alendronate (Fosamax) d. Insulin e. Potassium

a. sodium and fluid retention because of mineralcorticoid effect b. gastrointestinal irritation with an increase in secretion of pepsin and hydrochloric acid c. corticosteroid induced osteoperosis

The nurse determines that the patient in acute adrenal insufficiency is responding favorably to treatment when

a. the patient appears alert and oriented rationale- confusion, irritability, disorientation, or depression is often present in the patient with Addison's

A patient has been diagnosed with hypoparathyroidism. What manifestations should the nurse expect to observe (select all that apply)? a. Skeletal pain b. Dry, scaly skin c. Personality changes d. Abdominal cramping e. Cardiac dysrhythmias f. Muscle spasms and stiffness

b, c, d, e, f. In hypoparathyroidism the patient has inadequate circulating parathyroid hormone (PTH) that leads to hypocalcemia from the inability to maintain serum calcium levels. With hypocalcemia there is muscle stiffness and spasms, which can lead to cardiac dysrhythmias and abdominal cramps. There can also be personality and visual changes and dry, scaly skin.

A pt with suspected acromegaly is seen at the clinic. To assist in making the diagnosis, which question should the nurse ask?

b. "Do you have to wear larger shoes now?"

A pt is admitted to the hospital in addisonian crisis 1 month after a diagnosis of Addison's disease. The nurse identifies the nursing diagnosis of ineffective therapeutic regimen management related to lack of knowledge of management of condition when the patient says,

b. "I had the stomach flu earlier this week and couldn't take the hydrocortisone."

The nurse has identified the nursing diagnosis of fatigue for a patient who is hypothyroid. What should the nurse do while caring for this patient? a. Monitor for changes in orientation, cognition, and behavior. b. Monitor for vital signs and cardiac rhythm response to activity. c. Monitor bowel movement frequency, consistency, shape, volume, and color. d. Assist in developing well-balanced meal plans consistent with level of energy expenditure.

b. Cardiorespiratory response to activity is important to monitor in this patient to determine the effect of activities and plan activity increases. Monitoring changes in orientation, cognition, and behavior are interventions for impaired memory. Monitoring bowels is needed to plan care for the patient with constipation. Assisting with meal planning will help the patient with imbalanced nutrition: more than body requirements to lose weight if needed.

A client newly diagnosed with Addison's disease is giving a return explanation of teaching done by the primary nurse. Which of the following statements indicates that further teaching is necessary? a. "I need to increase how much I drink each day." b. "I need to weigh myself if I think I am losing or gaining weight." c. "I need to maintain a diet high in sodium and low in potassium." d. "I need to take my medications each day."

b. "I need to weigh myself if I think I am losing or gaining weight." The client is at risk for ineffective therapeutic regimen management. Clients with Addison's disease must learn to provide lifelong self-care that involves varied components: medications, diet, and recognizing and responding to stress. Changes in lifestyle are difficult to maintain permanently. The client needs to take the medications on a daily basis. The client needs to perform daily weights to monitor for signs of dehydration. The client needs to maintain a diet high in sodium and low in potassium, as well as maintain an increased fluid intake. # 2 is incorrect because daily weights need to be performed instead of weighing when a problem is suspected.

What is an appropriate nursing intervention for the patient with hyperparathyroidism? a. Pad side rails as a seizure precaution. b. Increase fluid intake to 3000 to 4000 mL daily. c. Maintain bed rest to prevent pathologic fractures. d. Monitor the patient for Trousseau's and Chvostek's signs.

b. A high fluid intake is indicated in hyperparathyroidism to dilute the hypercalcemia and flush the kidneys so that calcium stone formation is reduced. Seizures are not associated with hyperparathyroidism. Impending tetany of hypoparathyroidism after parathyroidectomy can be noted with Trousseau's and Chvostek's signs. The patient with hyperparathyroidism is at risk for pathologic fractures resulting from decreased bone density but mobility is encouraged to promote bone calcification.

During discharge teaching for the patient with Addison's disease, which statement by the patient indicates that the nurse needs to do additional teaching? a. "I should always call the doctor if I develop vomiting or diarrhea." b. "If my weight goes down, my dosage of steroid is probably too high." c. "I should double or triple my steroid dose if I undergo rigorous physical exercise." d. "I need to carry an emergency kit with injectable hydrocortisone in case I can't take my medication by mouth."

b. A weight reduction in the patient with Addison's disease may indicate a fluid loss and a dose of replacement therapy that is too low rather than too high. Because vomiting and diarrhea are early signs of crisis and because fluid and electrolytes must be replaced, patients should notify their health care provider if these symptoms occur. Patients with Addison's disease are taught to take two to three times their usual dose of steroids if they become ill, have teeth extracted, or engage in rigorous physical activity and should always have injectable hydrocortisone available if oral doses cannot be taken.

During discharge teaching for the patient with Addison's disease, which statement by the patient indicates that the nurse needs to do additional teaching? a. "I should always call the doctor if I develop vomiting or diarrhea." b. "If my weight goes down, my dosage of steroid is probably too high." c. "I should double or triple my steroid dose if I undergo rigorous physical exercise." d. "I need to carry an emergency kit with injectable hydrocortisone in case I can't take my medication by mouth."

b. A weight reduction in the patient with Addison's disease may indicate a fluid loss and a dose of replacement therapy that is too low rather than too high. Because vomiting and diarrhea are early signs of crisis and because fluid and electrolytes must be replaced, patients should notify their health care provider if these symptoms occur. Patients with Addison's disease are taught to take two to three times their usual dose of steroids if they become ill, have teeth extracted, or engage in rigorous physical activity and should always have injectablehydrocortisone available if oral doses cannot be taken.

A 35-year-old female patient with a possible pituitary adenoma is scheduled for a computed tomography (CT) scan with contrast media. Which patient information is most important for the nurse to communicate to the health care provider before the test? a. Bilateral poor peripheral vision b. Allergies to iodine and shellfish c. Recent weight loss of 20 pounds d. Complaint of ongoing headaches

b. Allergies to iodine and shellfish

A 30-year-old patient seen in the emergency department for severe headache and acute confusion is found to have a serum sodium level of 118 mEq/L. The nurse will anticipate the need for which diagnostic test? a. Urinary 17-ketosteroids b. Antidiuretic hormone level c. Growth hormone stimulation test d. Adrenocorticotropic hormone level

b. Antidiuretic hormone level

A patient with hypothyroidism is treated with levothyroxine (Synthroid). What should the nurse include when teaching the patient about this therapy? a. Explain that alternate-day dosage may be used if side effects occur. b. Provide written instruction for all information related to the drug therapy. c. Assure the patient that a return to normal function will occur with replacement therapy. d. Inform the patient that the drug must be taken until the hormone balance is reestablished.

b. Because of the mental sluggishness, inattentiveness, and memory loss that occur with hypothyroidism, it is important to provide written instructions and repeat information when teaching the patient. Replacement therapy must be taken for life and alternate-day dosing is not therapeutic. Although most patients return to a normal state with treatment, cardiovascular conditions and psychoses may persist.

The nurse has identified the nursing diagnosis of fatigue for a patient who is hypothyroid. What should the nurse do while caring for this patient? a. Monitor for changes in orientation, cognition, and behavior. b. Monitor for vital signs and cardiac rhythm response to activity. c. Monitor bowel movement frequency, consistency, shape, volume, and color. d. Assist in developing well-balanced meal plans consistent with level of energy expenditure.

b. Cardiorespiratory response to activity is important to monitor in this patient to determine the effect of activities and plan activity increases. Monitoring changes in orientation, cognition, and behavior are interventions for impaired memory. Monitoring bowels is needed to plan care for the patient with constipation. Assisting with meal planning will help the patient with imbalanced nutrition: more than body requirements to lose weight if needed.

While hospitalized for a fractured femur, a 68-year-old pt is diagnosed with hypothyroidism. Which of these medications on the original admission orders will the nurse need to consult with the HCP about before it is administered?

b. Diazepam (Valium) R: Worsening of mental status and myxedema coma can be precipitated by the use of sedatives, especially in older adults.

When caring for a patient with nephrogenic diabetes insipidus, what should the nurse expect the treatment to include? a. Fluid restriction b. Thiazide diuretics c. A high-sodium diet d. Chlorpropamide (Diabinese)

b. In nephrogenic diabetes insipidus, the kidney is unable to respond to ADH, so vasopressin or hormone analogs are not effective. Thiazide diuretics slow the glomerular filtration rate (GFR) in the kidney and produce a decrease in urine output. Low-sodium diets (<3 g/day) are also thought to decrease urine output. Fluids are not restricted because the patient could easily become dehydrated.

When caring for a patient with nephrogenic diabetes insipidus, what should the nurse expect the treatment to include? a. Fluid restriction b. Thiazide diuretics c. A high-sodium diet d. Chlorpropamide (Diabinese)

b. In nephrogenic diabetes insipidus, the kidney is unable to respond to ADH, so vasopressin or hormone analogs are not effective. Thiazide diuretics slow the glomerular filtration rate (GFR) in the kidney and produce a decrease in urine output. Low-sodium diets (<3 g/day) are also thought to decrease urine output. Fluids are not restricted because the patient could easily become dehydrated.

When developing a plan of care for a pt with SIADH, which interventions will the nurse include?

b. Offer patient hard candies to suck on. R: Sucking on hard candies decreases thirst for patient on a fluid restriction.

Priority Decision: What is the priority nursing intervention during the management of the patient with pheochromocytoma? a. Administering IV fluids b. Monitoring blood pressure c. Administering β-adrenergic blockers d. Monitoring intake and output and daily weights

b. Pheochromocytoma is a catecholamine-producing tumor of the adrenal medulla, which may cause severe, episodic hypertension; severe, pounding headache; and profuse sweating. Monitoring for a dangerously high BP before surgery is critical, as is monitoring for BP fluctuations during medical and surgical treatment.

When the patient with parathyroid disease experiences symptoms of hypocalcemia, what is a measure that can be used to temporarily raise serum calcium levels? a. Administer IV normal saline. b. Have patient rebreathe in a paper bag. c. Administer furosemide (Lasix) as ordered. d. Administer oral phosphorus supplements.

b. Rebreathing in a paper bag promotes carbon dioxide retention in the blood, which lowers pH and creates an acidosis. An acidemia enhances the solubility and ionization of calcium, increasing the proportion of total body calcium available in physiologically active form and relieving the symptoms of hypocalcemia. Saline promotes calcium excretion, as does furosemide. Phosphate levels in the blood are reciprocal to calcium and an increase in phosphate promotes calcium excretion.

When the patient with parathyroid disease experiences symptoms of hypocalcemia, what is a measure that can be used to temporarily raise serum calcium levels? a. Administer IV normal saline. b. Have patient rebreathe in a paper bag. c. Administer furosemide (Lasix) as ordered. d. Administer oral phosphorus supplements.

b. Rebreathing in a paper bag promotes carbon dioxide retention in the blood, which lowers pH and creates an acidosis. An acidemia enhances the solubility and ionization of calcium, increasing the proportion of total body calcium available in physiologically active form and relieving the symptoms of hypocalcemia. Saline promotes calcium excretion, as does furosemide. Phosphate levels in the blood are reciprocal to calcium and an increase in phosphate promotes calcium excretion.

A client presents to the emergency room with a history of Graves' disease. The client reports having symptoms for a few days, but has not previously sought or received any additional treatment. The client also reports having had a cold a few days back. Which of the following interventions would be appropriate to implement for this client, based on the history and current symptoms? Select all that apply. a. Administer aspirin b. Replace intravenous fluids c. Induce shivering d. Relieve respiratory distress e. Administer a cooling blanket

b. Replace intravenous fluids c. Induce shivering d. Relieve respiratory distress e. Administer a cooling blanket Rationale: Thyroid storm (also called thyroid crisis) is an extreme state of hyperthyroidism that is rare today because of improved diagnosis and treatment methods (Porth, 2005). When it does occur, those affected are usually people with untreated hyperthyroidism (most often Graves' disease) and people with hyperthyroidism who have experienced a stressor, such as an infection, trauma. The rapid increase in metabolic rate that results from the excessive TH causes the manifestations of thyroid storm. The manifestations include hyperthermia, with body temperatures ranging from 102°F (39°C) to 106°F (41°C); tachycardia; systolic hypertension; and gastrointestinal symptoms (abdominal pain, vomiting, diarrhea). Agitation, restlessness, and tremors are common, progressing to confusion, psychosis, delirium, and seizures. The mortality rate is high. Rapid treatment of thyroid storm is essential to preserve life. Treatment includes cooling without aspirin (which increases free TH) or inducing shivering, replacing fluids, glucose, and electrolytes, relieving respiratory distress, stabilizing cardiovascular function, and reducing TH synthesis and secretion. #1 is incorrect because cooling happens without the use of aspirin. All of the other choices are correct.

Which action by a new registered nurse (RN) caring for a patient with a goiter and possible hyperthyroidism indicates that the charge nurse needs to do more teaching? a. The RN checks the blood pressure on both arms. b. The RN palpates the neck thoroughly to check thyroid size. c. The RN lowers the thermostat to decrease the temperature in the room. d. The RN orders nonmedicated eye drops to lubricate the patient's bulging eyes.

b. The RN palpates the neck thoroughly to check thyroid size.

The nurse is caring for a 45-year-old male patient during a water deprivation test. Which finding is most important for the nurse to communicate to the health care provider? a. The patient complains of intense thirst. b. The patient has a 5-lb (2.3 kg) weight loss. c. The patient's urine osmolality does not increase. d. The patient feels dizzy when sitting on the edge of the bed.

b. The patient has a 5-lb (2.3 kg) weight loss.

A patient with SIADH is treated with water restriction. What does the patient experience when the nurse determines that treatment has been effective? a. Increased urine output, decreased serum sodium, and increased urine specific gravity b. Increased urine output, increased serum sodium, and decreased urine specific gravity c. Decreased urine output, increased serum sodium, and decreased urine specific gravity d. Decreased urine output, decreased serum sodium, and increased urine specific gravity

b. The patient with SIADH has water retention with hyponatremia, decreased urine output, and concentrated urine with high specific gravity. Improvement in the patient's condition is reflected by increased urine output, normalization of serum sodium, and more water in the urine, thus decreasing the specific gravity.

When providing postoperative care for a patient who has had bilateral adrenalectomy, which assessment information obtained by the nurse is most important to communicate to HCP?

b. The patient's blood pressure is 102/50.

What medication is used with thyrotoxicosis to block the effects of the sympathetic nervous stimulation of the thyroid hormones? a. Potassium iodide b. Atenolol (Tenormin) c. Propylthiouracil (PTU) d. Radioactive iodine (RAI)

b. The β-adrenergic blocker atenolol is used to block the sympathetic nervous system stimulation by thyroid hormones. Potassium iodide is used to prepare the patient for thyroidectomy or for treatment of thyrotoxic crisis to inhibit the synthesis of thyroid hormones. Antithyroid medications inhibit the synthesis of thyroid hormones. Radioactive iodine (RAI) therapy destroys thyroid tissue, which limits thyroid hormone secretion.

What medication is used with thyrotoxicosis to block the effects of the sympathetic nervous stimulation of the thyroid hormones? a. Potassium iodide b. Atenolol (Tenormin) c. Propylthiouracil (PTU) d. Radioactive iodine (RAI)

b. The β-adrenergic blocker atenolol is used to block the sympathetic nervous system stimulation by thyroid hormones. Potassium iodide is used to prepare the patient for thyroidectomy or for treatment of thyrotoxic crisis to inhibit the synthesis of thyroid hormones. Antithyroid medications inhibit the synthesis of thyroid hormones. Radioactive iodine (RAI) therapy destroys thyroid tissue, which limits thyroid hormone secretion.

A patient with Addison's disease comes to the emergency department with complaints of nausea, vomiting, diarrhea, and fever. What collaborative care should the nurse expect? a. IV administration of vasopressors b. IV administration of hydrocortisone c. IV administration of D5W with 20 mEq KCl d. Parenteral injections of adrenocorticotropic hormone (ACTH)

b. Vomiting and diarrhea are early indicators of Addisonian crisis and fever indicates an infection, which is causing additional stress for the patient. Treatment of a crisis requires immediate glucocorticoid replacement and IV hydrocortisone, fluids, sodium, and glucose are necessary for 24 hours. Addison's disease is a primary insufficiency of the adrenal gland and adrenocorticotropic hormone (ACTH) is not effective, nor would vasopressors be effective with the fluid deficiency of Addison's disease. Potassium levels are increased in Addison's disease and KCl would be contraindicated.

A patient with Addison's disease comes to the emergency department with complaints of nausea, vomiting, diarrhea, and fever. What collaborative care should the nurse expect? a. IV administration of vasopressors b. IV administration of hydrocortisone c. IV administration of D5W with 20 mEq KCl d. Parenteral injections of adrenocorticotropic hormone (ACTH)

b. Vomiting and diarrhea are early indicators of Addisonian crisis and fever indicates an infection, which is causing additional stress for the patient. Treatment of a crisis requires immediate glucocorticoid replacement and IV hydrocortisone, fluids, sodium, and glucose are necessary for 24 hours. Addison's disease is a primary insufficiency of the adrenal gland and adrenocorticotropic hormone (ACTH) is not effective, nor would vasopressors be effective with the fluid deficiency of Addison's disease. Potassium levels are increased in Addison's disease and KCl would be contraindicated.

The patient is diagnosed with syndrome of inappropriate antidiuretic hormone (SIADH). What manifestation should the nurse expect to find? a. Decreased body weight b. Decreased urinary output c. Increased plasma osmolality d. Increased serum sodium levels

b. With increased antidiuretic hormone (ADH), the permeability of the renal distal tubules is increased, so water is reabsorbed into circulation. Decreased output of concentrated urine with increased urine osmolality and specific gravity occur. In addition, fluid retention with weight gain, serum hypoosmolality, dilutional hyponatremia, and hypochloremia occur.

The patient is diagnosed with syndrome of inappropriate antidiuretic hormone (SIADH). What manifestation should the nurse expect to find? a. Decreased body weight b. Decreased urinary output c. Increased plasma osmolality d. Increased serum sodium levels

b. With increased antidiuretic hormone (ADH), the permeability of the renal distal tubules is increased, so water is reabsorbed into circulation. Decreased output of concentrated urine with increased urine osmolality and specific gravity occur. In addition, fluid retention with weight gain, serum hypoosmolality, dilutional hyponatremia, and hypochloremia occur.

A few hours after returning to the surgical nursing unit, a patient who has undergone a subtotal thyroidectomy develops laryngeal stridor and a cramp in the right hand. The nurse anticipates that intervention will include

b. administration of IV calcium gluconate.

A pt is hospitalized with acute adrenal insufficiency. The nurse determines that the pt is responding favorably to treatment upon finding

b. decreasing serum potassium. R: CMs of Addison's disease include hyperkalemia

During the physical examination of a 36-year-old female, the nurse finds that the patient's thyroid gland cannot be palpated. The most appropriate action by the nurse is to a. palpate the patient's neck more deeply. b. document that the thyroid was nonpalpable. c. notify the health care provider immediately. d. teach the patient about thyroid hormone testing.

b. document that the thyroid was nonpalpable.

The nurse identifies a nursing dx of risk for injury: corneal ulceration related to inability to close the eyelids secondary to exophthalmos for a patient with Graves' disease. An appropriate nursing intervention for this problem is to reduce periorbital fluid.

b. elevate the head of the patient's bed to reduce periorbital fluid.

When the patient with parathyroid disease experiences symptoms of hypocalcemia, a measure that can be used to temporarily raise serum calcium levels is to

b. have the patient rebreathe in a paper bag

An appropriate nursing intervention for the patient with hyperparathyroidism is to

b. increase fluid intake to 3000 to 4000ml/day Rationale-A high fluid intake is indicated in hyperparathyroidism to dilute hypercalcemia and flush the kidneys so that calcium stone formation is reduced

A nurse will teach a patient who is scheduled to complete a 24-hour urine collection for 17-ketosteroids to a. insert and maintain a retention catheter. b. keep the specimen refrigerated or on ice. c. drink at least 3 L of fluid during the 24 hours. d. void and save that specimen to start the collection.

b. keep the specimen refrigerated or on ice.

When teaching a patient with newly diagnosed hypothyroidism about management of the condition, the nurse should

b. provide written handouts of all instructions for continued reference as the patient improves. R: Written instructions will be helpful to the patient because initially the hypothyroid patient may be unable to remember to take medications and other aspects of self-care.

A patient with hypothyroidism is treated with Synthroid. When teaching the patient about the therapy, the nurse

b. provides written instruction for all information related to the medication therapy rationale- because of the mental sluggishness, inattentiveness, and memory loss that occur with hypothyroidism, it is important to provide written instructions

A 29-year-old patient in the outpatient clinic will be scheduled for blood cortisol testing. Which instruction will the nurse provide? a. "Avoid adding any salt to your foods for 24 hours before the test." b. "You will need to lie down for 30 minutes before the blood is drawn." c. "Come to the laboratory to have the blood drawn early in the morning." d. "Do not have anything to eat or drink before the blood test is obtained."

c. "Come to the laboratory to have the blood drawn early in the morning."

During the nursing assessment of a patient with Graves' disease, the nurse notes a bounding, rapid pulse and systolic hypertension. Based on these assessment data, which question is important for the nurse to ask the patient?

c. "Do you ever have any chest pain?"

Which question will provide the most useful information to a nurse who is interviewing a patient about a possible thyroid disorder? a. "What methods do you use to help cope with stress?" b. "Have you experienced any blurring or double vision?" c. "Have you had a recent unplanned weight gain or loss?" d. "Do you have to get up at night to empty your bladder?"

c. "Have you had a recent unplanned weight gain or loss?"

A 72-year-old patient is diagnosed with hypothyroidism, and levothyroxine (Synthroid) is prescribed. During initiation of thyroid replacement for the patient, it is most important for the nurse to assess

c. cardiac function. R: In older patients, initiation of levothyroxine therapy can increase myocardial oxygen demand and cause angina or dysrhythmias.

A pt with Cushing syndrome is admitted to the hospital to have laparoscopic adrenalectomy. During the admission assessment, the patient tells the nurse, "The worst thing about this disease is how terrible I look. I feel awful about it." best response by the nurse is

c. "Most of the physical and mental changes caused by the disease will gradually improve after surgery." Rationale: The most reassuring communication to the patient is that the physical and emotional changes caused by the Cushing syndrome will resolve after hormone levels return to normal postoperatively.

A patient is admitted to the hospital with thyrotoxicosis. On physical assessment of the patient, what should the nurse expect to find? a. Hoarseness and laryngeal stridor b. Bulging eyeballs and dysrhythmias c. Elevated temperature and signs of heart failure d. Lethargy progressing suddenly to impairment of consciousness

c. A hyperthyroid crisis results in marked manifestations of hyperthyroidism, with severe tachycardia, heart failure, shock, hyperthermia, restlessness, irritability, abdominal pain, vomiting, diarrhea, delirium, and coma. Although exophthalmos may be present in the patient with Graves' disease, it is not a significant factor in hyperthyroid crisis. Hoarseness and laryngeal stridor are characteristic of the tetany of hypoparathyroidism and lethargy progressing to coma is characteristic of myxedema coma, a complication of hypothyroidism.

A patient is admitted to the hospital with thyrotoxicosis. On physical assessment of the patient, what should the nurse expect to find? a. Hoarseness and laryngeal stridor b. Bulging eyeballs and dysrhythmias c. Elevated temperature and signs of heart failure d. Lethargy progressing suddenly to impairment of consciousness

c. A hyperthyroid crisis results in marked manifestations of hyperthyroidism, with severe tachycardia, heart failure, shock, hyperthermia, restlessness, irritability, abdominal pain, vomiting, diarrhea, delirium, and coma. Although exophthalmos may be present in the patient with Graves' disease, it is not a significant factor in hyperthyroid crisis. Hoarseness and laryngeal stridor are characteristic of the tetany of hypoparathyroidism and lethargy progressing to coma is characteristic of myxedema coma, a complication of hypothyroidism.

A patient who is on corticosteroid therapy for treatment of an autoimmune disorder has the following additional drugs ordered. Which one is used to prevent corticosteroid-induced osteoporosis? a. Potassium b. Furosemide (Lasix) c. Alendronate (Fosamax) d. Pantoprazole (Protonix)

c. Alendronate (Fosamax) is used to prevent corticosteroid- induced osteoporosis. Potassium is used to prevent the mineralocorticoid effect of hypokalemia. Furosemide (Lasix) is used to decrease sodium and fluid retention from the mineralocorticoid effect. Pantoprazole (Protonix) is used to prevent gastrointestinal (GI) irritation from an increase in secretion of pepsin and hydrochloric acid.

A patient who is on corticosteroid therapy for treatment of an autoimmune disorder has the following additional drugs ordered. Which one is used to prevent corticosteroid-induced osteoporosis? a. Potassium b. Furosemide (Lasix) c. Alendronate (Fosamax) d. Pantoprazole (Protonix)

c. Alendronate (Fosamax) is used to prevent corticosteroidinduced osteoporosis. Potassium is used to prevent the mineralocorticoid effect of hypokalemia. Furosemide (Lasix) is used to decrease sodium and fluid retention from the mineralocorticoid effect. Pantoprazole (Protonix) is used to prevent gastrointestinal (GI) irritation from an increase in secretion of pepsin and hydrochloric acid.

A patient is scheduled for a bilateral adrenalectomy. During the postoperative period, what should the nurse expect related to the administration of corticosteroids? a. Reduced to promote wound healing b. Withheld until symptoms of hypocortisolism appear c. Increased to promote an adequate response to the stress of surgery d. Reduced because excessive hormones are released during surgical manipulation of adrenal glands

c. Although the patient with Cushing syndrome has excess corticosteroids, removal of the glands and the stress of surgery require that high doses of corticosteroids (cortisone) be administered postoperatively for several days before weaning the dose. The nurse should monitor the patient's vital signs postoperatively to detect whether large amounts of hormones were released during surgical manipulation, obtain morning urine specimens for cortisol measurement to evaluate the effectiveness of the surgery, and provide dressing changes with aseptic technique to avoid infection as usual inflammatory responses are suppressed.

A patient is scheduled for a bilateral adrenalectomy. During the postoperative period, what should the nurse expect related to the administration of corticosteroids? a. Reduced to promote wound healing b. Withheld until symptoms of hypocortisolism appear c. Increased to promote an adequate response to the stress of surgery d. Reduced because excessive hormones are released during surgical manipulation of adrenal glands

c. Although the patient with Cushing syndrome has excess corticosteroids, removal of the glands and the stress of surgery require that high doses of corticosteroids (cortisone) be administered postoperatively for several days before weaning the dose. The nurse should monitor the patient's vital signs postoperatively to detect whether large amounts of hormones were released during surgical manipulation, obtain morning urine specimens for cortisol measurement to evaluate the effectiveness of the surgery, and provide dressing changes with aseptic technique to avoid infection as usual inflammatory responses are suppressed.

The nurse is teaching the patient with hyperthyroidism the importance of stress management. She is teaching how to do deep breathing exercises and listens to the patient's thoughts on coping mechanism and spirituality. The nurse knows this is an important part of this patients plan because: a) Knowing how to cope with stress will help this patient be able to work through some of the difficult treatments of this disease b) Every patient needs to know how to cope with stress c) Extra stress with the patient with hyperthyroidism can lead to life threatening complications d) Listening to the patients thoughts instead of imposing one's own thoughts encourages the patient to become self-reliant

c. Extra stress on the patient with hyperthyroidism can precipitate a life-threatening condition known as thyroid storm. It is important for the nurse to explain procedures to lower patient anxiety to prevent this condition.

A patient with Addison's disease comes to the emergency department with complaints of N/V/D, and fever. The nurse would expect collaborative care to include

c. IV administration of hydrocortisone rationale- vomiting and diarrhea are early indicators of addisonian crisis and fever indicates an infection, which s causing additional stress for the patient. treatment of a crisis requires immediate glucocorticoid replacement,

Which of the following nursing implications is most important in a client being medicated for Addison's disease? a. Administer oral forms of the drug with food to minimize its ulcerogenic effect. b. Monitor capillary blood glucose for hypoglycemia in the diabetic client. c. Instruct the client to never abruptly discontinue the medication. d. Teach the client to consume a diet that is high in potassium, low in sodium, and high in protein.

c. Instruct the client to never abruptly discontinue the medication. Rationale: The primary medical treatment of Addison's disease is replacement of corticosteroids and mineralcorticoids, accompanied by increased sodium in the diet. The client needs to know the importance of maintaining a diet high is sodium and low in potassium. Medications should never be discontinued abruptly because crisis can ensue. Oral forms of the drug are given with food in Cushing's disease.

A 44-year-old patient is admitted with tetany. Which laboratory value should the nurse monitor? a. Total protein b. Blood glucose c. Ionized calcium d. Serum phosphate

c. Ionized calcium

A patient with diabetes insipidus is treated with nasal desmopressin acetate (DDAVP). The nurse determines that the drug is not having an adequate therapeutic effect when the patient experiences a. headache and weight gain. b. nasal irritation and nausea. c. a urine specific gravity of 1.002. d. an oral intake greater than urinary output.

c. Normal urine specific gravity is 1.005 to 1.025 and urine with a specific gravity of 1.002 is very dilute, indicating that there continues to be excessive loss of water and that treatment of diabetes insipidus is inadequate. Headache, weight gain, and oral intake greater than urinary output are signs of volume excess that occur with overmedication. Nasal irritation and nausea may also indicate overdosage.

A patient with diabetes insipidus is treated with nasal desmopressin acetate (DDAVP). The nurse determines that the drug is not having an adequate therapeutic effect when the patient experiences a. headache and weight gain. b. nasal irritation and nausea. c. a urine specific gravity of 1.002. d. an oral intake greater than urinary output.

c. Normal urine specific gravity is 1.005 to 1.025 and urine with a specific gravity of 1.002 is very dilute, indicating that there continues to be excessive loss of water and that treatment of diabetes insipidus is inadequate. Headache, weight gain, and oral intake greater than urinary output are signs of volume excess that occur with overmedication. Nasal irritation and nausea may also indicate overdosage.

The patient with diabetes insipidus is brought to the emergency department with confusion and dehydration after excretion of a large volume of urine today even though several liters of fluid were drunk. What is a diagnostic test that the nurse should expect to be done to help make a diagnosis? a. Blood glucose b. Serum sodium level c. Urine specific gravity d. Computed tomography (CT) of the head

c. Patients with diabetes insipidus (DI) excrete large amounts of urine with a specific gravity of less than 1.005. Blood glucose would be tested to diagnose diabetes mellitus. The serum sodium level is expected to be low with DI but is not diagnostic. To diagnose central DI a water deprivation test is required. Then a CT of the head may be done to determine the cause. Nephrogenic DI is differentiated from central DI with determination of the level of ADH after an analog of ADH is given.

The patient with diabetes insipidus is brought to the emergency department with confusion and dehydration after excretion of a large volume of urine today even though several liters of fluid were drunk. What is a diagnostic test that the nurse should expect to be done to help make a diagnosis? a. Blood glucose b. Serum sodium level c. Urine specific gravity d. Computed tomography (CT) of the head

c. Patients with diabetes insipidus (DI) excrete large amounts of urine with a specific gravity of less than 1.005. Blood glucose would be tested to diagnose diabetes mellitus. The serum sodium level is expected to be low with DI but is not diagnostic. To diagnose central DI a water deprivation test is required. Then a CT of the head may be done to determine the cause. Nephrogenic DI is differentiated from central DI with determination of the level of ADH after an analog of ADH is given.

A patient with mild iatrogenic Cushing syndrome is on an alternate-day regimen of corticosteroid therapy. What does the nurse explain to the patient about this regimen? a. It maintains normal adrenal hormone balance. b. It prevents ACTH release from the pituitary gland. c. It minimizes hypothalamic-pituitary-adrenal suppression. d. It provides a more effective therapeutic effect of the drug.

c. Taking corticosteroids on an alternate-day schedule for pharmacologic purposes is less likely to suppress ACTH production from the pituitary and prevent adrenal atrophy. Normal adrenal hormone balance is not maintained during glucocorticoid therapy because excessive exogenous hormone is used.

A patient with mild iatrogenic Cushing syndrome is on an alternate-day regimen of corticosteroid therapy. What does the nurse explain to the patient about this regimen? a. It maintains normal adrenal hormone balance. b. It prevents ACTH release from the pituitary gland. c. It minimizes hypothalamic-pituitary-adrenal suppression. d. It provides a more effective therapeutic effect of the drug.

c. Taking corticosteroids on an alternate-day schedule for pharmacologic purposes is less likely to suppress ACTH production from the pituitary and prevent adrenal atrophy. Normal adrenal hormone balance is not maintained during glucocorticoid therapy because excessive exogenous hormone is used.

A patient with hypoparathyroidism resulting from surgical treatment of hyperparathyroidism is preparing for discharge. What should the nurse teach the patient? a. Milk and milk products should be increased in the diet. b. Parenteral replacement of parathyroid hormone will be required for life. c. Calcium supplements with vitamin D can effectively maintain calcium balance. d. Bran and whole-grain foods should be used to prevent GI effects of replacement therapy.

c. The hypocalcemia that results from PTH deficiency is controlled with calcium and vitamin D supplementation and possibly oral phosphate binders. Replacement with PTH is not used because of antibody formation to PTH, the need for parenteral administration, and cost. Milk products, although good sources of calcium, also have high levels of phosphate, which reduce calcium absorption. Whole grains and foods containing oxalic acid also impair calcium absorption.

During assessment of the patient with acromegaly, what should the nurse expect the patient to report? a. Infertility b. Dry, irritated skin m c. Undesirable changes in appearance d. An increase in height of 2 to 3 inches a year

c. The increased production of GH in acromegaly causes an increase in thickness and width of bones and enlargement of soft tissues, resulting in marked changes in facial features, oily and coarse skin, and speech difficulties. Infertility is not a common finding because GH is usually the only pituitary hormone involved in acromegaly. Height is not increased in adults with GH excess because the epiphyses of the bones are closed.

While assessing a patient who has just arrived in the postanesthesia recovery unit (PACU) after a thyroidectomy, the nurse obtains these data. Which information is most important to communicate to the surgeon?

c. The patient has increasing swelling in the neck

Which additional information will the nurse need to consider when reviewing the laboratory results for a patient's total calcium level? a. The blood glucose is elevated. b. The phosphate level is normal. c. The serum albumin level is low. d. The magnesium level is normal.

c. The serum albumin level is low.

Which laboratory value should the nurse review to determine whether a patient's hypothyroidism is caused by a problem with the anterior pituitary gland or with the thyroid gland? a. Thyroxine (T4) level b. Triiodothyronine (T3) level c. Thyroid-stimulating hormone (TSH) level d. Thyrotropin-releasing hormone (TRH) level

c. Thyroid-stimulating hormone (TSH) level

The nurse will teach a patient to plan to minimize physical and emotional stress while the patient is undergoing a. a water deprivation test. b. testing for serum T3 and T4 levels. c. a 24-hour urine test for free cortisol. d. a radioactive iodine (I-131) uptake test.

c. a 24-hour urine test for free cortisol.

When providing discharge instructions to a patient following a subtotal thyroidectomy, the nurse advises the patient to c. avoid eating foods such as soybeans, turnips, and rutabagas

c. avoid eating foods such as soybeans, turnips, and rutabagas Rationale- the patient should avoid goitrogens, foods that inhibit thyroid, such as soybeans, turnips, rutabagas, and peanut skins.

To prevent complications in the patient with Cushing syndrome, the nurse monitors the patient for

c. cardiac arrhythmias rationale- electrolyte changes that occur in Cushing syndrome include sodium retention and potassium excretion by the kidney, resulting in hypokalemia

A nurse is caring for a client with Addison's disease. Which of the following nursing considerations should be employed when caring for this client? a. avoid sodium in the clients diet b. monitor and protect skin integrity c. document the specific gravity of urine d. monitor increases in blood pressure

c. document the specific gravity of urine

A 60-year-old patient is taking spironolactone (Aldactone), a drug that blocks the action of aldosterone on the kidney, for hypertension. The nurse will monitor for a. increased serum sodium. b. decreased urinary output. c. elevated serum potassium. d. evidence of fluid overload.

c. elevated serum potassium.

A patient is admitted to the hospital in thyrotoxic crisis. On physical assessment of the patient, the nurse would expect to find c. elevated temperature and signs of heart failure

c. elevated temperature and signs of heart failure rationale- a hyperthyroid crisis results in marked manifestations of hyperthyroidism, with fever tachycardia, heart failure, shock, hyperthermia, agitation, N/V/D, delirium, and coma.

Which of the following would be a nursing priority for a client just DX with Addison's disease? a. avioding unnecessary activity b. encouraging client to wear a med alert tag c. ensuring the client is adequatly hydrated d. explaining that the client will need life long hormone therapy

c. ensuring the client is adequatly hydrated

A pt with a possible pheochromocytoma is admitted to the hospital for evaluation and diagnostic testing. During an attack, the nurse will monitor for hypertension and

c. headache. R: The classic CMs of pheochromocytoma are hypertension, tachycardia, severe headache, diaphoresis, and abdominal or chest pain.

A patient is scheduled for bilateral adrenalectomy. During the postoperative period, the nurse would expect administration of corticosteroids to be

c. increased to promote an adequate response to the stress of surgery rationale- although the patient with Cushing syndrome has excess corticosteroids, removal of the glands and the stress of surgery require that high doses of cortisone

A patient with Cushing syndrome returns to the surgical unit following an adrenalectomy. During the initial postoperative period, the nurse gives the highest priority to

c. maintaining fluid and electrolyte status.

A patient has an adrenocortical adenoma causing hyperaldosteronism and is scheduled for laparoscopic surgery to remove the tumor. During care before surgery, the nurse should

c. monitor the blood pressure every 4 hours. R: HTN caused by Na retention is a common complication of hyperaldosteronism.

A patient with Graves' disease is prepared for surgery with drug therapy consisting of 4 weeks of propylthiouracil (PTU) and 10 days of iodine before surgery. When teaching the patient about the drugs, the nurse explains that the drugs are given preoperatively to

c. normalize metabolism and decrease the size and vascularity of the gland.

After a patient with a pituitary adenoma has had a hypophysectomy, the nurse will plan to do discharge teaching about the need for

c. oral corticosteroids to replace endogenous cortisol R: ADH, cortisol, and thyroid hormone replacement will be needed for life after hypophysectomy.

A patient with an antidiuretic hormone (ADH)-secreting small-cell cancer of the lung is treated with demeclocycline (Declomycin) to control the symptoms of syndrome of inappropriate secretion of antidiuretic hormone (SIADH). The nurse determines that the demeclocycline is effective upon finding that the

c. patient's urinary output is increased.

A nursing assessment of a patient with Cushing syndrome reveals that the patient has truncal obesity and thin arms and legs. An additional manifestation of Cushing syndrome that the nurse would expect to find is

c. purplish red streaks on the abdomen.

During assessment of the patient with acromegaly, the nurse would expect the patient to report c. undesirable changes in appearance

c. undesirable changes in appearance Rationale- the increased production of growth hormone in acromegaly causes an increase in thickness and width of bones and enlargement of soft tissues, resulting in marked changes in facial features, oily and coarse skin, and speech difficulties.

A Clinical Instructor is questioning a student nurse about disorders of the parathyroid glands. Which statement by the nursing student, would indicate the need for further teaching? a. "Hyperparathyroidism results in an increased release of calcium and phosphorus by bones, with resultant bone decalcification." b. "Hyperparathyroidism results in deposits in soft tissues and the formation of renal calculi." c. "Hypoparathyroidism results in impaired renal tubular regulation of calcium and phosphate." d. "Hypoparathyroidism results in decreased activation of vitamin D which then results in decreased absorption of calcium by the pancreas."

d. "Hypoparathyroidism results in decreased activation of vitamin D which then results in decreased absorption of calcium by the pancreas." Rationale: Choices 1, 2, and 3 are all correct statements. # 4 demonstrates a need for further teaching because hypoparathyroidism results in decreased activation of vitamin D which then results in decreased absorption of calcium by the intestines, not the pancreas.

Which statement by a 50-year-old female patient indicates to the nurse that further assessment of thyroid function may be necessary? a. "I notice my breasts are tender lately." b. "I am so thirsty that I drink all day long." c. "I get up several times at night to urinate." d. "I feel a lump in my throat when I swallow."

d. "I feel a lump in my throat when I swallow."

A patient with Grave's asks the nurse what caused the disorder. The best response by the nurse is d. "In genetically susceptible persons antibodies form that attack thyroid tissue and stimulate overproduction of thyroid hormones."

d. "In genetically susceptible persons antibodies form that attack thyroid tissue and stimulate overproduction of thyroid hormones."

An 18-year-old male patient with a small stature is scheduled for a growth hormone stimulation test. In preparation for the test, the nurse will obtain a. ice in a basin. b. glargine insulin. c. a cardiac monitor. d. 50% dextrose solution.

d. 50% dextrose solution.

After receiving change-of-shift report about these pts, which patient should nurse assess first?

d. A 70-year-old who recently started levothyroxine (Synthroid) to treat hypothyroidism and has an irregular pulse of 134. R: Initiation of thyroid replacement in older adults may cause angina and cardiac dysrhythmias.

In a patient with central diabetes insipidus, what will the administration of ADH during a water deprivation test result in? a. Decrease in body weight c. Decrease in blood pressure b. Increase in urinary output d. Increase in urine osmolality

d. A patient with central diabetes insipidus has a deficiency of ADH with excessive loss of water from the kidney, hypovolemia, hypernatremia, and dilute urine with a low specific gravity. When vasopressin is administered, the symptoms are reversed, with water retention, decreased urinary output that increases urine osmolality, and an increase in BP.

Priority Decision: When replacement therapy is started for a patient with long-standing hypothyroidism, what is most important for the nurse to monitor the patient for? a. Insomnia c. Nervousness b. Weight loss d. Dysrhythmias

d. All these manifestations may occur with treatment of hypothyroidism. However, as a result of the effects of hypothyroidism on the cardiovascular system, when thyroid replacement therapy is started myocardial oxygen consumption is increased and the resultant oxygen demand may cause angina, cardiac dysrhythmias, and heart failure, so monitoring for dysrhythmias is most important.

When replacement therapy is started for a patient with long-standing hypothyroidism, what is most important for the nurse to monitor the patient for? a. Insomnia b. Weight loss c. Nervousness d. Dysrhythmias

d. All these manifestations may occur with treatment of hypothyroidism. However, as a result of the effects of hypothyroidism on the cardiovascular system, when thyroid replacement therapy is started myocardial oxygen consumption is increased and the resultant oxygen demand may cause angina, cardiac dysrhythmias, and heart failure, so monitoring for dysrhythmias is most important.

A nurse on a surgical floor is caring for a post-operative client who has just had a subtotal thyroidectomy. Which of the following assessments should be completed first on the client? a. Assess for signs of tetany by checking for Chvostek's and Trousseau's signs b. Assess dressing (if present) and the area under the client's neck and shoulders for drainage. c. Administer analgesic pain medications as ordered, and monitor their effectiveness. d. Assess respiratory rate, rhythm, depth, and effort.

d. Assess respiratory rate, rhythm, depth, and effort. Rationale: All of the above assessments have importance, but airway and breathing in a client should always be addressed first when prioritizing care. Assess for signs of latent tetany due to calcium deficiency, including tingling of toes, fingers, and lips; muscular twitches; positive Chvostek's and Trousseau's signs; and decreased serum calcium levels. However, tetany may occur in 1 to 7 days after thyroidectomy so # 1 is not the highest priority. Assessing for hemorrhage is always important, but the danger of hemorrhage is greatest in the first 12 to 24 hours after surgery, and as this client is immediately post operative it is not the main concern at this time. Pain medication is important but according to Maslow, pain is a psychosocial need to be addressed after a physiologic need.

What is a cause of primary hypothyroidism in adults? a. Malignant or benign thyroid nodules b. Surgical removal or failure of the pituitary gland c. Surgical removal or radiation of the thyroid gland d. Autoimmune-induced atrophy of the thyroid gland

d. Both Graves' disease and Hashimoto's thyroiditis are autoimmune disorders that eventually destroy the thyroid gland, leading to primary hypothyroidism. Thyroid tumors most often result in hyperthyroidism. Secondary hypothyroidism occurs as a result of pituitary failure and iatrogenic hypothyroidism results from thyroidectomy or radiation of the thyroid gland.

What findings are commonly found in a patient with a prolactinoma? a. Gynecomastia in men b. Profuse menstruation in women c. Excess follicle-stimulating hormone (FSH) and luteinizing hormone (LH) d. Signs of increased intracranial pressure, including headache, nausea, and vomiting

d. Compression of the optic chiasm can cause visual problems as well as signs of increased intracranial pressure, including headache, nausea, and vomiting. About 30% of prolactinomas will have excess prolactin secretion with manifestations of impotence in men, galactorrhea or amenorrhea in women without relationship to pregnancy, and decreased libido in both men and women. There is decreased follicle-stimulating hormone (FSH) and luteinizing hormone (LH).

Physical changes of hypothyroidism that must be monitored when replacement therapy is started include

d. decreased cardiac contractility and coronary atherosclerosis rationale- hypothyroidism affects the heart in many ways, causing cardiomyopathy, coronary atherosclerosis, bradycardia, pericardial effusions, and weakened cardiac contractility.

A patient with Graves' disease asks the nurse what caused the disorder. What is the best response by the nurse? a. "The cause of Graves' disease is not known, although it is thought to be genetic." b. "It is usually associated with goiter formation from an iodine deficiency over a long period of time." c. "Antibodies develop against thyroid tissue and destroy it, causing a deficiency of thyroid hormones." d. "In genetically susceptible persons, antibodies are formed that cause excessive thyroid hormone secretion."

d. In Graves' disease, antibodies to the TSH receptor are formed, attach to the receptors, and stimulate the thyroid gland to release triiodothyronine (T3), thyroxine (T4), or both, creating hyperthyroidism. The disease is not directly genetic but individuals appear to have a genetic susceptibility to develop autoimmune antibodies. Goiter formation from insufficient iodine intake is usually associated with hypothyroidism.

What characteristic is related to Hashimoto's thyroiditis? a. Enlarged thyroid gland b. Viral-induced hyperthyroidism c. Bacterial or fungal infection of thyroid gland d. Chronic autoimmune thyroiditis with antibody destruction of thyroid tissue

d. In Hashimoto's thyroiditis, thyroid tissue is destroyed by autoimmune antibodies. An enlarged thyroid gland is a goiter. Viral-induced hyperthyroidism is subacute granulomatous thyroiditis. Acute thyroiditis is caused by bacterial or fungal infection.

A nurse is caring for a 60 yr old client affected with hypoparathyroidism. When checking the lab report, the nurse finds that the clients calcium level was very low. Which of the following vitamins regulates the calcium level in the body? a. A b. D c. E d. K

d. K

Which information obtained when caring for a pt who has just been admitted for evaluation of DI will be of greatest concern to the nurse?

d. The patient is confused and lethargic.

A patient who recently had a calcium oxalate renal stone had a bone density study, which showed a decrease in her bone density. What endocrine problem could this patient have? a. SIADH c. Cushing syndrome b. Hypothyroidism d. Hyperparathyroidism

d. The patient with hyperparathyroidism may have calcium nephrolithiasis, skeletal pain, decreased bone density, psychomotor retardation, or cardiac dysrhythmias. The other endocrine problems would not be related to calcium kidney stones or decreased bone density.

A pt is admitted with possible SIADH. Which information obtained by nurse is most important to communicate rapidly to health care provider?

d. The pt has a serum sodium level of 119 mEq/L.

When providing discharge instructions to a patient who had a subtotal thyroidectomy for hyperthyroidism, what should the nurse teach the patient? a. Never miss a daily dose of thyroid replacement therapy. b. Avoid regular exercise until thyroid function is normalized. c. Use warm saltwater gargles several times a day to relieve throat pain. d. Substantially reduce caloric intake compared to what was eaten before surgery.

d. With the decrease in thyroid hormone postoperatively, calories need to be reduced substantially to prevent weight gain. When a patient has had a subtotal thyroidectomy, thyroid replacement therapy is not given because exogenous hormone inhibits pituitary production of TSH and delays or prevents the restoration of thyroid tissue regeneration. Regular exercise stimulates the thyroid gland and is encouraged. Saltwater gargles are used for dryness and irritation of the mouth and throat following radioactive iodine therapy.

When providing discharge instructions to a patient who had a subtotal thyroidectomy for hyperthyroidism, what should the nurse teach the patient? a. Never miss a daily dose of thyroid replacement therapy. b. Avoid regular exercise until thyroid function is normalized. c. Use warm saltwater gargles several times a day to relieve throat pain. d. Substantially reduce caloric intake compared to what was eaten before surgery.

d. With the decrease in thyroid hormone postoperatively, calories need to be reduced substantially to prevent weight gain. When a patient has had a subtotal thyroidectomy, thyroid replacement therapy is not given because exogenous hormone inhibits pituitary production of TSH and delays or prevents the restoration of thyroid tissue regeneration. Regular exercise stimulates the thyroid gland and is encouraged. Saltwater gargles are used for dryness and irritation of the mouth and throat following radioactive iodine therapy.

A patient suspected of having acromegaly has an elevated plasma growth hormone level. In acromegaly, the nurse would also expect the patient's diagnostic results to include d. a serum somatomedin C (Insulin-like-growth-factor) of more than 300

d. a serum somatomedin C (Insulin-like-growth-factor) of more than 300 rationale- a normal response to growth hormone secretion is stimulation of the liver to produce somatomedin C which stimulates growth of bones and soft tissue.

When teaching a patient newly diagnosed with Graves' disease about the disorder, the nurse explains that

d. antithyroid medications may take several weeks to have an effect. R: Improvement usually begins in 1-2 wks w good results at 4-6 weeks.

Causes of primary hypothyroidism in adults include

d. autoimmune-induced atrophy of the gland rationale- both Graves disease and Hasimotos thyroiditis are autoimmune disorders that eventually destroy the thyroid gland, leading to primary hypothyroidism.

A 61-year-old female patient admitted with pneumonia has a total serum calcium level of 13.3 mg/dL (3.3 mmol/L). The nurse will anticipate the need to teach the patient about testing for _____ levels. a. calcitonin b. catecholamine c. thyroid hormone d. parathyroid hormone

d. parathyroid hormone

A patient with hypoparathyroidism receives instructions from the nurse regarding symptoms of hypocalcemia and hypercalcemia. The nurse teaches the patient that if mild symptoms of hypocalcemia occur, the patient should

d. rebreathe with a paper bag and then seek medical assistance.

A pt who uses every-other-day prednisone therapy for rheumatoid arthritis complains of not feeling as well on the non-prednisone days and asks nurse about taking prednisone daily instead. The best response to the pt is that

d. there is less effect on normal adrenal function when prednisone is taken every other day.

A client is admitted to the hospital with a medical DX of hyperthyroidism. When taking a history which information would be most significant? A. edema, intolerance to cold, lethargy b. peri-orbital edema, lethargy mask like face c. weight loss, intolerance to cold, muscle wasting d. weight loss, intolerance to heat, exophthalmos

d. weight loss, intolerance to heat, exophthalmos

demopressin acetate (DDVAP) is prescribed for the treatment of diabetes insipidus. The nurse monitors the client after medication administration for which therapeutic response

decreased urine output it (promotes renal conservation)

A client is admitted to the emergency department and they diagnose of myxedema coma is made. What nursing action with the nurse prepared to carry out initially

maintain a patent airway

adverse effects of long term corticosteroid therapy include

osteroperosis, moon face, increased risk of infection, weigt gain, blurred vision

an indication of hypocalcemia following a thyroidectomy

tingling in hands & feet (paresthesia); cool, clammy, nervous, tachycardia, anxiety, confusion

A nurses caring for a client after a thyroidectomy and notes that calcium gluconate is prescribed for the client. The nurse determines that this medicine has been prescribed to

treat hypoglycemic tetany


संबंधित स्टडी सेट्स

FIN 1115 01---Personal Finance Question Set #3: Financial Instruments & Institutions

View Set

A&P- Chapter 5: The Integumentary System

View Set

Interpersonal Communications Module 4 Quiz

View Set

BIO1010 - Chapter 2: Review Questions

View Set

Exam 1 - All Cards Class Combined

View Set

AP Psych Practice Test Questions

View Set

Module 2 - Virtual Microscopy Activity

View Set